You are on page 1of 185

1

G.R. No. L-32542 November 26, 1970


THE COMMISSIONER OF CUSTOMS and THE COLLECTOR OF CUSTOMS for the Port
of Manila, petitioners,
vs.
HON. FEDERICO C. ALIKPALA, in his capacity as Judge of the Court of First
Instance of Manila, Branch XXII, GONZALO SY and TOMAS Y. DE LEON,

MAKALINTAL, J.:

The Commissioner of Customs and the Collector of Customs for the port of
Manila have come to this Court on a petition for certiorari and prohibition with
preliminary injunction, to declare null and void and set aside certain orders of
respondent Court of First Instance of Manila, Judge Federico C. Alikpala
presiding, in Civil Case No. 80655 entitled "Gonzalo Sy, doing business under the
name and style of Gonzalo Sy Trading, and Tomas Y. de Leon, doing business
under the name and style of T. Y. de Leon Enterprises, petitioners, vs. The
Commissioner of Customs and the Collector of Customs, respondents." That
case was a petition for injunction with a prayer for a writ of preliminary
injunction.

The basic order complained of is that issued on August 26, 1970, which recites
the essential pertinent facts of the case and is reproduced as follows:

On August 11, 1970, the petitioners filed an action wherein it was


prayed that the Commissioner of Customs and the Collector of
Customs be restrained from carrying out the seizure and scheduled
auction sale of the fruits they imported from abroad and that the
said cargo be released to them under the surety bonds which they
have already submitted to respondent Collector of Customs.

On August 13, 1970, the Court issued an order setting the hearing of
the petition for the issuance of a writ of preliminary injunction on
August 19, 1970, and restraining the respondents, their agents,
representatives and attorneys in the meantime from carrying out the
scheduled auction sale of the fruits imported by the petitioners, until
further orders from the Court.

The respondents filed a motion to dissolve the restraining order and


an opposition to the issuance of a writ of preliminary injunction
invoking several grounds in support thereof.

It appears that the Collector of Customs of the port of Manila issued


several warrants of seizure and detention against the cargo of the
petitioners consisting of apples, lemons, oranges and grapes, on the
ground that they were imported in violation of Central Bank circulars
in relation to Section 2530-F of the Tariff and Customs Code. In due
time, the petitioners were notified of the seizure, but before they
could be heard, respondent Collector of Customs issued a notice of
2

sale of the imported fruits which was scheduled for sale on August
10, 1970.

The petitioners filed with the Court of Tax Appeals a petition seeking
a review of the action taken by the Collector of Customs of Manila
who ordered the seizure of the imported fresh fruits, with a prayer
that pending final determination of the case, a writ of preliminary
injunction be issued restraining the Commissioner of Customs and
Collector of Customs from carrying out the seizure. On August 12,
1970, said Court, however, denied the petition on the ground that it
had no jurisdiction over the subject matter thereof and to grant the
writ of preliminary injunction.

Counsel for the respondents admitted that the petitioners have not
been heard on the seizure proceedings and the imported cargo
have already been advertised for sale and the same would have
been sold had not this Court issued a restraining order. The first
question submitted for resolution is whether the Court has jurisdiction
over the subject matter of the petition and to issue the ancillary
remedy prayed for.

The question involved herein is not whether the imported fruits are
subject to seizure but whether the respondent Collector of Customs
of Manila acted in accordance with law in scheduling the sale
thereof without first giving the petitioners an opportunity to be heard.
In short, the question presented for resolution is whether there was
observance of due process and in the case of Nadeco vs. Collector
of Customs (G.R. No. L-19180, Oct. 31, 1963) the Supreme Court in
effect held that the Court of First Instance has jurisdiction over the
subject matter of the action.

The provision of Tariff and Customs Code relied upon by the


respondents in issuing the warrants of seizure is Section 2530-F (which
declares that articles of prohibited importation are subject to
forfeiture) in relation to circulars issued by the Central Bank of the
Philippines beginning March 10, 1970 prohibiting the issuance of
release certificates on no-dollar imports.

Section 2301 of the Tariff and Customs Code, however, provides that
upon making any seizure, the Collector shall issue a warrant for the
detention of the property, but if the owner or importer desires to
secure the release of the property for legitimate use, said official
may surrender it upon the filing of a sufficient bond, in an amount to
be fixed by him, conditioned for the payment of the appraised value
of the articles and/or any fine, expenses and costs which may be
adjudged in the case.

The Tariff and Customs Code further requires the Collector to give
the owner or importer of the property written notice of the seizure
and an opportunity to be heard in relation thereto (Section 2303)
and that properties under seizure shall not be sold except after
3

liability to sale shall have been established by proper administrative


or judicial proceedings in conformity with the provisions of said
Code.

Evidently, the respondent Collector of Customs should not have


ordered the sale at public auction of the imported fruits until after
the petitioners have been given an opportunity to be heard.
Moreover, they availed of the remedy granted them by Section
2301, which respondent Collector of Customs granted but required
the submission of a cash instead of a surety bond.

The statute under consideration (Section 2301, Tariff and Customs


Code) merely provided that the release would be upon the filing of
a sufficient "bond." The petitioners affirmed that they presented to
respondent Collector of Customs surety bonds conditioned for the
payment of the appraised value of the imported fruits and/or any
fine, expenses and costs which may be adjudged in the case. The
attention of the petitioners have not been called by the respondent
Collector of Customs to the "insufficiency" of the bonds nor did he
raise any question as to the solvency of the bonding company.

On the basis of the foregoing facts, the Court finds that the
petitioners are entitled to the relief prayed for. The imported goods
are perishable in nature and unless immediate relief is granted to
petitioners, irreparable damage may be caused to them and in the
event petitioners' contention would be upheld, the judgment that
may be subsequently rendered would become ineffectual.

WHEREFORE, upon filing of a bond in the sum of P500, subject to the


approval of the Court, let a writ of preliminary injunction be issued
enjoining the respondents, their agents, representatives and any
other person acting in their behalf from proceeding with the seizure
and sale at public auction of the imported fruits, until further orders
from this Court. The respondents are further directed to release
immediately the imported goods to the custody of the petitioners on
the strength of the surety bonds filed by them unless the respondents
file with this Court their objection to the sufficiency of said bonds,
which should be done within twenty-four (24) hours from notice of a
copy of this order.

SO ORDERED.

On September 23, 1970 this Court gave, due course to the present petition and
resolved to issue a restraining order "enjoining respondent Judge from
executing his order dated August 26, 1970 ... insofar as it directed the
petitioners herein from releasing to the custody of the respondents the
imported goods in question." In due time the respondents filed their answer to
the petition and subsequently both parties submitted their respective
memorandum in lieu of oral argument.
4

Three grounds are relied upon in the petition for the issuance of the writ prayed
for, namely:

1. Respondent Court has no jurisdiction over the subject matter of


the case; it follows that it does not have the authority to grant the
writ of preliminary injunction ordering the release of the imported
fruits in question.

2. Assuming, ad arguendo, that it has jurisdiction over the subject


matter of the case, respondent Court acted with grave abuse of
discretion amounting to lack of jurisdiction in granting the writ of
preliminary injunction despite the fact that the respondents'
complaint states no cause of action upon which the grant of
injunction may be predicated.

3. Respondent Court gravely abused its discretion amounting to


black of jurisdiction in insisting on the sufficiency of the bonds filed by
petitioners, undertaken by the Communications Insurance Co., Inc.
in the total amount of P513,865.46, (P513,866.06), despite the fact
that its writing capacity is P50,465.52 only.

For a proper understanding and resolution of the issues it is necessary to state


the facts in greater detail, as they appear from the pleadings and memoranda
submitted by the parties as well as from the different documents attached
thereto and marked as annexes.

We first take up the case of Gonzalo Sy Trading. On Nov. 19, 1968 this firm was
authorized by the Central Bank, under Monetary Board Revolution No. 2038, to
import fresh fruits from Japan to the extent of $350,000.00, on a no-dollar basis
and without letters of credit. As of November 1969 the amount of $144,306.15
had been used. On October 30 of that year Gonzalo Sy Trading asked the
Central Bank for an amendment of the terms of the aforesaid resolution so that
the importations authorized under it could be procured not only from Japan
but from other sources as well. On November 19, 1969 the Deputy Governor of
the Central Bank denied the request, and pointed out that Monetary Board
Resolution No. 2038 was intended only for the Christmas season of 1968 and did
not extend through 1969. Two days thereafter, however, or on November 21,
the Director of the Foreign Exchange Department of the Central Bank wrote
the Prudential Bank and Trust Company in connection with the release
certificates so far issued by it covering the no-dollar importations of fresh fruits
by its client, Gonzalo Sy Trading, and noting that only $144,306.15 had been
used out of the total amount of $350,000.00, authorized the Prudential Bank
and Trust Company to "continue to issue release certificates to cover the No-
Dollar importations of fresh fruits by your client, subject to the same terms and
conditions imposed by the Monetary Board under the above-mentioned
resolution." Pursuant to such authority Gonzalo Sy Trading continued importing
fresh fruits, until by the beginning of June 1970 the total amount already used
was $314,142.51, leaving a balance of $35,857.49.

On June 3, 1970, Gonzalo Sy Trading wrote a letter to the Central Bank, making
reference to a previous letter of May 27 requesting permission to utilize the said
5

balance to pay for two shipments of fresh fruits coming on June 4 and 6,
respectively. This request was denied by the Central Bank in its letter of June 10,
1970. On the following June 16 warrants of seizure and detention were issued
by the Collector of Customs after the customs duties, taxes and other charges
had been paid by the importer.

With respect to respondent Tomas T. de Leon, it appears that on many


occasions in the past he had always been allowed by the Central Bank to
import fresh fruits on a dollar co-assignment basis. The 1968 imports alone were
valued at over half a million dollars. The corresponding release certificates were
invariably authorized by said bank after the arrival of the shipments in the
Philippines. On November 20, 1969 De Leon filed the customary application
with the bank for the issuance of a "no-dollar import permit" to cover
consignments of fruits from suppliers abroad. Pending action on said
application, orders were placed and the shipments arrived during the months
of May through July 1970, and the customs duties, taxes and other charges
were also paid by the importer. As in the case of Gonzalo Sy Trading however,
the said shipments were seized by the Collector of Customs.

On July 30, 1970 the Collector of Customs issued a notice of auction sale of the
goods under seizure to be held on the following August 12 and every day
thereafter until terminated. On July 31 counsel for both importers wrote a letter
to the Collector requesting that they be allowed to file sufficient bonds for the
release of the goods, without prejudice to their right to contest the validity of
seizure. On the same date the Collector granted the request by means of a
handwritten marginal notation on the letter itself, provided "duty and taxes
have already been paid." This condition had been previously met, and so the
corresponding surety bonds were filed, in the aggregate amount of
P513,865.46. Their approval was requested in another letter dated August 10,
1970, but the Collector of Customs thereupon required a cash bond instead, as
indicated in a similar marginal notation on this second letter.

On the same date — August 10 — the two importers filed a petition with the
Court of Tax Appeals to stop the sale at public auction of the fruit shipments in
question, with a prayer for preliminary injunction until the final determination of
the validity of the seizure proceedings. The said Court, however, by resolution
dated August 12, 1970, dismissed the petition on the ground of lack of
jurisdiction, stating that neither the Collector of Customs nor the Commissioner
of Customs had yet rendered any decision from which an appeal could be
taken pursuant to Section 7 of Republic Act, No. 1125. Evidently anticipating
such a ruling and considering the urgency of the matter, the importers went to
the Court of First Instance on a petition for injunction, wherein the resolution
reproduced in the beginning of this decision was thereafter promulgated after
hearing.

That there must be some forum to which a party may apply for relief from an
alleged violation or denial of his rights is a legal principle from which there can
be no dissent. Otherwise the rule of law would be defeated. The choice in this
case was between the Court of Tax Appeals and the Court of First Instance.
Recourse to the former was sought and denied. The Tax Court held that it could
not issue the preliminary injunction prayed for except in the exercise of its
6

appellate jurisdiction, and no appeal had been taken since no appealable


decision had been rendered. The ruling appears to find support in the decisions
of this Court, thus:

... Nowhere does the law expressly vest in the Court of Tax Appeals
original jurisdiction to issue writs of prohibition or injunction
independently of, and apart from, an appealed case. The writ of
prohibition or injunction that it may issue under the provisions of
section 11, Republic Act No. 1125, to suspend the collection of taxes,
is merely ancillary to and in furtherance of its appellate jurisdiction in
the cases mentioned in section 7 of the Act. The power to issue the
writ exists only in cases appealed to it. This is reflected in the
explanatory note of the bill (House No. 175), creating the Court of
Tax Appeal. (Coll. of Int. Rev. v. Yuseco, G.R. No. L-12518, Oct. 28,
1961.)

Respondent Court of First Instance assumed jurisdiction over the petition before
it on the ground that "the question presented for resolution (was) whether there
was absence of due process," citing our decision in Nadeco vs. Collector of
Customs, G.R. No.
L-19180, Oct. 31, 1969. The said Court found: "Counsel for the respondents
admitted that the petitioners have not been heard on the seizure proceedings
and the imported cargo have already been advertised for sale and some
would have been sold had not this Court issued a restraining order." Due notice
and hearing, besides being an inherent element of due process, is provided for
in Section 2303 of the Tariff and Customs Code, which requires the Collector to
give the owner or importer of the property written notice of the seizure and an
opportunity to be heard in relation to the delinquency which was the occasion
for such seizure, as well as in Section 2601, which directs that seized property,
other than contraband, shall be subject to sale after liability to sale shall have
been established by proper administrative or judicial proceedings in conformity
with the provisions of said Code.

In view of the foregoing, we hold that respondent Court of First Instance had
jurisdiction to take cognizance of the petition for injunction before it. The
remedy prayed for was one in equity, which the petitioner below tried to seek
in the Court of Tax Appeals, but was denied on the ground that no appealable
decision had yet been rendered by the Collector and the Commissioner of
Customs. The jurisdiction of respondent Court was not invoked to determine the
validity of the seizure proceedings, which are pending before the Collector of
Customs and regarding which an appeal could be eventually taken only to the
Tax Court, but rather to stop the projected auction sale of the goods in
question and secure the release thereof under surety bond, without prejudice
to the main issue concerning the validity of the seizure. Such relief is
interlocutory in nature, and is sanctioned by Section 2301 of the Tariff and
Customs Code, which provides that "upon making any seizure the Collector
shall issue a warrant for the detention of the property; but if the owner or
importer desires to secure the release of the property for legitimate use, the
Collector may surrender it upon the filing of a sufficient bond, in an amount to
7

be fixed by him, conditioned for payment of the appraised value of the article
and/or any fine, expenses and costs which may be adjudged in the case."

The really basic issue before us is whether or not respondent Court gravely
abused its discretion in issuing the orders complained of, particularly that dated
August 26, 1970. For the resolution of this issue we need not pass squarely upon
the question of whether the importations in question are prohibited by law
within the meaning of the proviso in Section 2301 of the Tariff and Customs
Code which says that such prohibited importation may not be released under
bond. That question is involved and should properly be decided in the seizure
proceedings. For purposes of the equitable remedy of injunction granted by
respondent Court, however, as well, as of the petition for certiorari and
prohibition before us, it is sufficient to note, first, that there is no clear showing
that the importations subject of seizure are prohibited by law; and second, that
the Collector of Customs has in fact agreed in the beginning to release the
importations provided surety bonds were filed, although he subsequently
required a cash bond instead.

The warrants of seizure were issued in view of Central Bank Circulars Nos. 294
and 295, promulgated on March 10 and 20, 1970, respectively, which provide
that "no-dollar imports not covered by Circular No. 247 shall not be issued any
release certificates and shall be referred to the Central Bank for official
transmittal to the Bureau of Customs for appropriate seizure proceedings."

Evidently, in the opinion of the Collector of Customs himself, even in the light of
those circulars there exists no legal impediment to the release of the subject
importations under bond, otherwise he would not have agreed thereto,
although he changed his requirement from surety bond to cash. In any case, as
pointed out by private respondents, the said importations had been ordered
before Central Bank Circulars 294 and 295 were promulgated, and since the
orders were made in accordance with previous practice there could be no
bad faith or intent to violate those circulars.

The options presented in this case are few and clearcut: (1) to sell the imported
fresh fruits at public auction, as the petitioners due insist; (2) to release them to
the private respondents upon the filing of sufficient surety bonds, as respondent
Court has directed; and (3) to require the private respondents to file a cash
bond instead.

We fail to see what good it would do either the Government or the private
respondents to have the fruits sold at public auction. The Government's interest,
ultimately, is in the proceeds which may be realized from such sale, in the event
the fruits are declared forfeited in the seizure proceedings. By now a
considerable portion thereof must have deteriorated, and the rest will in all
probability not command the same prices as before. Besides, as pointed out by
the respondents — and this has not been denied — the Commissioner of
Customs has been quoted by a newspaper on September 29, 1970, to the
effect that "seized items worth hundreds of thousands of pesos could not be
disposed of because of the unrealistic bids received by the Bureau of Customs
when the goods were offered for sale at public auction. ... Some of the offers
were not even enough to pay the import taxes and customs duties due on the
8

articles." To sell the goods at public auction, therefore, cannot but entail great
loss either to the Government or to the importers.

On the other hand the filing of sufficient bond would serve the purpose
envisaged, that is, protect the interest of the Government in the value of the
imported goods should they be finally declared forfeited, while at the same
time avoiding needless damage or prejudice to the importers should the
forfeiture fail. The release on bond, it may be repeated, is expressly authorized
by Section 2301 of the Tariff and Customs Code.

But the petitioners would have the private respondents put up cash, alleging
that it may be difficult to realize upon a surety bond if it is allowed. We do not
believe this reason is justified. In the first place, a bond, when required by law, is
commonly understood to mean an undertaking that is sufficiently secured, and
not cash or currency. According to the respondents this is the established
practice in the Bureau of Customs, and this statement has not been denied. Of
course whatever surety bonds are submitted by the importers are subject to
any objections by the Collector of Customs as to their sufficiency or as to the
solvency of the bondsman. In the second place, to require the private
respondents here to put up cash in the sum of P513,865.46 is prohibitive and
unrealistic, and amounts to an arbitrary exercise of discretion under the
circumstances of this case, assuming that the matter is discretionary.

We note, however, that the bonds offered by the respondents are all
subscribed by the same bonding company, namely, the Communications
Insurance Co., Inc., which has a net worth of only P504,655.15 and a maximum
writing capacity of P50,465.52, on the basis of its financial statement as of
December 31, 1969, according to a letter of the Acting Insurance
Commissioner dated August 28, 1970. The figure given by the petitioners in their
objection to the sufficiency of the bonds before respondent court is P596,342.51
in reference to the net worth of said company. In any case the petitioners have
expressed doubts as to whether the bondsman can satisfy a liability of
P513,865.46, which is the aggregate amount of the bonds submitted. The
objection on this ground has been brushed aside by the lower court in its order
of September 8, 1970, since the private respondents "have shown that the
bonding company obtained reinsurance on part of their liability for those
bonds." But it appears, as manifested by said respondents themselves, that only
two of the bonds submitted by them, in the respective amounts of P94,647.80
and P78,981.24, are covered by reinsurance, leaving more than P340,000.00 not
reinsured. In view thereof, it is incumbent upon the respondents to either cause
of sufficient portion of the other bonds submitted by it to be covered by
reinsurance or to put up other surety bonds acceptable to the Collector of
Customs, the same to be justified before respondent Court in case of dispute.

WHEREFORE, subject to the condition stated in the preceding paragraph, the


writ prayed for is denied, the petition dismissed, and the restraining order issued
by this Court hereby lifted. No pronouncement as to costs.

G.R. No. 135210. July 11, 2001


9

COMMISSIONER OF INTERNAL REVENUE, Petitioner, vs. ISABELA CULTURAL


CORPORATION, respondent.

DECISION

PANGANIBAN, J.:

A final demand letter from the Bureau of Internal Revenue, reiterating to the
taxpayer the immediate payment of a tax deficiency assessment previously
made, is tantamount to a denial of the taxpayers request for reconsideration.
Such letter amounts to a final decision on a disputed assessment and is thus
appealable to the Court of Tax Appeals (CTA).

The Case

Before this Court is a Petition for Review on Certiorari 1 pursuant to Rule 45 of the
Rules of Court, seeking to set aside the August 19, 1998 Decision 2 of the Court
of Appeals 3 (CA) in CA-GR SP No. 46383 and ultimately to affirm the dismissal of
CTA Case No. 5211. The dispositive portion of the assailed Decision reads as
follows:

WHEREFORE, the assailed decision is REVERSED and SET ASIDE. Accordingly,


judgment is hereby rendered REMANDING the case to the CTA for proper
disposition.4

The Facts

The facts are undisputed. The Court of Appeals quoted the summary of the
CTA as follows:

As succinctly summarized by the Court of Tax appeals (CTA for brevity), the
antecedent facts are as follows:

In an investigation conducted on the 1986 books of account of [respondent,


petitioner] had the preliminary [finding] that [respondent] incurred a total
income tax deficiency of P9,985,392.15, inclusive of increments. Upon protest
by [respondents] counsel, the said preliminary assessment was reduced to the
amount of P325,869.44, a breakdown of which follows:

Deficiency Income Tax P321,022.68


Deficiency Expanded
Withholding Tax 4,846.76
___________
Total P325,869.44
==========

(pp. 187-189, BIR records)

On February 23, 1990, [respondent] received from [petitioner] an assessment


letter, dated February 9, 1990, demanding payment of the amounts of
P333,196.86 and P4,897.79 as deficiency income tax and expanded
10

withholding tax inclusive of surcharge and interest, respectively, for the taxable
period from January 1, 1986 to December 31, 1986. (pp. 204 and 205, BIR rec.)

In a letter, dated March 22, 1990, filed with the [petitioners] office on March 23,
1990 (pp. 296-311, BIR rec.), [respondent] requested x x x a reconsideration of
the subject assessment.

Supplemental to its protest was a letter, dated April 2, 1990, filed with the
[petitioners] office on April 18, 1990 (pp. 224 & 225, BIR rec.), to which x x x were
attached certain documents supportive of its protest, as well as a Waiver of
Statute of Limitation, dated April 17, 1990, where it was indicated that
[petitioner] would only have until April 5, 1991 within which to asses and collect
the taxes that may be found due from [respondent] after the re-investigation.

On February 9, 1995, [respondent] received from [petitioner] a Final Notice


Before Seizure, dated December 22, 1994 (p. 340, BIR rec.). In said letter,
[petitioner] demanded payment of the subject assessment within ten (10) days
from receipt thereof. Otherwise, failure on its part would constrain [petitioner] to
collect the subject assessment through summary remedies.

[Respondent] considered said final notice of seizure as [petitioners] final


decision. Hence, the instant petition for review filed with this Court on March 9,
1995.

The CTA having rendered judgment dismissing the petition, [respondent] filed
the instant petition anchored on the argument that [petitioners] issuance of the
Final Notice Before Seizure constitutes [its] decision on [respondents] request for
reinvestigation, which the [respondent] may appeal to the CTA.5

Ruling of the Court of Appeals

In its Decision, the Court of Appeals reversed the Court of Tax Appeals. The CA
considered the final notice sent by petitioner as the latters decision, which was
appealable to the CTA. The appellate court reasoned that the final Notice
before seizure had effectively denied petitioners request for a reconsideration
of the commissioners assessment. The CA relied on the long-settled tax
jurisprudence that a demand letter reiterating payment of delinquent taxes
amounted to a decision on a disputed assessment.

Hence, this recourse. 6

Issues

In his Memorandum, 7 petitioner presents for this Courts consideration a solitary


issue:

Whether or not the Final Notice Before Seizure dated February 9, 1995 signed by
Acting Chief Revenue Collection Officer Milagros Acevedo against ICC
constitutes the final decision of the CIR appealable to the CTA.8

The Courts Ruling


11

The Petition is not meritorious.

Sole Issue: The Nature of the Final Notice Before Seizure

The Final Notice Before Seizure sent by the Bureau of Internal Revenue (BIR) to
respondent reads as follows:

On Feb.9, 1990, [this] Office sent you a letter requesting you to settle the above-
captioned assessment. To date, however, despite the lapse of a considerable
length of time, we have not been honored with a reply from you.

In this connection, we are giving you this LAST OPPORTUNITY to settle the
adverted assessment within ten (10) days after receipt hereof. Should you
again fail, and refuse to pay, this Office will be constrained to enforce its
collection by summary remedies of Warrant of Levy of Road Property, Distraint
of Personal Property or Warrant of Garnishment, and/or simultaneous court
action.

Please give this matter your preferential attention.

Very truly yours,

ISIDRO B. TECSON, JR.


Revenue District Officer

By:

(Signed)
MILAGROS M. ACEVEDO
Actg. Chief Revenue Collection
Officer9cräläwvirtualibräry

Petitioner maintains that this Final Notice was a mere reiteration of the
delinquent taxpayers obligation to pay the taxes due. It was supposedly a
mere demand that should not have been mistaken for a decision on a
protested assessment. Such decision, the commissioner contends, must
unequivocably indicate that it is the resolution of the taxpayers request for
reconsideration and must likewise state the reason therefor.

Respondent, on the other hand, points out that the Final Notice Before Seizure
should be considered as a denial of its request for reconsideration of the
disputed assessment. The Notice should be deemed as petitioners last act,
since failure to comply with it would lead to the distraint and levy of
respondents properties, as indicated therein.

We agree with respondent. In the normal course, the revenue district officer
sends the taxpayer a notice of delinquent taxes, indicating the period covered,
the amount due including interest, and the reason for the delinquency. If the
taxpayer disagrees with or wishes to protest the assessment, it sends a letter to
the BIR indicating its protest, stating the reasons therefor, and submitting such
proof as may be necessary. That letter is considered as the taxpayers request
for reconsideration of the delinquent assessment. After the request is filed and
12

received by the BIR, the assessment becomes a disputed assessment on which


it must render a decision. That decision is appealable to the Court of Tax
Appeals for review.

Prior to the decision on a disputed assessment, there may still be exchanges


between the commissioner of internal revenue (CIR) and the taxpayer. The
former may ask clarificatory questions or require the latter to submit additional
evidence. However, the CIRs position regarding the disputed assessment must
be indicated in the final decision. It is this decision that is properly appealable
to the CTA for review.

Indisputably, respondent received an assessment letter dated February 9, 1990,


stating that it had delinquent taxes due; and it subsequently filed its motion for
reconsideration on March 23, 1990. In support of its request for reconsideration,
it sent to the CIR additional documents on April 18, 1990. The next
communication respondent received was already the Final Notice Before
Seizure dated November 10, 1994.

In the light of the above facts, the Final Notice Before Seizure cannot but be
considered as the commissioners decision disposing of the request for
reconsideration filed by respondent, who received no other response to its
request. Not only was the Notice the only response received; its content and
tenor supported the theory that it was the CIRs final act regarding the request
for reconsideration. The very title expressly indicated that it was a final notice
prior to seizure of property. The letter itself clearly stated that respondent was
being given this LAST OPPORTUNITY to pay; otherwise, its properties would be
subjected to distraint and levy. How then could it have been made to believe
that its request for reconsideration was still pending determination, despite the
actual threat of seizure of its properties?

Furthermore, Section 228 of the National Internal Revenue Code states that a
delinquent taxpayer may nevertheless directly appeal a disputed assessment, if
its request for reconsideration remains unacted upon 180 days after submission
thereof. We quote:

Sec. 228. Protesting an Assessment. x x x

Within a period to be prescribed by implementing rules and regulations, the


taxpayer shall be required to respond to said notice. If the taxpayer fails to
respond, the Commissioner or his duly authorized representative shall issue an
assessment based on his findings.

Such assessment may be protested administratively by filing a request for


reconsideration or reinvestigation within thirty (30) days from receipt of the
assessment in such form and manner as may be prescribed by implementing
rules and regulations. Within sixty (60) days from filing of the protest, all relevant
supporting documents shall have become final.

If the protest is denied in whole or in part, or is not acted upon within one
hundred eighty (180) days from submission of documents, the taxpayer
adversely affected by the decision or inaction may appeal to the Court of Tax
13

Appeals within (30) days from receipt of the said decision, or from the lapse of
the one hundred eighty (180)-day period; otherwise the decision shall become
final, executory and demandable.10cräläwvirtualibräry

In this case, the said period of 180 days had already lapsed when respondent
filed its request for reconsideration on March 23, 1990, without any action on
the part of the CIR.

Lastly, jurisprudence dictates that a final demand letter for payment of


delinquent taxes may be considered a decision on a disputed or protested
assessment. In Commissioner of Internal Revenue v. Ayala Securities
Corporation, this Court held:

The letter of February 18, 1963 (Exh. G), in the view of the Court, is tantamount
to a denial of the reconsideration or [respondent corporations] x x x protest o[f]
the assessment made by the petitioner, considering that the said letter [was] in
itself a reiteration of the demand by the Bureau of Internal Revenue for the
settlement of the assessment already made, and for the immediate payment
of the sum of P758,687.04 in spite of the vehement protest of the respondent
corporation on April 21, 1961. This certainly is a clear indication of the firm stand
of petitioner against the reconsideration of the disputed assessment, in view of
the continued refusal of the respondent corporation to execute the waiver of
the period of limitation upon the assessment in question.

This being so, the said letter amount[ed] to a decision on a disputed or


protested assessment and, there, the court a quo did not err in taking
cognizance of this case.11cräläwvirtualibräry

Similarly, in Surigao Electric Co., Inc. v. Court of Tax Appeals 12 and again in CIR
v. Union Shipping Corp., 13 we ruled:

x x x. The letter of demand dated April 29, 1963 unquestionably constitutes the
final action taken by the commissioner on the petitioners several requests for
reconsideration and recomputation. In this letter the commissioner not only in
effect demanded that the petitioner pay the amount of P11,533.53 but also
gave warning that in the event it failed to pay, the said commissioner would be
constrained to enforce the collection thereof by means of the remedies
provided by law. The tenor of the letter, specifically the statement regarding
the resort to legal remedies, unmistakably indicate[d] the final nature of the
determination made by the commissioner of the petitioners deficiency
franchise tax liability.

As in CIR v. Union Shipping, 14 petitioner failed to rule on the Motion for


Reconsideration filed by private respondent, but simply continued to demand
payment of the latters alleged tax delinquency. Thus, the Court reiterated the
dictum that the BIR should always indicate to the taxpayer in clear and
unequivocal language what constitutes final action on a disputed assessment.
The object of this policy is to avoid repeated requests for reconsideration by the
taxpayer, thereby delaying the finality of the assessment and, consequently,
the collection of the taxes due. Furthermore, the taxpayer would not be
14

groping in the dark, speculating as to which communication or action of the


BIR may be the decision appealable to the tax court. 15cräläwvirtualibräry

In the instant case, the second notice received by private respondent verily
indicated its nature that it was final. Unequivocably, therefore, it was
tantamount to a rejection of the request for reconsideration.

Commissioner v. Algue 16 is not in point here. In that case, the Warrant of


Distraint and Levy, issued to the taxpayer without any categorical ruling on its
request for reconsideration, was not deemed equivalent to a denial of the
request. Because such request could not in fact be found in its records, the BIR
cannot be presumed to have taken it into consideration. The request was
considered only when the taxpayer gave a copy of it, duly stamp-received by
the BIR. Hence, the Warrant was deemed premature.

In the present case, petitioner does not deny receipt of private respondents
protest letter. As a matter of fact, it categorically relates the following in its
Statement of Relevant Facts: 17cräläwvirtualibräry

3. On March 23, 1990, respondent ICC wrote the CIR requesting for a
reconsideration of the assessment on the ground that there was an error
committed in the computation of interest and that there were expenses which
were disallowed (Ibid., pp. 296-311).

4. On April 2, 1990, respondent ICC sent the CIR additional documents in


support of its protest/reconsideration. The letter was received by the BIR on April
18, 1990. Respondent ICC further executed a Waiver of Statute of Limitation
(dated April 17, 1990) whereby it consented to the BIR to assess and collect any
taxes that may be discovered in the process of reinvestigation, until April 3, 1991
(Ibid., pp. 296-311). A copy of the waiver is hereto attached as Annex C.

Having admitted as a fact private respondents request for reconsideration,


petitioner must have passed upon it prior to the issuance of the Final Notice
Before Seizure.

WHEREFORE , the Petition is hereby DENIED and the assailed Decision AFFIRMED.

SO ORDERED.

G.R. No. 127777. October 1, 1999]

PETRONILA C. TUPAZ, Petitioner, v. HONORABLE BENEDICTO B. ULEP Presiding


Judge of RTC Quezon City, Branch 105, and PEOPLE OF THE
PHILIPPINES, Respondents.

DECISION

PARDO, J.:

The case before us is a special civil action for certiorari with application for
temporary restraining order seeking to enjoin respondent Judge Benedicto B.
Ulep of the Regional Trial Court, Quezon City, Branch 105, from trying Criminal
15

Case No. Q-91-17321, and to nullify respondent judges order reviving the
information therein against petitioner, for violation of the Tax Code, as the
offense charged has prescribed or would expose petitioner to double
jeopardy.

The facts are as follows:

On June 8, 1990, State Prosecutor (SP) Esteban A. Molon, Jr. filed with the
Metropolitan Trial Court (MeTC), Quezon City, Branch 33, an information against
accused Petronila C. Tupaz and her late husband Jose J. Tupaz, Jr., as
corporate officers of El Oro Engravers Corporation, for nonpayment of
deficiency corporate income tax for the year 1979, amounting to P2,369,085.46,
in violation of Section 51 (b) in relation to Section 73 of the Tax Code of
1977.1 On September 11, 1990, the MeTC dismissed the information for lack of
jurisdiction. On November 16, 1990, the trial court denied the prosecutions
motion for reconsideration.

On January 10, 1991, SP Molon filed with the Regional Trial Court, Quezon City,
two (2) informations, docketed as Criminal Case Nos. Q-91-173212 and Q-91-
17322,3 against accused and her late husband, for the same alleged
nonpayment of deficiency corporate income tax for the year 1979. Criminal
Case No. Q-91-17321 was raffled to Branch 105,4 presided over by respondent
Judge Benedicto B. Ulep; Q-91-17322 was raffled to Branch 86, then presided
over by Judge Antonio P. Solano. The identical informations read as follows:

That in Quezon City, Metro Manila and within the jurisdiction of this Honorable
Court and upon verification and audit conducted by the Bureau of Internal
Revenue on the 1979 corporate annual income tax return and financial
statements of El Oro Engravers Corp., with office address at 809 Epifanio delos
Santos Avenue, Quezon City, Metro Manila, it was ascertained that said
corporation was found liable to pay the amount of P2,369,085.46, as deficiency
corporate income tax for the year 1979 and that, despite demand of the
payment of the aforesaid deficiency tax by the Bureau of Internal Revenue
and received by said corporation, which demand has already become final,
said El Oro Engravers Corp., through above-named accused, the responsible
corporate-officers of said corporation, failed and refused, despite repeated
demands, and still fail and refuse to pay said tax liability.

CONTRARY TO LAW.5

On September 25, 1991, both accused posted bail bond in the sum of P1,000.00
each, for their provisional liberty.

On November 6, 1991, accused filed with the Regional Trial Court, Quezon City,
Branch 86, a motion to dismiss/quash6 information (Q-91-17322) for the reason
that it was exactly the same as the information against the accused pending
before RTC, Quezon City, Branch 105 (Q-91-17321). However, on November 11,
1991, Judge Solano denied the motion.7cräläwvirtualibräry

In the meantime, on July 25, 1993, Jose J. Tupaz, Jr. died in Quezon City.
16

Subsequently, accused Petronila C. Tupaz filed with the Regional Trial Court,
Quezon City, Branch 105, a petition for reinvestigation, which Judge Ulep
granted in an order dated August 30, 1994.8cräläwvirtualibräry

On September 5, 1994, Senior State Prosecutor Bernelito R. Fernandez stated


that no new issues were raised in the request for reinvestigation, and no cogent
reasons existed to alter, modify or reverse the findings of the investigating
prosecutor. He considered the reinvestigation as terminated, and
recommended the prompt arraignment and trial of the
accused.9cräläwvirtualibräry

On September 20, 1994, the trial court (Branch No. 105) arraigned accused
Petronila C. Tupaz in Criminal Case No. Q-91-17321, and she pleaded not guilty
to the information therein.

On October 17, 1994, the prosecution filed with the Regional Trial Court,
Quezon City, Branch 105, a motion for leave to file amended information in
Criminal Case No. Q91-17321 to allege expressly the date of the commission of
the offense, to wit: on or about August 1984 or subsequently thereafter. Despite
opposition of the accused, on March 2, 1995, the trial court granted the motion
and admitted the amended information.10 Petitioner was not re-arraigned on
the amended information. However, the amendment was only on a matter of
form.11 Hence, there was no need to re-arraign the
accused.12cräläwvirtualibräry

On December 5, 1995, accused filed with the Regional Trial Court, Quezon City,
Branch 105, a motion for leave to file and admit motion for reinvestigation. The
trial court granted the motion in its order dated December 13, 1995.

Prior to this, on October 18, 1995, Judge Ulep issued an order directing the
prosecution to withdraw the information in Criminal Case No. Q-91-17322,
pending before Regional Trial Court, Quezon City, Branch 86, after discovering
that said information was identical to the one filed with Regional Trial Court,
Quezon City, Branch 105. On April 16, 1996, State Prosecutor Alfredo P. Agcaoili
filed with the trial court a motion to withdraw information in Criminal Case No.
Q-91-17321. Prosecutor Agcaoili thought that accused was charged in Criminal
Case No. Q-91-17321, for nonpayment of deficiency contractors tax, but found
that accused was exempted from paying said tax.

On May 15, 1996, Prosecutor Agcaoili filed with the Regional Trial Court, Quezon
City, Branch 86, a motion for consolidation of Criminal Case No. Q-91-17322
with Criminal Case No. Q-91-17321 pending before the Regional Trial Court,
Quezon City, Branch 105. On the same date, the court13 granted the motion for
consolidation.

On May 20, 1996, Judge Ulep of Regional Trial Court, Quezon City, Branch 105,
granted the motion for withdrawal of the information in Criminal Case No. Q-91-
17321 and dismissed the case, as prayed for by the prosecution.

On May 28, 1996, Prosecutor Agcaoili filed with the Regional Trial Court, Quezon
City, Branch 105, a motion to reinstate information in Criminal Case Q-91-
17

17321,14 stating that the motion to withdraw information was made through
palpable mistake, and was the result of excusable neglect. He thought that
Criminal Case No. Q-91-17321 was identical to Criminal Case No Q-90-12896,
wherein accused was charged with nonpayment of deficiency contractors tax,
amounting to P346,879.29.

Over the objections of accused, on August 6, 1996, the Regional Trial Court,
Quezon City, Branch 105, granted the motion and ordered the information in
Criminal Case No. Q-91-17321 reinstated.15 On September 24, 1996, accused
filed with the trial court a motion for reconsideration. On December 4, 1996, the
trial court denied the motion.

Hence, this petition.

On July 9, 1997, we required respondents to comment on the petition within ten


(10) days from notice. On October 10, 1997, the Solicitor General filed his
comment.16cräläwvirtualibräry

On October 26, 1998, the Court resolved to give due course to the petition and
required the parties to file their respective memoranda within twenty (20) days
from notice. The parties have complied.

Petitioner submits that respondent judge committed a grave abuse of


discretion in reinstating the information in Criminal Case No. Q-91-17321
because (a) the offense has prescribed; or (b) it exposes her to double
jeopardy.

As regards the issue of prescription, petitioner contends that: (a) the period of
assessment has prescribed, applying the three (3) year period provided under
Batas Pambansa No. 700; (b) the offense has prescribed since the complaint
for preliminary investigation was filed with the Department of Justice only on
June 8, 1989, and the offense was committed in April 1980 when she filed the
income tax return covering taxable year 1979.

Petitioner was charged with nonpayment of deficiency corporate income tax


for the year 1979, which tax return was filed in April 1980. On July 16, 1984, the
Bureau of Internal Revenue (BIR) issued a notice of assessment. Petitioner
contends that the July 16, 1984 assessment was made out of time.

Petitioner avers that while Sections 318 and 319 of the NIRC of 1977 provide a
five (5) year period of limitation for the assessment and collection of internal
revenue taxes, Batas Pambansa Blg. 700, enacted on February 22, 1984,
amended the two sections and reduced the period to three (3) years. As
provided under B.P. Blg. 700, the BIR has three (3) years to assess the tax liability,
counted from the last day of filing the return, or from the date the return is filed,
whichever comes later. Since the tax return was filed in April 1980, the
assessment made on July 16, 1984 was beyond the three (3) year prescriptive
period.

Petitioner submits that B.P. Blg. 700 must be given retroactive effect since it is
favorable to the accused. Petitioner argues that Article 22 of the Revised Penal
18

Code, regarding the allowance of retroactive application of penal laws when


favorable to the accused shall apply in this case.

The Solicitor General, in his comment, maintains that the prescriptive period for
assessment and collection of petitioners deficiency corporate income tax was
five (5) years. The Solicitor General asserts that the shortened period of three (3)
years provided under B.P. Blg. 700 applies to assessments and collections of
internal revenue taxes beginning taxable year 1984. Since the deficiency
corporate income tax was for taxable year 1979, then petitioner was still
covered by the five (5) year period. Thus, the July 16, 1984 tax assessment was
made within the prescribed period.

At the outset, it must be stressed that internal revenue taxes are self-assessing
and no further assessment by the government is required to create the tax
liability. An assessment, however, is not altogether inconsequential; it is relevant
in the proper pursuit of judicial and extra judicial remedies to enforce taxpayer
liabilities and certain matters that relate to it, such as the imposition of
surcharges and interest, and in the application of statues of limitations and in
the establishment of tax liens.17cräläwvirtualibräry

An assessment contains not only a computation of tax liabilities, but also a


demand for payment within a prescribed period. The ultimate purpose of
assessment is to ascertain the amount that each taxpayer is to pay.18 An
assessment is a notice to the effect that the amount therein stated is due as tax
and a demand for payment thereof.19 Assessments made beyond the
prescribed period would not be binding on the taxpayer.20cräläwvirtualibräry

We agree with the Solicitor General that the shortened period of three (3) years
prescribed under B.P. Blg. 700 is not applicable to petitioner. B.P. Blg. 700,
effective April 5, 1984, specifically states that the shortened period of three
years shall apply to assessments and collections of internal revenue taxes
beginning taxable year 1984. Assessments made on or after April 5, 1984 are
governed by the five-year period if the taxes assessed cover taxable years prior
to January 1, 1984.21 The deficiency income tax under consideration is for
taxable year 1979. Thus, the period of assessment is still five (5) years, under the
old law. The income tax return was filed in April 1980. Hence, the July 16, 1984
tax assessment was issued within the prescribed period of five (5) years, from
the last day of filing the return, or from the date the return is filed, whichever
comes later.

Article 22 of the Revised Penal Code finds no application in this case for the
simple reason that the provisions on the period of assessment can not be
considered as penal in nature.

Petitioner also asserts that the offense has prescribed. Petitioner invokes Section
340 (now 281 of 1997 NIRC) of the Tax Code which provides that violations of
any provision of the Code prescribe in five (5) years. Petitioner asserts that in this
case, it began to run in 1979, when she failed to pay the correct corporate tax
due during that taxable year. Hence, when the BIR instituted criminal
proceedings on June 8, 1989, by filing a complaint for violation of the Tax Code
with the Department of Justice for preliminary investigation it was beyond the
19

prescriptive period of five (5) years. At most, the BIR had until 1984 to institute
criminal proceedings.

On the other hand, the Solicitor General avers that the information for violation
of the Tax Code was filed within the prescriptive period of five (5) years
provided in Section 340 (now 281 in 1997 NIRC) of the Code. It is only when the
assessment has become final and unappealable that the five (5) year period
commences to run. A notice of assessment was issued on July 16, 1984. When
petitioner failed to question or protest the deficiency assessment thirty (30) days
therefrom, or on August 16, 1984, it became final and unappealable.
Consequently, it was from this period that the prescriptive period of five (5)
years commenced. Thus, the complaint filed with the Department of Justice on
June 8, 1989 was within the prescribed period.

We agree with the Solicitor General that the offense has not prescribed.
Petitioner was charged with failure to pay deficiency income tax after
repeated demands by the taxing authority. In Lim, Sr. v. Court of Appeals,22 we
stated that by its nature the violation could only be committed after service of
notice and demand for payment of the deficiency taxes upon the taxpayer.
Hence, it cannot be said that the offense has been committed as early as
1980, upon filing of the income tax return. This is so because prior to the finality
of the assessment, the taxpayer has not committed any violation for
nonpayment of the tax. The offense was committed only after the finality of the
assessment coupled with taxpayers willful refusal to pay the taxes within the
allotted period. In this case, when the notice of assessment was issued on July
16, 1984, the taxpayer still had thirty (30) days from receipt thereof to protest or
question the assessment. Otherwise, the assessment would become final and
unappealable.23 As he did not protest, the assessment became final and
unappealable on August 16, 1984. Consequently, when the complaint for
preliminary investigation was filed with the Department of Justice on June 8,
1989, the criminal action was instituted within the five (5) year prescriptive
period.

Petitioner contends that by reinstating the information, the trial court exposed
her to double jeopardy. Neither the prosecution nor the trial court obtained her
permission before the case was dismissed. She was placed in jeopardy for the
first time after she pleaded to a valid complaint filed before a competent court
and the case was dismissed without her express consent. When the trial court
reinstated the information charging the same offense, it placed her in double
jeopardy.

Petitioner also asserts that the trial court gravely erred when, over her
objections, it admitted the amended information. She submits that the
amendment is substantial in nature, and would place her in double jeopardy.

On the other hand, the Solicitor General contends that reinstating the
information does not violate petitioners right against double jeopardy. He
asserts that petitioner induced the dismissal of the complaint when she sought
the reinvestigation of her tax liabilities. By such inducement, petitioner waived
or was estopped from claiming her right against double jeopardy.
20

The Solicitor General further contends that, assuming arguendo that the case
was dismissed without petitioners consent, there was no valid dismissal of the
case since Prosecutor Agcaoili was under a mistaken assumption that it was a
charge of nonpayment of contractors tax.

We sustain petitioners contention. The reinstatement of the information would


expose her to double jeopardy. An accused is placed in double jeopardy if he
is again tried for an offense for which he has been convicted, acquitted or in
another manner in which the indictment against him was dismissed without his
consent. In the instant case, there was a valid complaint filed against petitioner
to which she pleaded not guilty. The court dismissed the case at the instance of
the prosecution, without asking for accused-petitioners consent. This consent
cannot be implied or presumed.24 Such consent must be expressed as to have
no doubt as to the accuseds conformity.25 As petitioners consent was not
expressly given, the dismissal of the case must be regarded as final and with
prejudice to the re-filing of the case.26 Consequently, the trial court committed
grave abuse of discretion in reinstating the information against petitioner in
violation of her constitutionally protected right against double jeopardy.

WHEREFORE, we GRANT the petition. We enjoin the lower court, the Regional
Trial Court of Quezon City, Branch 105, from trying Criminal Case No. Q-91-
17321 and order its dismissal. Costs de oficio.

SO ORDERED.

G.R. No. 185371 December 8, 2010

COMMISSIONER OF INTERNAL REVENUE, Petitioner,


vs.
METRO STAR SUPERAMA, INC., Respondent.

DECISION

MENDOZA, J.:

This petition for review on certiorari under Rule 45 of the Rules of Court filed by
the petitioner Commissioner of Internal Revenue (CIR) seeks to reverse and set
aside the 1] September 16, 2008 Decision1 of the Court of Tax Appeals En Banc
(CTA-En Banc), in C.T.A. EB No. 306 and 2] its November 18, 2008
Resolution2 denying petitioner’s motion for reconsideration.

The CTA-En Banc affirmed in toto the decision of its Second Division (CTA-
Second Division) in CTA Case No. 7169 reversing the February 8, 2005 Decision
of the CIR which assessed respondent Metro Star Superama, Inc. (Metro Star) of
deficiency value-added tax and withholding tax for the taxable year 1999.

Based on a Joint Stipulation of Facts and Issues3 of the parties, the CTA Second
Division summarized the factual and procedural antecedents of the case, the
pertinent portions of which read:

Petitioner is a domestic corporation duly organized and existing by virtue of the


laws of the Republic of the Philippines, x x x.
21

On January 26, 2001, the Regional Director of Revenue Region No. 10, Legazpi
City, issued Letter of Authority No. 00006561 for Revenue Officer Daisy G.
Justiniana to examine petitioner’s books of accounts and other accounting
records for income tax and other internal revenue taxes for the taxable year
1999. Said Letter of Authority was revalidated on August 10, 2001 by Regional
Director Leonardo Sacamos.

For petitioner’s failure to comply with several requests for the presentation of
records and Subpoena Duces Tecum, [the] OIC of BIR Legal Division issued an
Indorsement dated September 26, 2001 informing Revenue District Officer of
Revenue Region No. 67, Legazpi City to proceed with the investigation based
on the best evidence obtainable preparatory to the issuance of assessment
notice.

On November 8, 2001, Revenue District Officer Socorro O. Ramos-Lafuente


issued a Preliminary 15-day Letter, which petitioner received on November 9,
2001. The said letter stated that a post audit review was held and it was
ascertained that there was deficiency value-added and withholding taxes due
from petitioner in the amount of ₱ 292,874.16.

On April 11, 2002, petitioner received a Formal Letter of Demand dated April 3,
2002 from Revenue District No. 67, Legazpi City, assessing petitioner the amount
of Two Hundred Ninety Two Thousand Eight Hundred Seventy Four Pesos and
Sixteen Centavos (₱292,874.16.) for deficiency value-added and withholding
taxes for the taxable year 1999, computed as follows:

ASSESSMENT NOTICE NO. 067-99-003-579-072

VALUE ADDED TAX


Gross Sales ₱1,697,718.90
Output Tax ₱ 154,338.08
Less: Input Tax _____________
VAT Payable ₱ 154,338.08
Add: 25% Surcharge ₱ 38,584.54
20% Interest 79,746.49
Compromise Penalty
Late Payment ₱16,000.00
Failure to File VAT returns 2,400.00 18,400.00 136,731.01
TOTAL ₱ 291,069.09
WITHHOLDING TAX
Compensation 2,772.91
Expanded 110,103.92
Total Tax Due ₱ 112,876.83
22

Less: Tax Withheld 111,848.27


Deficiency Withholding Tax ₱ 1,028.56
Add: 20% Interest p.a. 576.51
Compromise Penalty 200.00
TOTAL ₱ 1,805.07
*Expanded Withholding Tax ₱1,949,334.25 x 5% 97,466.71
Film Rental 10,000.25 x 10% 1,000.00
Audit Fee 193,261.20 x 5% 9,663.00
Rental Expense 41,272.73 x 1% 412.73
Security Service 156,142.01 x 1% 1,561.42
Service Contractor ₱ 110,103.92
Total
SUMMARIES OF DEFICIENCIES
VALUE ADDED TAX ₱ 291,069.09
WITHHOLDING TAX 1,805.07
TOTAL ₱ 292,874.16
Subsequently, Revenue District Office No. 67 sent a copy of the Final Notice of
Seizure dated May 12, 2003, which petitioner received on May 15, 2003, giving
the latter last opportunity to settle its deficiency tax liabilities within ten (10)
[days] from receipt thereof, otherwise respondent BIR shall be constrained to
serve and execute the Warrants of Distraint and/or Levy and Garnishment to
enforce collection.

On February 6, 2004, petitioner received from Revenue District Office No. 67 a


Warrant of Distraint and/or Levy No. 67-0029-23 dated May 12, 2003 demanding
payment of deficiency value-added tax and withholding tax payment in the
amount of ₱292,874.16.

On July 30, 2004, petitioner filed with the Office of respondent Commissioner a
Motion for Reconsideration pursuant to Section 3.1.5 of Revenue Regulations
No. 12-99.

On February 8, 2005, respondent Commissioner, through its authorized


representative, Revenue Regional Director of Revenue Region 10, Legaspi City,
issued a Decision denying petitioner’s Motion for Reconsideration. Petitioner,
through counsel received said Decision on February 18, 2005.

x x x.

Denying that it received a Preliminary Assessment Notice (PAN) and claiming


that it was not accorded due process, Metro Star filed a petition for
23

review4 with the CTA. The parties then stipulated on the following issues to be
decided by the tax court:

1. Whether the respondent complied with the due process requirement as


provided under the National Internal Revenue Code and Revenue
Regulations No. 12-99 with regard to the issuance of a deficiency tax
assessment;

1.1 Whether petitioner is liable for the respective amounts of


₱291,069.09 and ₱1,805.07 as deficiency VAT and withholding tax for
the year 1999;

1.2. Whether the assessment has become final and executory and
demandable for failure of petitioner to protest the same within 30
days from its receipt thereof on April 11, 2002, pursuant to Section
228 of the National Internal Revenue Code;

2. Whether the deficiency assessments issued by the respondent are void


for failure to state the law and/or facts upon which they are based.

2.2 Whether petitioner was informed of the law and facts on which
the assessment is made in compliance with Section 228 of the
National Internal Revenue Code;

3. Whether or not petitioner, as owner/operator of a movie/cinema


house, is subject to VAT on sales of services under Section 108(A) of the
National Internal Revenue Code;

4. Whether or not the assessment is based on the best evidence


obtainable pursuant to Section 6(b) of the National Internal Revenue
Code.

The CTA-Second Division found merit in the petition of Metro Star and, on
March 21, 2007, rendered a decision, the decretal portion of which reads:

WHEREFORE, premises considered, the Petition for Review is hereby GRANTED.


Accordingly, the assailed Decision dated February 8, 2005 is hereby REVERSED
and SET ASIDE and respondent is ORDERED TO DESIST from collecting the
subject taxes against petitioner.

The CTA-Second Division opined that "[w]hile there [is] a disputable


presumption that a mailed letter [is] deemed received by the addressee in the
ordinary course of mail, a direct denial of the receipt of mail shifts the burden
upon the party favored by the presumption to prove that the mailed letter was
indeed received by the addressee."5 It also found that there was no clear
showing that Metro Star actually received the alleged PAN, dated January 16,
2002. It, accordingly, ruled that the Formal Letter of Demand dated April 3,
2002, as well as the Warrant of Distraint and/or Levy dated May 12, 2003 were
void, as Metro Star was denied due process.6

The CIR sought reconsideration7 of the decision of the CTA-Second Division, but
the motion was denied in the latter’s July 24, 2007 Resolution.8
24

Aggrieved, the CIR filed a petition for review9 with the CTA-En Banc, but the
petition was dismissed after a determination that no new matters were raised.
The CTA-En Banc disposed:

WHEREFORE, the instant Petition for Review is hereby DENIED DUE COURSE and
DISMISSED for lack of merit. Accordingly, the March 21, 2007 Decision and July
27, 2007 Resolution of the CTA Second Division in CTA Case No. 7169 entitled,
"Metro Star Superama, Inc., petitioner vs. Commissioner of Internal Revenue,
respondent" are hereby AFFIRMED in toto.

SO ORDERED.

The motion for reconsideration10 filed by the CIR was likewise denied by the
CTA-En Banc in its November 18, 2008 Resolution.11

The CIR, insisting that Metro Star received the PAN, dated January 16, 2002, and
that due process was served nonetheless because the latter received the Final
Assessment Notice (FAN), comes now before this Court with the sole issue of
whether or not Metro Star was denied due process.

The general rule is that the Court will not lightly set aside the conclusions
reached by the CTA which, by the very nature of its functions, has accordingly
developed an exclusive expertise on the resolution unless there has been an
abuse or improvident exercise of authority.12 In Barcelon, Roxas Securities, Inc.
(now known as UBP Securities, Inc.) v. Commissioner of Internal Revenue,13 the
Court wrote:

Jurisprudence has consistently shown that this Court accords the findings of
fact by the CTA with the highest respect. In Sea-Land Service Inc. v. Court of
Appeals [G.R. No. 122605, 30 April 2001, 357 SCRA 441, 445-446], this Court
recognizes that the Court of Tax Appeals, which by the very nature of its
function is dedicated exclusively to the consideration of tax problems, has
necessarily developed an expertise on the subject, and its conclusions will not
be overturned unless there has been an abuse or improvident exercise of
authority. Such findings can only be disturbed on appeal if they are not
supported by substantial evidence or there is a showing of gross error or abuse
on the part of the Tax Court. In the absence of any clear and convincing proof
to the contrary, this Court must presume that the CTA rendered a decision
which is valid in every respect.

On the matter of service of a tax assessment, a further perusal of our ruling in


Barcelon is instructive, viz:

Jurisprudence is replete with cases holding that if the taxpayer denies ever
having received an assessment from the BIR, it is incumbent upon the latter to
prove by competent evidence that such notice was indeed received by the
addressee. The onus probandi was shifted to respondent to prove by contrary
evidence that the Petitioner received the assessment in the due course of mail.
The Supreme Court has consistently held that while a mailed letter is deemed
received by the addressee in the course of mail, this is merely a disputable
presumption subject to controversion and a direct denial thereof shifts the
25

burden to the party favored by the presumption to prove that the mailed letter
was indeed received by the addressee (Republic vs. Court of Appeals, 149
SCRA 351). Thus as held by the Supreme Court in Gonzalo P. Nava vs.
Commissioner of Internal Revenue, 13 SCRA 104, January 30, 1965:

"The facts to be proved to raise this presumption are (a) that the letter was
properly addressed with postage prepaid, and (b) that it was mailed. Once
these facts are proved, the presumption is that the letter was received by the
addressee as soon as it could have been transmitted to him in the ordinary
course of the mail. But if one of the said facts fails to appear, the presumption
does not lie. (VI, Moran, Comments on the Rules of Court, 1963 ed, 56-57 citing
Enriquez vs. Sunlife Assurance of Canada, 41 Phil 269)."

x x x. What is essential to prove the fact of mailing is the registry receipt issued
by the Bureau of Posts or the Registry return card which would have been
signed by the Petitioner or its authorized representative. And if said documents
cannot be located, Respondent at the very least, should have submitted to the
Court a certification issued by the Bureau of Posts and any other pertinent
document which is executed with the intervention of the Bureau of Posts. This
Court does not put much credence to the self serving documentations made
by the BIR personnel especially if they are unsupported by substantial evidence
establishing the fact of mailing. Thus:

"While we have held that an assessment is made when sent within the
prescribed period, even if received by the taxpayer after its expiration (Coll. of
Int. Rev. vs. Bautista, L-12250 and L-12259, May 27, 1959), this ruling makes it the
more imperative that the release, mailing or sending of the notice be clearly
and satisfactorily proved. Mere notations made without the taxpayer’s
intervention, notice or control, without adequate supporting evidence cannot
suffice; otherwise, the taxpayer would be at the mercy of the revenue offices,
without adequate protection or defense." (Nava vs. CIR, 13 SCRA 104, January
30, 1965).

x x x.

The failure of the respondent to prove receipt of the assessment by the


Petitioner leads to the conclusion that no assessment was issued. Consequently,
the government’s right to issue an assessment for the said period has already
prescribed. (Industrial Textile Manufacturing Co. of the Phils., Inc. vs. CIR CTA
Case 4885, August 22, 1996). (Emphases supplied.)

The Court agrees with the CTA that the CIR failed to discharge its duty and
present any evidence to show that Metro Star indeed received the PAN dated
January 16, 2002. It could have simply presented the registry receipt or the
certification from the postmaster that it mailed the PAN, but failed. Neither did
it offer any explanation on why it failed to comply with the requirement of
service of the PAN. It merely accepted the letter of Metro Star’s chairman
dated April 29, 2002, that stated that he had received the FAN dated April 3,
2002, but not the PAN; that he was willing to pay the tax as computed by the
CIR; and that he just wanted to clarify some matters with the hope of lessening
its tax liability.
26

This now leads to the question: Is the failure to strictly comply with notice
requirements prescribed under Section 228 of the National Internal Revenue
Code of 1997 and Revenue Regulations (R.R.) No. 12-99 tantamount to a denial
of due process? Specifically, are the requirements of due process satisfied if
only the FAN stating the computation of tax liabilities and a demand to pay
within the prescribed period was sent to the taxpayer?

The answer to these questions require an examination of Section 228 of the Tax
Code which reads:

SEC. 228. Protesting of Assessment. - When the Commissioner or his duly


authorized representative finds that proper taxes should be assessed, he shall
first notify the taxpayer of his findings: provided, however, that a preassessment
notice shall not be required in the following cases:

(a) When the finding for any deficiency tax is the result of mathematical
error in the computation of the tax as appearing on the face of the return;
or

(b) When a discrepancy has been determined between the tax withheld
and the amount actually remitted by the withholding agent; or

(c) When a taxpayer who opted to claim a refund or tax credit of excess
creditable withholding tax for a taxable period was determined to have
carried over and automatically applied the same amount claimed
against the estimated tax liabilities for the taxable quarter or quarters of
the succeeding taxable year; or

(d) When the excise tax due on exciseable articles has not been paid; or

(e) When the article locally purchased or imported by an exempt person,


such as, but not limited to, vehicles, capital equipment, machineries and
spare parts, has been sold, traded or transferred to non-exempt persons.

The taxpayers shall be informed in writing of the law and the facts on which the
assessment is made; otherwise, the assessment shall be void.

Within a period to be prescribed by implementing rules and regulations, the


taxpayer shall be required to respond to said notice. If the taxpayer fails to
respond, the Commissioner or his duly authorized representative shall issue an
assessment based on his findings.

Such assessment may be protested administratively by filing a request for


reconsideration or reinvestigation within thirty (30) days from receipt of the
assessment in such form and manner as may be prescribed by implementing
rules and regulations. Within sixty (60) days from filing of the protest, all relevant
supporting documents shall have been submitted; otherwise, the assessment
shall become final.

If the protest is denied in whole or in part, or is not acted upon within one
hundred eighty (180) days from submission of documents, the taxpayer
adversely affected by the decision or inaction may appeal to the Court of Tax
27

Appeals within thirty (30) days from receipt of the said decision, or from the
lapse of one hundred eighty (180)-day period; otherwise, the decision shall
become final, executory and demandable. (Emphasis supplied).

Indeed, Section 228 of the Tax Code clearly requires that the taxpayer must first
be informed that he is liable for deficiency taxes through the sending of a PAN.
He must be informed of the facts and the law upon which the assessment is
made. The law imposes a substantive, not merely a formal, requirement. To
proceed heedlessly with tax collection without first establishing a valid
assessment is evidently violative of the cardinal principle in administrative
investigations - that taxpayers should be able to present their case and adduce
supporting evidence.14

This is confirmed under the provisions R.R. No. 12-99 of the BIR which pertinently
provide:

SECTION 3. Due Process Requirement in the Issuance of a Deficiency Tax


Assessment. —

3.1 Mode of procedures in the issuance of a deficiency tax assessment:

3.1.1 Notice for informal conference. — The Revenue Officer who audited
the taxpayer's records shall, among others, state in his report whether or
not the taxpayer agrees with his findings that the taxpayer is liable for
deficiency tax or taxes. If the taxpayer is not amenable, based on the said
Officer's submitted report of investigation, the taxpayer shall be informed,
in writing, by the Revenue District Office or by the Special Investigation
Division, as the case may be (in the case Revenue Regional Offices) or by
the Chief of Division concerned (in the case of the BIR National Office) of
the discrepancy or discrepancies in the taxpayer's payment of his internal
revenue taxes, for the purpose of "Informal Conference," in order to afford
the taxpayer with an opportunity to present his side of the case. If the
taxpayer fails to respond within fifteen (15) days from date of receipt of
the notice for informal conference, he shall be considered in default, in
which case, the Revenue District Officer or the Chief of the Special
Investigation Division of the Revenue Regional Office, or the Chief of
Division in the National Office, as the case may be, shall endorse the case
with the least possible delay to the Assessment Division of the Revenue
Regional Office or to the Commissioner or his duly authorized
representative, as the case may be, for appropriate review and issuance
of a deficiency tax assessment, if warranted.

3.1.2 Preliminary Assessment Notice (PAN). — If after review and


evaluation by the Assessment Division or by the Commissioner or his duly
authorized representative, as the case may be, it is determined that there
exists sufficient basis to assess the taxpayer for any deficiency tax or taxes,
the said Office shall issue to the taxpayer, at least by registered mail, a
Preliminary Assessment Notice (PAN) for the proposed assessment,
showing in detail, the facts and the law, rules and regulations, or
jurisprudence on which the proposed assessment is based (see illustration
in ANNEX A hereof). If the taxpayer fails to respond within fifteen (15) days
28

from date of receipt of the PAN, he shall be considered in default, in


which case, a formal letter of demand and assessment notice shall be
caused to be issued by the said Office, calling for payment of the
taxpayer's deficiency tax liability, inclusive of the applicable penalties.

3.1.3 Exceptions to Prior Notice of the Assessment. — The notice for


informal conference and the preliminary assessment notice shall not be
required in any of the following cases, in which case, issuance of the
formal assessment notice for the payment of the taxpayer's deficiency tax
liability shall be sufficient:

(i) When the finding for any deficiency tax is the result of
mathematical error in the computation of the tax appearing on the
face of the tax return filed by the taxpayer; or

(ii) When a discrepancy has been determined between the tax


withheld and the amount actually remitted by the withholding
agent; or

(iii) When a taxpayer who opted to claim a refund or tax credit of


excess creditable withholding tax for a taxable period was
determined to have carried over and automatically applied the
same amount claimed against the estimated tax liabilities for the
taxable quarter or quarters of the succeeding taxable year; or

(iv) When the excise tax due on excisable articles has not been paid;
or

(v) When an article locally purchased or imported by an exempt


person, such as, but not limited to, vehicles, capital equipment,
machineries and spare parts, has been sold, traded or transferred to
non-exempt persons.

3.1.4 Formal Letter of Demand and Assessment Notice. — The formal letter
of demand and assessment notice shall be issued by the Commissioner or
his duly authorized representative. The letter of demand calling for
payment of the taxpayer's deficiency tax or taxes shall state the facts, the
law, rules and regulations, or jurisprudence on which the assessment is
based, otherwise, the formal letter of demand and assessment notice
shall be void (see illustration in ANNEX B hereof).

The same shall be sent to the taxpayer only by registered mail or by personal
delivery.

If sent by personal delivery, the taxpayer or his duly authorized representative


shall acknowledge receipt thereof in the duplicate copy of the letter of
demand, showing the following: (a) His name; (b) signature; (c) designation
and authority to act for and in behalf of the taxpayer, if acknowledged
received by a person other than the taxpayer himself; and (d) date of receipt
thereof.

x x x.
29

From the provision quoted above, it is clear that the sending of a PAN to
taxpayer to inform him of the assessment made is but part of the "due process
requirement in the issuance of a deficiency tax assessment," the absence of
which renders nugatory any assessment made by the tax authorities. The use of
the word "shall" in subsection 3.1.2 describes the mandatory nature of the
service of a PAN. The persuasiveness of the right to due process reaches both
substantial and procedural rights and the failure of the CIR to strictly comply
with the requirements laid down by law and its own rules is a denial of Metro
Star’s right to due process.15 Thus, for its failure to send the PAN stating the facts
and the law on which the assessment was made as required by Section 228 of
R.A. No. 8424, the assessment made by the CIR is void.

The case of CIR v. Menguito16 cited by the CIR in support of its argument that
only the non-service of the FAN is fatal to the validity of an assessment, cannot
apply to this case because the issue therein was the non-compliance with the
provisions of R. R. No. 12-85 which sought to interpret Section 229 of the old tax
law. RA No. 8424 has already amended the provision of Section 229 on
protesting an assessment. The old requirement of merely notifying the taxpayer
of the CIR’s findings was changed in 1998 to informing the taxpayer of not only
the law, but also of the facts on which an assessment would be made.
Otherwise, the assessment itself would be invalid.17 The regulation then, on the
other hand, simply provided that a notice be sent to the respondent in the form
prescribed, and that no consequence would ensue for failure to comply with
that form.1avvphi1

The Court need not belabor to discuss the matter of Metro Star’s failure to file its
protest, for it is well-settled that a void assessment bears no fruit.18

It is an elementary rule enshrined in the 1987 Constitution that no person shall


be deprived of property without due process of law.19 In balancing the scales
between the power of the State to tax and its inherent right to prosecute
perceived transgressors of the law on one side, and the constitutional rights of a
citizen to due process of law and the equal protection of the laws on the other,
the scales must tilt in favor of the individual, for a citizen’s right is amply
protected by the Bill of Rights under the Constitution. Thus, while "taxes are the
lifeblood of the government," the power to tax has its limits, in spite of all its
plenitude. Hence in Commissioner of Internal Revenue v. Algue, Inc.,20 it was
said –

Taxes are the lifeblood of the government and so should be collected without
unnecessary hindrance. On the other hand, such collection should be made in
accordance with law as any arbitrariness will negate the very reason for
government itself. It is therefore necessary to reconcile the apparently
conflicting interests of the authorities and the taxpayers so that the real purpose
of taxation, which is the promotion of the common good, may be achieved.

xxx xxx xxx

It is said that taxes are what we pay for civilized society. Without taxes, the
government would be paralyzed for the lack of the motive power to activate
and operate it. Hence, despite the natural reluctance to surrender part of
30

one’s hard-earned income to taxing authorities, every person who is able to


must contribute his share in the running of the government. The government for
its part is expected to respond in the form of tangible and intangible benefits
intended to improve the lives of the people and enhance their moral and
material values. This symbiotic relationship is the rationale of taxation and
should dispel the erroneous notion that it is an arbitrary method of exaction by
those in the seat of power.

But even as we concede the inevitability and indispensability of taxation, it is a


requirement in all democratic regimes that it be exercised reasonably and in
accordance with the prescribed procedure. If it is not, then the taxpayer has a
right to complain and the courts will then come to his succor. For all the
awesome power of the tax collector, he may still be stopped in his tracks if the
taxpayer can demonstrate x x x that the law has not been
observed.21 (Emphasis supplied).

WHEREFORE, the petition is DENIED.

SO ORDERED.

G.R. No. 174942 March 7, 2008

BANK OF THE PHILIPPINE ISLANDS (Formerly: Far East Bank and Trust
Company), petitioner,
vs.
COMMISSIONER OF INTERNAL REVENUE,

TINGA, J.:

The Bank of the Philippine Islands (BPI) seeks a review of the Decision1dated 15
August 2006 and the Resolution2dated 5 October 2006, both of the Court of Tax
Appeals (CTA or tax court), which ruled that BPI is liable for the deficiency
documentary stamp tax (DST) on its cabled instructions to its foreign
correspondent bank and that prescription had not yet set in against the
government.

The following undisputed facts are culled from the CTA decision:

Petitioner, the surviving bank after its merger with Far East Bank and Trust
Company, is a corporation duly created and existing under the laws of
the Republic of the Philippines with principal office at Ayala Avenue
corner Paseo de Roxas Ave., Makati City.

Respondent thru then Revenue Service Chief Cesar M. Valdez, issued to


the petitioner a pre-assessment notice (PAN) dated November 26, 1986.

Petitioner, in a letter dated November 29, 1986, requested for the details
of the amounts alleged as 1982-1986 deficiency taxes mentioned in the
November 26, 1986 PAN.

On April 7, 1989, respondent issued to the petitioner, assessment/demand


notices FAS-1-82 to 86/89-000 and FAS 5-82 to 86/89-000 for deficiency
31

withholding tax at source (Swap Transactions) and DST involving the


amounts of P190,752,860.82 and P24,587,174.63, respectively, for the years
1982 to 1986.

On April 20, 1989, petitioner filed a protest on the demand/assessment


notices. On May 8, 1989, petitioner filed a supplemental protest.

On March 12, 1993, petitioner requested for an opportunity to present or


submit additional documentation on the Swap Transactions with the then
Central Bank (page 240, BIR Records). Attached to the letter dated June
17, 1994, in connection with the reinvestigation of the abovementioned
assessment, petitioner submitted to the BIR, Swap Contracts with the
Central Bank.

Petitioner executed several Waivers of the Statutes of Limitations, the last


of which was effective until December 31, 1994.

On August 9, 2002, respondent issued a final decision on petitioner’s


protest ordering the withdrawal and cancellation of the deficiency
withholding tax assessment in the amount of P190,752,860.82 and
considered the same as closed and terminated. On the other hand, the
deficiency DST assessment in the amount of P24,587,174.63 was reiterated
and the petitioner was ordered to pay the said amount within thirty (30)
days from receipt of such order. Petitioner received a copy of the said
decision on January 15, 2003. Thereafter, on January 24, 2003, petitioner
filed a Petition for Review before the Court.

On August 31, 2004, the Court rendered a Decision denying the


petitioner’s Petition for Review, the dispositive portion of which is quoted
hereunder:

IN VIEW OF ALL THE FOREGOING, the petition is hereby DENIED for


lack of merit. Accordingly, petitioner is ORDERED to PAY the
respondent the amount of P24,587,174.63 representing deficiency
documentary stamp tax for the period 1982-1986, plus 20% interest
starting February 14, 2003 until the amount is fully paid pursuant to
Section 249 of the Tax Code.

SO ORDERED.

On September 21, 2004, petitioner filed a Motion for Reconsideration of


the abovementioned Decision which was denied for lack of merit in a
Resolution dated February 14, 2005.

On March 9, 2005, petitioner filed with the Court En Banc a Motion for
Extension of Time to File Petition for Review praying for an extension of
fifteen (15) days from March 10, 2005 or until March 25, 2005. Petitioner’s
motion was granted in a Resolution dated March 16, 2005.

On March 28, 2005, (March 25 was Good Friday), petitioner filed the
instant Petition for Review, advancing the following assignment of errors.
32

I. THIS HONORABLE COURT OVERLOOKED THE SIGNIFICANCE OF


THE WAIVER DULY AND VALIDLY AGREED UPON BY THE PARTIES AND
EFFECTIVE UNTIL DECEMBER 31, 1994;

II. THIS TAX COURT ERRED IN HOLDING THAT THE COLLECTION OF


ALLEGED DEFICIENCY TAX HAS NOT PRESCRIBED.

III. THIS HONORABLE COURT ERRED IN HOLDING THAT RESPONDENT


DID NOT VIOLATE PROCEDURAL DUE PROCESS IN THE ISSUANCE OF
ASSESSMENT NOTICE RELATIVE TO DOCUMENTARY STAMP
DEFICIENCY.

IV. THIS HONORABLE COURT ERRED IN HOLDING THAT THE 4 MARCH


1987 MEMORANDUM OF THE LEGAL SERVICE CHIEF DULY APPROVED
BY THE BIR COMMISISONER VESTS NO RIGHTS TO PETITIONER.

V. THIS HONORABLE COURT ERRED IN HOLDING THAT PETITIONER IS


LIABLE FOR DOCUMENTARY STAMP TAX ON SWAP LOANS
TRANSACTIONS FROM 1982 TO 1986.3

The CTA synthesized the foregoing issues into whether the collection of the
deficiency DST is barred by prescription and whether BPI is liable for DST on its
SWAP loan transactions.

On the first issue, the tax court, applying the case of Commissioner of Internal
Revenue v. Wyeth Suaco Laboratories, Inc.,4(Wyeth Suaco case), ruled that
BPI’s protest and supplemental protest should be considered requests for
reinvestigation which tolled the prescriptive period provided by law to collect a
tax deficiency by distraint, levy, or court proceeding. It further held, as regards
the second issue, that BPI’s cabled instructions to its foreign correspondent
bank to remit a specific sum in dollars to the Federal Reserve Bank, the same to
be credited to the account of the Central Bank, are in the nature of a
telegraphic transfer subject to DST under Section 195 of the Tax Code.

In its Petition for Review5 dated 24 November 2006, BPI argues that the
government’s right to collect the DST had already prescribed because the
Commissioner of Internal Revenue (CIR) failed to issue any reply granting BPI’s
request for reinvestigation manifested in the protest letters dated 20 April and 8
May 1989. It was only through the 9 August 2002 Decision ordering BPI to pay
deficiency DST, or after the lapse of more than thirteen (13) years, that the CIR
acted on the request for reinvestigation, warranting the conclusion that
prescription had already set in. It further claims that the CIR was not precluded
from collecting the deficiency within three (3) years from the time the notice of
assessment was issued on 7 April 1989, or even until the expiration on 31
December 1994 of the last waiver of the statute of limitations signed by BPI.

Moreover, BPI avers that the cabled instructions to its correspondent bank are
not subject to DST because the National Internal Revenue Code of 1977 (Tax
Code of 1977) does not contain a specific provision that cabled instructions on
SWAP transactions are subject to DST.
33

The Office of the Solicitor General (OSG) filed a Comment6 dated 1 June 2007,
on behalf of the CIR, asserting that the prescriptive period was tolled by the
protest letters filed by BPI which were granted and acted upon by the CIR.
Such action was allegedly communicated to BPI as, in fact, the latter submitted
additional documents pertaining to its SWAP transactions in support of its
request for reinvestigation. Thus, it was only upon BPI’s receipt on 13 January
2003 of the 9 August 2002 Decision that the period to collect commenced to
run again.

The OSG cites the case of Collector of Internal Revenue v. Suyoc Consolidated
Mining Company, et al.7(Suyoc case) in support of its argument that BPI is
already estopped from raising the defense of prescription in view of its
repeated requests for reinvestigation which allegedly induced the CIR to delay
the collection of the assessed tax.

In its Reply8dated 30 August 2007, BPI argues against the application of


the Suyoc case on two points: first, it never induced the CIR to postpone tax
collection; second, its request for reinvestigation was not categorically acted
upon by the CIR within the three-year collection period after assessment. BPI
maintains that it did not receive any communication from the CIR in reply to its
protest letters.

We grant the petition.

Section 3189 of the Tax Code of 1977 provides:

Sec. 318. Period of limitation upon assessment and collection.—Except as


provided in the succeeding section, internal revenue taxes shall be
assessed within five years after the return was filed, and no proceeding in
court without assessment for the collection of such taxes shall be begun
after the expiration of such period. For the purposes of this section, a
return filed before the last day prescribed by law for the filing thereof shall
be considered as filed on such last day: Provided, That this limitation shall
not apply to cases already investigated prior to the approval of this Code.

The statute of limitations on assessment and collection of national internal


revenue taxes was shortened from five (5) years to three (3) years by Batas
Pambansa Blg. 700.10 Thus, the CIR has three (3) years from the date of actual
filing of the tax return to assess a national internal revenue tax or to commence
court proceedings for the collection thereof without an assessment.

When it validly issues an assessment within the three (3)-year period, it has
another three (3) years within which to collect the tax due by distraint, levy, or
court proceeding. The assessment of the tax is deemed made and the three
(3)-year period for collection of the assessed tax begins to run on the date the
assessment notice had been released, mailed or sent to the taxpayer.11

As applied to the present case, the CIR had three (3) years from the time he
issued assessment notices to BPI on 7 April 1989 or until 6 April 1992 within which
to collect the deficiency DST. However, it was only on 9 August 2002 that the
CIR ordered BPI to pay the deficiency.
34

In order to determine whether the prescriptive period for collecting the tax
deficiency was effectively tolled by BPI’s filing of the protest letters dated 20
April and 8 May 1989 as claimed by the CIR, we need to examine Section
32012 of the Tax Code of 1977, which states:

Sec. 320. Suspension of running of statute.—The running of the statute of


limitations provided in Sections 318 or 319 on the making of assessment
and the beginning of distraint or levy or a proceeding in court for
collection, in respect of any deficiency, shall be suspended for the period
during which the Commissioner is prohibited from making the assessment
or beginning distraint or levy or a proceeding in court and for sixty days
thereafter; when the taxpayer requests for a re-investigation which is
granted by the Commissioner; when the taxpayer cannot be located in
the address given by him in the return filed upon which a tax is being
assessed or collected: Provided, That if the taxpayer informs the
Commissioner of any change in address, the running of the statute of
limitations will not be suspended; when the warrant of distraint and levy is
duly served upon the taxpayer, his authorized representative, or a
member of his household with sufficient discretion, and no property could
be located; and when the taxpayer is out of the Philippines. (Emphasis
supplied)

The above section is plainly worded. In order to suspend the running of the
prescriptive periods for assessment and collection, the request for
reinvestigation must be granted by the CIR.

In BPI v. Commissioner of Internal Revenue,13the Court emphasized the rule that


the CIR must first grant the request for reinvestigation as a requirement for the
suspension of the statute of limitations. The Court said:

In the case of Republic of the Philippines v. Gancayco, taxpayer


Gancayco requested for a thorough reinvestigation of the assessment
against him and placed at the disposal of the Collector of Internal
Revenue all the evidences he had for such purpose; yet, the Collector
ignored the request, and the records and documents were not at all
examined. Considering the given facts, this Court pronounced that—

x x x The act of requesting a reinvestigation alone does not suspend the


period. The request should first be granted, in order to effect
suspension. (Collector v. Suyoc Consolidated, supra; also Republic v.
Ablaza, supra). Moreover, the Collector gave appellee until April 1, 1949,
within which to submit his evidence, which the latter did one day before.
There were no impediments on the part of the Collector to file the
collection case from April 1, 1949…

In Republic of the Philippines v. Acebedo, this Court similarly found that—

x x x T]he defendant, after receiving the assessment notice of September


24, 1949, asked for a reinvestigation thereof on October 11, 1949 (Exh.
"A"). There is no evidence that this request was considered or acted
upon. In fact, on October 23, 1950 the then Collector of Internal Revenue
35

issued a warrant of distraint and levy for the full amount of the assessment
(Exh. "D"), but there was follow-up of this warrant. Consequently, the
request for reinvestigation did not suspend the running of the period for
filing an action for collection. [Emphasis in the original]14

The Court went on to declare that the burden of proof that the request for
reinvestigation had been actually granted shall be on the CIR. Such grant may
be expressed in its communications with the taxpayer or implied from the
action of the CIR or his authorized representative in response to the request for
reinvestigation.

There is nothing in the records of this case which indicates, expressly or


impliedly, that the CIR had granted the request for reinvestigation filed by BPI.
What is reflected in the records is the piercing silence and inaction of the CIR
on the request for reinvestigation, as he considered BPI’s letters of protest to be.

In fact, it was only in his comment to the present petition that the CIR, through
the OSG, argued for the first time that he had granted the request for
reinvestigation. His consistent stance invoking the Wyeth Suaco case, as
reflected in the records, is that the prescriptive period was tolled by BPI’s
request for reinvestigation, without any assertion that the same had been
granted or at least acted upon.15

In the Wyeth Suaco case, private respondent Wyeth Suaco Laboratories, Inc.
sent letters seeking the reinvestigation or reconsideration of the deficiency tax
assessments issued by the BIR. The records of the case showed that as a result
of these protest letters, the BIR Manufacturing Audit Division conducted a
review and reinvestigation of the assessments. The records further showed that
the company, thru its finance manager, communicated its inability to settle the
tax deficiency assessment and admitted that it knew of the ongoing review
and consideration of its protest.

As differentiated from the Wyeth Suaco case, however, there is no evidence in


this case that the CIR actually conducted a reinvestigation upon the request of
BPI or that the latter was made aware of the action taken on its request.
Hence, there is no basis for the tax court’s ruling that the filing of the request for
reinvestigation tolled the running of the prescriptive period for collecting the tax
deficiency.

Neither did the waiver of the statute of limitations signed by BPI supposedly
effective until 31 December 1994 suspend the prescriptive period. The CIR
himself contends that the waiver is void as it shows no date of acceptance in
violation of RMO No. 20-90.16 At any rate, the records of this case do not
disclose any effort on the part of the Bureau of Internal Revenue to collect the
deficiency tax after the expiration of the waiver until eight (8) years thereafter
when it finally issued a decision on the protest.

We also find the Suyoc case inapplicable. In that case, several requests for
reinvestigation and reconsideration were filed by Suyoc Consolidated Mining
Company purporting to question the correctness of tax assessments against it.
As a result, the Collector of Internal Revenue refrained from collecting the tax
36

by distraint, levy or court proceeding in order to give the company every


opportunity to prove its claim. The Collector also conducted several
reinvestigations which eventually led to a reduced assessment. The company,
however, filed a petition with the CTA claiming that the right of the government
to collect the tax had already prescribed.

When the case reached this Court, we ruled that Suyoc could not set up the
defense of prescription since, by its own action, the government was induced
to delay the collection of taxes to make the company feel that the demand
was not unreasonable or that no harassment or injustice was meant by the
government.

In this case, BPI’s letters of protest and submission of additional documents


pertaining to its SWAP transactions, which were never even acted upon, much
less granted, cannot be said to have persuaded the CIR to postpone the
collection of the deficiency DST.

The inordinate delay of the CIR in acting upon and resolving the request for
reinvestigation filed by BPI and in collecting the DST allegedly due from the
latter had resulted in the prescription of the government’s right to collect the
deficiency. As this Court declared in Republic of the Philippines v. Ablaza:17

The law prescribing a limitation of actions for the collection of the income
tax is beneficial both to the Government and to its citizens; to the
Government because tax officers would be obliged to act promptly in the
making of assessment, and to citizens because after the lapse of the
period of prescription citizens would have a feeling of security against
unscrupulous tax agents who will always find an excuse to inspect the
books of taxpayers, not to determine the latter’s real liability, but to take
advantage of every opportunity to molest peaceful, law-abiding citizens.
Without such a legal defense taxpayers would furthermore be under
obligation to always keep their books and keep them open for inspection
subject to harassment by unscrupulous tax agents. The law on prescription
being a remedial measure should be interpreted in a way conducive to
bringing about the beneficent purpose of affording protection to the
taxpayer within the contemplation of the Commission which recommend
the approval of the law.18

Given the prescription of the government’s claim, we no longer deem it


necessary to pass upon the validity of the assessment.

WHEREFORE, the petition is GRANTED. The Decisionof the Court of Tax Appeals
dated 15 August 2006 and its Resolution dated 5 October 2006, are hereby
REVERSED and SET ASIDE. No pronouncement as to costs.

SO ORDERED.

G.R. No. 166387 January 19, 2009


37

COMMISSIONER OF INTERNAL REVENUE, Petitioners,


vs.
ENRON SUBIC POWERCORPORATION, Respondents.

RESOLUTION

CORONA, J.:

In this petition for review on certiorari under Rule 45 of the Rules of Court,
petitioner Commissioner of Internal Revenue (CIR) assails the November 24,
2004 decision1 of the Court of Appeals (CA) annulling the formal assessment
notice issued by the CIR against respondent Enron Subic Power Corporation
(Enron) for failure to state the legal and factual bases for such assessment.

Enron, a domestic corporation registered with the Subic Bay Metropolitan


Authority as a freeport enterprise,2 filed its annual income tax return for the year
1996 on April 12, 1997. It indicated a net loss of P7,684,948. Subsequently, the
Bureau of Internal Revenue, through a preliminary five-day letter,3 informed it of
a proposed assessment of an alleged P2,880,817.25 deficiency income
tax.4 Enron disputed the proposed deficiency assessment in its first protest
letter.5

On May 26, 1999, Enron received from the CIR a formal assessment
notice6 requiring it to pay the alleged deficiency income tax of P2,880,817.25
for the taxable year 1996. Enron protested this deficiency tax assessment.7

Due to the non-resolution of its protest within the 180-day period, Enron filed a
petition for review in the Court of Tax Appeals (CTA). It argued that the
deficiency tax assessment disregarded the provisions of Section 228 of the
National Internal Revenue Code (NIRC), as amended,8and Section 3.1.4 of
Revenue Regulations (RR) No. 12-999 by not providing the legal and factual
bases of the assessment. Enron likewise questioned the substantive validity of
the assessment.10

In a decision dated September 12, 2001, the CTA granted Enron’s petition and
ordered the cancellation of its deficiency tax assessment for the year 1996. The
CTA reasoned that the assessment notice sent to Enron failed to comply with
the requirements of a valid written notice under Section 228 of the NIRC and RR
No. 12-99. The CIR’s motion for reconsideration of the CTA decision was denied
in a resolution dated November 12, 2001.

The CIR appealed the CTA decision to the CA but the CA affirmed it. The CA
held that the audit working papers did not substantially comply with Section
228 of the NIRC and RR No. 12-99 because they failed to show the applicability
of the cited law to the facts of the assessment. The CIR filed a motion for
reconsideration but this was deemed abandoned when he filed a motion for
extension to file a petition for review in this Court.

The CIR now argues that respondent was informed of the legal and factual
bases of the deficiency assessment against it.
38

We adopt in toto the findings of fact of the CTA, as affirmed by the CA.
In Compagnie Financiere Sucres et Denrees v. CIR,11 we held:

We reiterate the well-established doctrine that as a matter of practice and


principle, [we] will not set aside the conclusion reached by an agency, like the
CTA, especially if affirmed by the [CA]. By the very nature of its function, it has
dedicated itself to the study and consideration of tax problems and has
necessarily developed an expertise on the subject, unless there has been an
abuse or improvident exercise of authority on its part, which is not present here.

The CIR errs in insisting that the notice of assessment in question complied with
the requirements of the NIRC and RR No. 12-99.

A notice of assessment is:

[A] declaration of deficiency taxes issued to a [t]axpayer who fails to respond


to a Pre-Assessment Notice (PAN) within the prescribed period of time, or
whose reply to the PAN was found to be without merit. The Notice of
Assessment shall inform the [t]axpayer of this fact, and that the report of
investigation submitted by the Revenue Officer conducting the audit shall be
given due course.

The formal letter of demand calling for payment of the taxpayer’s deficiency
tax or taxes shall state the fact, the law, rules and regulations or jurisprudence
on which the assessment is based, otherwise the formal letter of demand and
the notice of assessment shall be void. (emphasis supplied)12

Section 228 of the NIRC provides that the taxpayer shall be informed in writing
of the law and the facts on which the assessment is made. Otherwise, the
assessment is void. To implement the provisions of Section 228 of the NIRC, RR
No. 12-99 was enacted. Section 3.1.4 of the revenue regulation reads:

3.1.4. Formal Letter of Demand and Assessment Notice. – The formal letter of
demand and assessment notice shall be issued by the Commissioner or his duly
authorized representative. The letter of demand calling for payment of the
taxpayer’s deficiency tax or taxes shall state the facts, the law, rules and
regulations, or jurisprudence on which the assessment is based, otherwise, the
formal letter of demand and assessment notice shall be void. The same shall be
sent to the taxpayer only by registered mail or by personal delivery. xxx
(emphasis supplied)

It is clear from the foregoing that a taxpayer must be informed in writing of the
legal and factual bases of the tax assessment made against him. The use of the
word “shall” in these legal provisions indicates the mandatory nature of the
requirements laid down therein. We note the CTA’s findings:

In [this] case, [the CIR] merely issued a formal assessment and indicated therein
the supposed tax, surcharge, interest and compromise penalty due thereon.
The Revenue Officers of the [the CIR] in the issuance of the Final Assessment
Notice did not provide Enron with the written bases of the law and facts on
which the subject assessment is based. [The CIR] did not bother to explain how
39

it arrived at such an assessment. Moreso, he failed to mention the specific


provision of the Tax Code or rules and regulations which were not complied
with by Enron.13

Both the CTA and the CA concluded that the deficiency tax assessment merely
itemized the deductions disallowed and included these in the gross income. It
also imposed the preferential rate of 5% on some items categorized by Enron as
costs. The legal and factual bases were, however, not indicated.

The CIR insists that an examination of the facts shows that Enron was properly
apprised of its tax deficiency. During the pre-assessment stage, the CIR advised
Enron’s representative of the tax deficiency, informed it of the proposed tax
deficiency assessment through a preliminary five-day letter and furnished Enron
a copy of the audit working paper14 allegedly showing in detail the legal and
factual bases of the assessment. The CIR argues that these steps sufficed to
inform Enron of the laws and facts on which the deficiency tax assessment was
based.

We disagree. The advice of tax deficiency, given by the CIR to an employee of


Enron, as well as the preliminary five-day letter, were not valid substitutes for the
mandatory notice in writing of the legal and factual bases of the assessment.
These steps were mere perfunctory discharges of the CIR’s duties in correctly
assessing a taxpayer.15 The requirement for issuing a preliminary or final notice,
as the case may be, informing a taxpayer of the existence of a deficiency tax
assessment is markedly different from the requirement of what such notice must
contain. Just because the CIR issued an advice, a preliminary letter during the
pre-assessment stage and a final notice, in the order required by law, does not
necessarily mean that Enron was informed of the law and facts on which the
deficiency tax assessment was made.

The law requires that the legal and factual bases of the assessment be stated in
the formal letter of demand and assessment notice. Thus, such cannot be
presumed. Otherwise, the express provisions of Article 228 of the NIRC and RR
No. 12-99 would be rendered nugatory. The alleged “factual bases” in the
advice, preliminary letter and “audit working papers” did not suffice. There was
no going around the mandate of the law that the legal and factual bases of
the assessment be stated in writing in the formal letter of demand
accompanying the assessment notice.

We note that the old law merely required that the taxpayer be notified of the
assessment made by the CIR. This was changed in 1998 and the taxpayer must
now be informed not only of the law but also of the facts on which the
assessment is made.16 Such amendment is in keeping with the constitutional
principle that no person shall be deprived of property without due process.17 In
view of the absence of a fair opportunity for Enron to be informed of the legal
and factual bases of the assessment against it, the assessment in question was
void. We reiterate our ruling in Reyes v. Almanzor, et al.:18

Verily, taxes are the lifeblood of the Government and so should be collected
without unnecessary hindrance. However, such collection should be made in
40

accordance with law as any arbitrariness will negate the very reason for the
Government itself.

WHEREFORE, the petition is hereby DENIED. The November 24, 2004 decision of
the Court of Appeals isAFFIRMED.

No costs.

SO ORDERED.

G.R. No. 162155 August 28, 2007

COMMISSIONER OF INTERNAL REVENUE and ARTURO V. PARCERO in his official


capacity as Revenue District Officer of Revenue District No. 049
(Makati), Petitioners,
vs.
PRIMETOWN PROPERTY GROUP, INC., Respondent.

DECISION

CORONA, J.:

This petition for review on certiorari1 seeks to set aside the August 1, 2003
decision2 of the Court of Appeals (CA) in CA-G.R. SP No. 64782 and its February
9, 2004 resolution denying reconsideration.3

On March 11, 1999, Gilbert Yap, vice chair of respondent Primetown Property
Group, Inc., applied for the refund or credit of income tax respondent paid in
1997. In Yap's letter to petitioner revenue district officer Arturo V. Parcero of
Revenue District No. 049 (Makati) of the Bureau of Internal Revenue (BIR),4 he
explained that the increase in the cost of labor and materials and difficulty in
obtaining financing for projects and collecting receivables caused the real
estate industry to slowdown.5 As a consequence, while business was good
during the first quarter of 1997, respondent suffered losses amounting to
₱71,879,228 that year.6

According to Yap, because respondent suffered losses, it was not liable for
income taxes.7 Nevertheless, respondent paid its quarterly corporate income
tax and remitted creditable withholding tax from real estate sales to the BIR in
the total amount of ₱26,318,398.32.8 Therefore, respondent was entitled to tax
refund or tax credit.9

On May 13, 1999, revenue officer Elizabeth Y. Santos required respondent to


submit additional documents to support its claim.10 Respondent complied but
its claim was not acted upon. Thus, on April 14, 2000, it filed a petition for
review11 in the Court of Tax Appeals (CTA).

On December 15, 2000, the CTA dismissed the petition as it was filed beyond
the two-year prescriptive period for filing a judicial claim for tax refund or tax
credit.12 It invoked Section 229 of the National Internal Revenue Code (NIRC):
41

Sec. 229. Recovery of Taxes Erroneously or Illegally Collected. -- No suit or


proceeding shall be maintained in any court for the recovery of any national
internal revenue tax hereafter alleged to have been erroneously or illegally
assessed or collected, or of any penalty claimed to have been collected
without authority, or of any sum alleged to have been excessively or in any
manner wrongfully collected, until a claim for refund or credit has been duly
filed with the Commissioner; but such suit or proceeding may be maintained,
whether or not such tax, penalty, or sum has been paid under protest or duress.

In any case, no such suit or proceeding shall be filed after the expiration of two
(2) years from the date of payment of the tax or penalty regardless of any
supervening cause that may arise after payment: Provided, however, That the
Commissioner may, even without a claim therefor, refund or credit any tax,
where on the face of the return upon which payment was made, such
payment appears clearly to have been erroneously paid. (emphasis supplied)

The CTA found that respondent filed its final adjusted return on April 14, 1998.
Thus, its right to claim a refund or credit commenced on that date.13

The tax court applied Article 13 of the Civil Code which states:

Art. 13. When the law speaks of years, months, days or nights, it shall be
understood that years are of three hundred sixty-five days each; months, of
thirty days; days, of twenty-four hours, and nights from sunset to sunrise.

If the months are designated by their name, they shall be computed by the
number of days which they respectively have.

In computing a period, the first day shall be excluded, and the last included.
(emphasis supplied)

Thus, according to the CTA, the two-year prescriptive period under Section 229
of the NIRC for the filing of judicial claims was equivalent to 730 days. Because
the year 2000 was a leap year, respondent's petition, which was filed 731
days14 after respondent filed its final adjusted return, was filed beyond the
reglementary period.15

Respondent moved for reconsideration but it was denied.16 Hence, it filed an


appeal in the CA.17

On August 1, 2003, the CA reversed and set aside the decision of the CTA.18 It
ruled that Article 13 of the Civil Code did not distinguish between a regular year
and a leap year. According to the CA:

The rule that a year has 365 days applies, notwithstanding the fact that a
particular year is a leap year.19

In other words, even if the year 2000 was a leap year, the periods covered by
April 15, 1998 to April 14, 1999 and April 15, 1999 to April 14, 2000 should still be
counted as 365 days each or a total of 730 days. A statute which is clear and
explicit shall be neither interpreted nor construed.20
42

Petitioners moved for reconsideration but it was denied.21 Thus, this appeal.

Petitioners contend that tax refunds, being in the nature of an exemption,


should be strictly construed against claimants.22 Section 229 of the NIRC should
be strictly applied against respondent inasmuch as it has been consistently held
that the prescriptive period (for the filing of tax refunds and tax credits) begins
to run on the day claimants file their final adjusted returns.23 Hence, the claim
should have been filed on or before April 13, 2000 or within 730 days, reckoned
from the time respondent filed its final adjusted return.

The conclusion of the CA that respondent filed its petition for review in the CTA
within the two-year prescriptive period provided in Section 229 of the NIRC is
correct. Its basis, however, is not.

The rule is that the two-year prescriptive period is reckoned from the filing of the
final adjusted return.24 But how should the two-year prescriptive period be
computed?

As already quoted, Article 13 of the Civil Code provides that when the law
speaks of a year, it is understood to be equivalent to 365 days. In National
Marketing Corporation v. Tecson,25 we ruled that a year is equivalent to 365
days regardless of whether it is a regular year or a leap year.26

However, in 1987, EO27 292 or the Administrative Code of 1987 was enacted.
Section 31, Chapter VIII, Book I thereof provides:

Sec. 31. Legal Periods. — "Year" shall be understood to be twelve calendar


months; "month" of thirty days, unless it refers to a specific calendar month in
which case it shall be computed according to the number of days the specific
month contains; "day", to a day of twenty-four hours and; "night" from sunrise to
sunset. (emphasis supplied)

A calendar month is "a month designated in the calendar without regard to


the number of days it may contain."28 It is the "period of time running from the
beginning of a certain numbered day up to, but not including, the
corresponding numbered day of the next month, and if there is not a sufficient
number of days in the next month, then up to and including the last day of that
month."29 To illustrate, one calendar month from December 31, 2007 will be
from January 1, 2008 to January 31, 2008; one calendar month from January 31,
2008 will be from February 1, 2008 until February 29, 2008.30

A law may be repealed expressly (by a categorical declaration that the law is
revoked and abrogated by another) or impliedly (when the provisions of a
more recent law cannot be reasonably reconciled with the previous
one).31 Section 27, Book VII (Final Provisions) of the Administrative Code of 1987
states:

Sec. 27. Repealing clause. — All laws, decrees, orders, rules and regulation, or
portions thereof, inconsistent with this Code are hereby repealed or modified
accordingly.
43

A repealing clause like Sec. 27 above is not an express repealing clause


because it fails to identify or designate the laws to be abolished.32 Thus, the
provision above only impliedly repealed all laws inconsistent with the
Administrative Code of 1987.1avvphi1

Implied repeals, however, are not favored. An implied repeal must have been
clearly and unmistakably intended by the legislature. The test is whether the
subsequent law encompasses entirely the subject matter of the former law and
they cannot be logically or reasonably reconciled.33

Both Article 13 of the Civil Code and Section 31, Chapter VIII, Book I of the
Administrative Code of 1987 deal with the same subject matter — the
computation of legal periods. Under the Civil Code, a year is equivalent to 365
days whether it be a regular year or a leap year. Under the Administrative
Code of 1987, however, a year is composed of 12 calendar months. Needless
to state, under the Administrative Code of 1987, the number of days is
irrelevant.

There obviously exists a manifest incompatibility in the manner of computing


legal periods under the Civil Code and the Administrative Code of 1987. For this
reason, we hold that Section 31, Chapter VIII, Book I of the Administrative Code
of 1987, being the more recent law, governs the computation of legal
periods. Lex posteriori derogat priori.

Applying Section 31, Chapter VIII, Book I of the Administrative Code of 1987 to
this case, the two-year prescriptive period (reckoned from the time respondent
filed its final adjusted return34 on April 14, 1998) consisted of 24 calendar
months, computed as follows:

Year 1st calendar April 15, to May 14,


1 month 1998 1998
2nd calendar May 15, to June 14,
month 1998 1998
3rd calendar June 15, to July 14, 1998
month 1998
4th calendar July 15, 1998 to August 14,
month 1998
5th calendar August 15, to September
month 1998 14, 1998
6th calendar September to October 14,
month 15, 1998 1998
7th calendar October 15, to November
month 1998 14, 1998
8th calendar November to December
month 15, 1998 14, 1998
9th calendar December to January 14,
44

month 15, 1998 1999


10th calendar January 15, to February 14,
month 1999 1999
11th calendar February 15, to March 14,
month 1999 1999
12th calendar March 15, to April 14,
month 1999 1999
Year 13th calendar April 15, to May 14,
2 month 1999 1999
14th calendar May 15, to June 14,
month 1999 1999
15th calendar June 15, to July 14, 1999
month 1999
16th calendar July 15, 1999 to August 14,
month 1999
17th calendar August 15, to September
month 1999 14, 1999
18th calendar September to October 14,
month 15, 1999 1999
19th calendar October 15, to November
month 1999 14, 1999
20th calendar November to December
month 15, 1999 14, 1999
21st calendar December to January 14,
month 15, 1999 2000
22nd calendar January 15, to February 14,
month 2000 2000
23rd calendar February 15, to March 14,
month 2000 2000
24th calendar March 15, to April 14,
month 2000 2000
We therefore hold that respondent's petition (filed on April 14, 2000) was filed
on the last day of the 24th calendar month from the day respondent filed its
final adjusted return. Hence, it was filed within the reglementary period.

Accordingly, the petition is hereby DENIED. The case is REMANDED to the Court
of Tax Appeals which is ordered to expeditiously proceed to hear C.T.A. Case
No. 6113 entitled Primetown Property Group, Inc. v. Commissioner of Internal
Revenue and Arturo V. Parcero.

No costs.
45

SO ORDERED

[G.R. No. L-29131. August 27, 1969.]

NATIONAL MARKETING CORPORATION, plaintiff- appellant, v. MIGUEL D.


TECSON, ET AL., Defendants, MIGUEL D. TECSON, defendant-appellee, THE
INSURANCE COMMISSIONER, Petitioner.

Government Corporate Counsel Leopoldo M. Abellera and Trial Atty. Antonio


M. Brillantes, for Plaintiff-Appellant.

Antonio T. Lacdan, for Defendant-Appellee.

The Solicitor General for Petitioner.

SYLLABUS

1. CIVIL LAW; PRESCRIPTION OF ACTIONS; ACTION FOR REVIVAL OF JUDGMENT;


PERIOD THEREFOR. — Pursuant to Article 1144-(3) of our Civil Code, an action
upon a judgment "must be brought within 10 years from the time the right of
action accrues," which, in the language of Art. 1152 of the same Code
"commences from the time judgment sought to be revived has become
final."cralaw virtua1aw library

2. ID.; ID.; ID.; ID.; PRESCRIBED IN INSTANT CASE. — An action for revival of
judgment which become final on December 21, 1955, was filed on December
21, 1965. The lower court dismissed the action on the ground of prescription, it
having found that the aggregate of 10 years or 3,650 days from December 21,
1955 expired on December 19, 1965, there being two leap years with the month
of February of 29 days. HELD: The order of dismissal should be affirmed. Art. 13
of the Civil Code of the Philippines limits the computation of each "year" to 365
days.

3. ID.; ID.; INTERPRETATION OF MONTHS; JURISPRUDENCE. — Prior to the approval


of the Civil Code of Spain, the Supreme Court thereof had held on March 30,
1887, that, when the law spoke of months, it meant a "natural" month or "solar"
month, in the absence of express provision to the contrary. Such provision was
incorporated into the Civil Code of Spain, subsequently promulgated. Hence,
the same Supreme Court declared that, pursuant to Art. 7 of said Code,
"whenever months . . . are referred to in the law, it shall be understood that the
months are of 30 days," not the "natural," "solar" or "calendar" months, unless
they are "designated by name," in which case "they shall be computed by the
actual number of days they have." This concept was later, modified in the
Philippines, by Section 13 of the Revised Administrative Code, pursuant to
which, "month shall be understood to refer to a calendar month." In the
language of this Court, in People v. Del Rosario "with the approval of the Civil
Code of the Philippines (Republic Act 386) . . . we have reverted to the
46

provisions of the Spanish Civil Code in accordance with which a month is to be


considered as the regular 30-day month and not the solar or civil month," with
the particularity that, whereas the Spanish Code merely mentioned "months,
days or nights," ours has added thereto the term "years" and explicitly ordains
that "it shall be understood that years are of three hundred sixty-five
days."cralaw virtua1aw library

4. SUPREME COURT; NO POWER OF LEGISLATION BY JUDICIAL DECREE. — Where,


by upholding the theory of appellant, Article 13 of the Civil Code of the
Philippines is ignored and Section 13 of the Revised Administrative Code is
revived, the Court by such an interpretation would be engaging in judicial
legislation, and in effect, repealing an act of Congress. If public interest
demands a reversion to the policy embodied in the Revised Administrative
Code, this may be done through legislative process, not by judicial decree.

DECISION

CONCEPCION, J.:

This appeal has been certified to us by the Court of Appeals, only one question
of law being involved therein.

On November 14, 1955, the Court of First Instance of Manila rendered


judgment, in Civil Case No. 20520 thereof, entitled "Price Stabilization
Corporation v. Miguel D. Tecson and Alto Surety and Insurance Co., Inc.," the
dispositive part of which reads as follows.

"For the foregoing consideration, the Court decides this


ease:jgc:chanrobles.com.ph

"(a) Ordering the defendants Miguel D. Tecson, and Alto Surety & Insurance Co.
Inc. to pay jointly and severally plaintiff PRATRA the sum of P7,200.00 plus 7%
interest from May 25, 1960 until the amount is fully paid, plus P500.00 for
attorney’s fees, and plus costs;

"(b) Ordering defendant Miguel D. Tecson to indemnify his co-defendant Alto


Surety & Insurance Co., Inc. on the cross-claim for all the amounts it would be
made to pay in this decision, in case defendant Alto Surety & Insurance Co.,
Inc. pay the amount adjudged to plaintiff in this decision. From the date of
such payment defendant Miguel D. Tecson would pay the Alto Surety &
Insurance Co., Inc., interest at 12% per annum until Miguel D. Tecson has fully
reimbursed plaintiff of the said amount."cralaw virtua1aw library

Copy of this decision was, on November 21, 1955, served upon the defendants
in said case. On December 21, 1965, the National Marketing Corporation, as
successor to all the properties, assets, rights and chooses in action of the Price
Stabilization Corporation, as plaintiff in that case and judgment creditor therein,
47

filed, with the same court, a complaint, docketed as Civil Case No. 63701
thereof, against the same defendants, for the revival of the judgment rendered
in said Case No. 20520. Defendant Miguel D. Tecson moved to dismiss said
complaint, upon the ground of lack of jurisdiction over the subject-matter
thereof and prescription of action. Acting upon the motion and plaintiff’s
opposition thereto, said Court issued, on February 14, 1966, an order
reading:jgc:chanrobles.com.ph

"Defendant Miguel Tecson seeks the dismissal of the complaint on the ground
of lack of jurisdiction and prescription. As for lack of jurisdiction, as the amount
involved is less than P10,000 as actually these proceedings are a revival of a
decision issued by this same court, the matter of jurisdiction must be admitted.
But as for prescription. Plaintiffs admit the decision of this Court became final on
December 21,1955. This case was filed exactly on December 21, 1965 — but
more than ten years have passed a year is a period of 365 days (Art. 13, CCP).
Plaintiff forgot that 1960, 1964 were both leap years so that when this present
case was filed it was filed two days too late.

"The complaint insofar as Miguel Tecson is concerned is, therefore, dismissed as


having prescribed."cralaw virtua1aw library

The National Marketing Corporation appealed from such order to the Court of
Appeals, which, on March 20, 1969, certified the case to this Court, upon the
ground that the only question therein raised is one of law, namely, whether or
not the present action for the revival of a judgment is barred by the statute of
limitations.

Pursuant to Art. 1144-(3) of our Civil Code, an action upon a judgment "must be
brought within ten years-from the time the right of action accrues," which, in
the language of Art. 1152 of the same Code, "commences from the time the
judgment sought to be revived has become final." This, in turn, took place on
December 21, 1955, or thirty (30) days from notice of the judgment — which
was received by the defendants herein on November 21, 1955 — no appeal
having been taken therefrom. 1 The issue is thus confined to the date on which
ten (10) years from December 21, 1955 expired.

Plaintiff-appellant alleges that it was December 21, 1965, but appellee Tecson
maintains otherwise, because "when the laws speak of years . . . it shall be
understood that years are of three hundred sixty-five days each" — according
to Art. 13 of our Civil Code - and, 1960 and 1964 being leap years, the month of
February in both had 29 days, so that ten (10) years of 365 days each, or an
aggregate of 3,650 days, from December 21, 1955, expired on December 19,
1965. The lower court accepted this view in its appealed order of dismissal.

Plaintiff-appellant insists that the same "is erroneous, because a year means a
calendar year (Statutory Construction, Interpretation of Laws, by Crowford, p.
383) and since what is being computed here is the number of years, a
calendar year should be used as the basis of computation. There is no question
that when it is not a leap year, December 21 to December 21 of the following
year is one year. If the extra day in a leap year is not a day of the year,
48

because it is the 366 day, then to what year does it belong? Certainly, it must
belong to the year where it falls and, therefore, that the 366 days constitute
one year." 2

The very conclusion thus reached by appellant shows that its theory
contravenes the explicit provision of Art. 13 of the Civil Code of the Philippines,
limiting the connotation of each "year" — as the term is used in our laws — to
365 days. Indeed, prior to the approval of the Civil Code of Spain, the Supreme
Court thereof had held, on March 30, 1887, that, when the law spoke of
months, it meant a "natural" month or "solar" month, in the absence of express
provision to the contrary. Such provision was incorporated into the Civil Code of
Spain, subsequently promulgated. Hence, the same Supreme Court declared 3
that, pursuant to Art. 7 of said Code," ‘whenever months . . . are referred to in
the law, it shall be understood that the months, are of 30 days", not the
"natural", "solar" or "calendar" months, unless they are "designated by name," in
which case "they shall be computed by the actual number of days they have."
This concept was, later, modified in the Philippines, by Section 13 of the Revised
Administrative Code, pursuant to which, "month shall be understood to refer to
a calendar month." 4 In the language of this Court, in People v. Del Rosario, 5
"with the approval of the Civil Code of the Philippines (Republic Act 386) . . . we
have reverted to the provisions of the Spanish Civil Code in accordance with
which a month is to be considered as the regular 30-day month . . . and not the
solar or civil month," with the particularity that, whereas the Spanish Code
merely mentioned "months, days or nights," ours has added thereto the term
"years" and explicitly ordains that "it shall be understood that years are of three
hundred sixty-five days."cralaw virtua1aw library

Although some members of the Court are inclined to think that this legislation is
not realistic, for failure to conform with ordinary experience or practice, the
theory of plaintiff-appellant herein cannot be upheld without ignoring, if not
nullifying, Art. 13 of our Civil Code, and reviving Section 13 of The Revised
Administrative Code, thereby engaging in judicial legislation, and, in effect,
repealing an act of Congress. If public interest demands a reversion to the
policy embodied in the Revised Administrative Code, this may be done
through legislative process, not by judicial decree
G.R. No. 159694 January 27, 2006

COMMISSIONER OF INTERNAL REVENUE, Petitioner,


vs.
AZUCENA T. REYES, Respondent.

x -- -- -- -- -- -- -- -- -- -- -- -- -- x

G.R. No. 163581 January 27, 2006

AZUCENA T. REYES, Petitioner,


vs.
COMMISSIONER OF INTERNAL REVENUE, Respondent.

DECISION
49

PANGANIBAN, CJ.:

Under the present provisions of the Tax Code and pursuant to elementary due
process, taxpayers must be informed in writing of the law and the facts upon
which a tax assessment is based; otherwise, the assessment is void. Being
invalid, the assessment cannot in turn be used as a basis for the perfection of a
tax compromise.

The Case

Before us are two consolidated1 Petitions for Review2 filed under Rule 45 of the
Rules of Court, assailing the August 8, 2003 Decision3 of the Court of Appeals
(CA) in CA-GR SP No. 71392. The dispositive portion of the assailed Decision
reads as follows:

"WHEREFORE, the petition is GRANTED. The assailed decision of the Court of Tax
Appeals is ANNULLED and SET ASIDE without prejudice to the action of the
National Evaluation Board on the proposed compromise settlement of the
Maria C. Tancinco estate’s tax liability."4

The Facts

The CA narrated the facts as follows:

"On July 8, 1993, Maria C. Tancinco (or ‘decedent’) died, leaving a 1,292
square-meter residential lot and an old house thereon (or ‘subject property’)
located at 4931 Pasay Road, Dasmariñas Village, Makati City.

"On the basis of a sworn information-for-reward filed on February 17, 1997 by a


certain Raymond Abad (or ‘Abad’), Revenue District Office No. 50 (South
Makati) conducted an investigation on the decedent’s estate (or ‘estate’).
Subsequently, it issued a Return Verification Order. But without the required
preliminary findings being submitted, it issued Letter of Authority No. 132963 for
the regular investigation of the estate tax case. Azucena T. Reyes (or ‘[Reyes]’),
one of the decedent’s heirs, received the Letter of Authority on March 14, 1997.

"On February 12, 1998, the Chief, Assessment Division, Bureau of Internal
Revenue (or ‘BIR’), issued a preliminary assessment notice against the estate in
the amount of P14,580,618.67. On May 10, 1998, the heirs of the decedent (or
‘heirs’) received a final estate tax assessment notice and a demand letter,
both dated April 22, 1998, for the amount of P14,912,205.47, inclusive of
surcharge and interest.

"On June 1, 1998, a certain Felix M. Sumbillo (or ‘Sumbillo’) protested the
assessment [o]n behalf of the heirs on the ground that the subject property had
already been sold by the decedent sometime in 1990.

"On November 12, 1998, the Commissioner of Internal Revenue (or ‘[CIR]’)
issued a preliminary collection letter to [Reyes], followed by a Final Notice
Before Seizure dated December 4, 1998.
50

"On January 5, 1999, a Warrant of Distraint and/or Levy was served upon the
estate, followed on February 11, 1999 by Notices of Levy on Real Property and
Tax Lien against it.

"On March 2, 1999, [Reyes] protested the notice of levy. However, on March 11,
1999, the heirs proposed a compromise settlement of P1,000,000.00.

"In a letter to [the CIR] dated January 27, 2000, [Reyes] proposed to pay 50% of
the basic tax due, citing the heirs’ inability to pay the tax assessment. On March
20, 2000, [the CIR] rejected [Reyes’s] offer, pointing out that since the estate tax
is a charge on the estate and not on the heirs, the latter’s financial incapacity
is immaterial as, in fact, the gross value of the estate amounting
to P32,420,360.00 is more than sufficient to settle the tax liability. Thus, [the CIR]
demanded payment of the amount of P18,034,382.13 on or before April 15,
2000[;] otherwise, the notice of sale of the subject property would be published.

"On April 11, 2000, [Reyes] again wrote to [the CIR], this time proposing to pay
100% of the basic tax due in the amount of P5,313,891.00. She reiterated the
proposal in a letter dated May 18, 2000.

"As the estate failed to pay its tax liability within the April 15, 2000 deadline, the
Chief, Collection Enforcement Division, BIR, notified [Reyes] on June 6, 2000 that
the subject property would be sold at public auction on August 8, 2000.

"On June 13, 2000, [Reyes] filed a protest with the BIR Appellate Division.
Assailing the scheduled auction sale, she asserted that x x x the assessment,
letter of demand[,] and the whole tax proceedings against the estate are void
ab initio. She offered to file the corresponding estate tax return and pay the
correct amount of tax without surcharge [or] interest.

"Without acting on [Reyes’s] protest and offer, [the CIR] instructed the
Collection Enforcement Division to proceed with the August 8, 2000 auction
sale. Consequently, on June 28, 2000, [Reyes] filed a [P]etition for [R]eview with
the Court of Tax Appeals (or ‘CTA’), docketed as CTA Case No. 6124.

"On July 17, 2000, [Reyes] filed a Motion for the Issuance of a Writ of Preliminary
Injunction or Status Quo Order, which was granted by the CTA on July 26, 2000.
Upon [Reyes’s] filing of a surety bond in the amount of P27,000,000.00, the CTA
issued a [R]esolution dated August 16, 2000 ordering [the CIR] to desist and
refrain from proceeding with the auction sale of the subject property or from
issuing a [W]arrant of [D]istraint or [G]arnishment of [B]ank [A]ccount[,] pending
determination of the case and/or unless a contrary order is issued.

"[The CIR] filed a [M]otion to [D]ismiss the petition on the grounds (i) that the
CTA no longer has jurisdiction over the case[,] because the assessment against
the estate is already final and executory; and (ii) that the petition was filed out
of time. In a [R]esolution dated November 23, 2000, the CTA denied [the CIR’s]
motion.

"During the pendency of the [P]etition for [R]eview with the CTA, however, the
BIR issued Revenue Regulation (or ‘RR’) No. 6-2000 and Revenue Memorandum
51

Order (or ‘RMO’) No. 42-2000 offering certain taxpayers with delinquent
accounts and disputed assessments an opportunity to compromise their tax
liability.

"On November 25, 2000, [Reyes] filed an application with the BIR for the
compromise settlement (or ‘compromise’) of the assessment against the estate
pursuant to Sec. 204(A) of the Tax Code, as implemented by RR No. 6-2000 and
RMO No. 42-2000.

"On December 26, 2000, [Reyes] filed an Ex-Parte Motion for Postponement of
the hearing before the CTA scheduled on January 9, 2001, citing her pending
application for compromise with the BIR. The motion was granted and the
hearing was reset to February 6, 2001.

"On January 29, 2001, [Reyes] moved for postponement of the hearing set on
February 6, 2001, this time on the ground that she had already paid the
compromise amount of P1,062,778.20 but was still awaiting approval of the
National Evaluation Board (or ‘NEB’). The CTA granted the motion and reset the
hearing to February 27, 2001.

"On February 19, 2001, [Reyes] filed a Motion to Declare Application for the
Settlement of Disputed Assessment as a Perfected Compromise. In said motion,
she alleged that [the CIR] had not yet signed the compromise[,] because of
procedural red tape requiring the initials of four Deputy Commissioners on
relevant documents before the compromise is signed by the [CIR]. [Reyes]
posited that the absence of the requisite initials and signature[s] on said
documents does not vitiate the perfected compromise.

"Commenting on the motion, [the CIR] countered that[,] without the approval
of the NEB, [Reyes’s] application for compromise with the BIR cannot be
considered a perfected or consummated compromise.

"On March 9, 2001, the CTA denied [Reyes’s] motion, prompting her to file a
Motion for Reconsideration Ad Cautelam. In a [R]esolution dated April 10, 2001,
the CTA denied the [M]otion for [R]econsideration with the suggestion that[,]
for an orderly presentation of her case and to prevent piecemeal resolutions of
different issues, [Reyes] should file a [S]upplemental [P]etition for [R]eview[,]
setting forth the new issue of whether there was already a perfected
compromise.

"On May 2, 2001, [Reyes] filed a Supplemental Petition for Review with the CTA,
followed on June 4, 2001 by its Amplificatory Arguments (for the Supplemental
Petition for Review), raising the following issues:

‘1. Whether or not an offer to compromise by the [CIR], with the acquiescence
by the Secretary of Finance, of a tax liability pending in court, that was
accepted and paid by the taxpayer, is a perfected and consummated
compromise.

‘2. Whether this compromise is covered by the provisions of Section 204 of the
Tax Code (CTRP) that requires approval by the BIR [NEB].’
52

"Answering the Supplemental Petition, [the CIR] averred that an application for
compromise of a tax liability under RR No. 6-2000 and RMO No. 42-2000 requires
the evaluation and approval of either the NEB or the Regional Evaluation Board
(or ‘REB’), as the case may be.

"On June 14, 2001, [Reyes] filed a Motion for Judgment on the Pleadings; the
motion was granted on July 11, 2001. After submission of memoranda, the case
was submitted for [D]ecision.

"On June 19, 2002, the CTA rendered a [D]ecision, the decretal portion of which
pertinently reads:

‘WHEREFORE, in view of all the foregoing, the instant [P]etition for [R]eview is
hereby DENIED. Accordingly, [Reyes] is hereby ORDERED to PAY deficiency
estate tax in the amount of Nineteen Million Five Hundred Twenty Four
Thousand Nine Hundred Nine and 78/100 (P19,524,909.78), computed as
follows:

xxxxxxxxx

‘[Reyes] is likewise ORDERED to PAY 20% delinquency interest on deficiency


estate tax due of P17,934,382.13 from January 11, 2001 until full payment
thereof pursuant to Section 249(c) of the Tax Code, as amended.’

"In arriving at its decision, the CTA ratiocinated that there can only be a
perfected and consummated compromise of the estate’s tax liability[,] if the
NEB has approved [Reyes’s] application for compromise in accordance with
RR No. 6-2000, as implemented by RMO No. 42-2000.

"Anent the validity of the assessment notice and letter of demand against the
estate, the CTA stated that ‘at the time the questioned assessment notice and
letter of demand were issued, the heirs knew very well the law and the facts on
which the same were based.’ It also observed that the petition was not filed
within the 30-day reglementary period provided under Sec. 11 of Rep. Act No.
1125 and Sec. 228 of the Tax Code."5

Ruling of the Court of Appeals

In partly granting the Petition, the CA said that Section 228 of the Tax Code and
RR 12-99 were mandatory and unequivocal in their requirement. The
assessment notice and the demand letter should have stated the facts and the
law on which they were based; otherwise, they were deemed void.6 The
appellate court held that while administrative agencies, like the BIR, were not
bound by procedural requirements, they were still required by law and equity
to observe substantive due process. The reason behind this requirement, said
the CA, was to ensure that taxpayers would be duly apprised of -- and could
effectively protest -- the basis of tax assessments against them.7 Since the
assessment and the demand were void, the proceedings emanating from
them were likewise void, and any order emanating from them could never
attain finality.
53

The appellate court added, however, that it was premature to declare as


perfected and consummated the compromise of the estate’s tax liability. It
explained that, where the basic tax assessed exceeded P1 million, or where the
settlement offer was less than the prescribed minimum rates, the National
Evaluation Board’s (NEB) prior evaluation and approval were the conditio sine
qua non to the perfection and consummation of any compromise.8 Besides,
the CA pointed out, Section 204(A) of the Tax Code applied to all
compromises, whether government-initiated or not.9 Where the law did not
distinguish, courts too should not distinguish.

Hence, this Petition.10

The Issues

In GR No. 159694, petitioner raises the following issues for the Court’s
consideration:

"I.

Whether petitioner’s assessment against the estate is valid.

"II.

Whether respondent can validly argue that she, as well as the other heirs, was
not aware of the facts and the law on which the assessment in question is
based, after she had opted to propose several compromises on the estate tax
due, and even prematurely acting on such proposal by paying 20% of the
basic estate tax due."11

The foregoing issues can be simplified as follows: first, whether the assessment
against the estate is valid; and, second, whether the compromise entered into
is also valid.

The Court’s Ruling

The Petition is unmeritorious.

First Issue:

Validity of the Assessment Against the Estate

The second paragraph of Section 228 of the Tax Code12 is clear and
mandatory. It provides as follows:

"Sec. 228. Protesting of Assessment. --

xxxxxxxxx

"The taxpayers shall be informed in writing of the law and the facts on which
the assessment is made: otherwise, the assessment shall be void."

In the present case, Reyes was not informed in writing of the law and the facts
on which the assessment of estate taxes had been made. She was merely
54

notified of the findings by the CIR, who had simply relied upon the provisions of
former Section 22913 prior to its amendment by Republic Act (RA) No. 8424,
otherwise known as the Tax Reform Act of 1997.

First, RA 8424 has already amended the provision of Section 229 on protesting
an assessment. The old requirement of merely notifying the taxpayer of the
CIR’s findings was changed in 1998 to informing the taxpayer of not only the
law, but also of the facts on which an assessment would be made; otherwise,
the assessment itself would be invalid.

It was on February 12, 1998, that a preliminary assessment notice was issued
against the estate. On April 22, 1998, the final estate tax assessment notice, as
well as demand letter, was also issued. During those dates, RA 8424 was
already in effect. The notice required under the old law was no longer sufficient
under the new law.

To be simply informed in writing of the investigation being conducted and of


the recommendation for the assessment of the estate taxes due is nothing but
a perfunctory discharge of the tax function of correctly assessing a taxpayer.
The act cannot be taken to mean that Reyes already knew the law and the
facts on which the assessment was based. It does not at all conform to the
compulsory requirement under Section 228. Moreover, the Letter of Authority
received by respondent on March 14, 1997 was for the sheer purpose of
investigation and was not even the requisite notice under the law.

The procedure for protesting an assessment under the Tax Code is found in
Chapter III of Title VIII, which deals with remedies. Being procedural in nature,
can its provision then be applied retroactively? The answer is yes.

The general rule is that statutes are prospective. However, statutes that are
remedial, or that do not create new or take away vested rights, do not fall
under the general rule against the retroactive operation of statutes.14 Clearly,
Section 228 provides for the procedure in case an assessment is protested. The
provision does not create new or take away vested rights. In both instances, it
can surely be applied retroactively. Moreover, RA 8424 does not state, either
expressly or by necessary implication, that pending actions are excepted from
the operation of Section 228, or that applying it to pending proceedings would
impair vested rights.

Second, the non-retroactive application of Revenue Regulation (RR) No. 12-99


is of no moment, considering that it merely implements the law.

A tax regulation is promulgated by the finance secretary to implement the


provisions of the Tax Code.15 While it is desirable for the government authority or
administrative agency to have one immediately issued after a law is passed,
the absence of the regulation does not automatically mean that the law itself
would become inoperative.

At the time the pre-assessment notice was issued to Reyes, RA 8424 already
stated that the taxpayer must be informed of both the law and facts on which
the assessment was based. Thus, the CIR should have required the assessment
55

officers of the Bureau of Internal Revenue (BIR) to follow the clear mandate of
the new law. The old regulation governing the issuance of estate tax
assessment notices ran afoul of the rule that tax regulations -- old as they were -
- should be in harmony with, and not supplant or modify, the law.16

It may be argued that the Tax Code provisions are not self-executory. It would
be too wide a stretch of the imagination, though, to still issue a regulation that
would simply require tax officials to inform the taxpayer, in any manner, of the
law and the facts on which an assessment was based. That requirement is
neither difficult to make nor its desired results hard to achieve.

Moreover, an administrative rule interpretive of a statute, and not declarative


of certain rights and corresponding obligations, is given retroactive effect as of
the date of the effectivity of the statute.17 RR 12-99 is one such rule. Being
interpretive of the provisions of the Tax Code, even if it was issued only on
September 6, 1999, this regulation was to retroact to January 1, 1998 -- a date
prior to the issuance of the preliminary assessment notice and demand letter.

Third, neither Section 229 nor RR 12-85 can prevail over Section 228 of the Tax
Code.

No doubt, Section 228 has replaced Section 229. The provision on protesting an
assessment has been amended. Furthermore, in case of discrepancy between
the law as amended and its implementing but old regulation, the former
necessarily prevails.18 Thus, between Section 228 of the Tax Code and the
pertinent provisions of RR 12-85, the latter cannot stand because it cannot go
beyond the provision of the law. The law must still be followed, even though the
existing tax regulation at that time provided for a different procedure. The
regulation then simply provided that notice be sent to the respondent in the
form prescribed, and that no consequence would ensue for failure to comply
with that form.

Fourth, petitioner violated the cardinal rule in administrative law that the
taxpayer be accorded due process. Not only was the law here disregarded,
but no valid notice was sent, either. A void assessment bears no valid fruit.

The law imposes a substantive, not merely a formal, requirement. To proceed


heedlessly with tax collection without first establishing a valid assessment is
evidently violative of the cardinal principle in administrative investigations: that
taxpayers should be able to present their case and adduce supporting
evidence.19 In the instant case, respondent has not been informed of the basis
of the estate tax liability. Without complying with the unequivocal mandate of
first informing the taxpayer of the government’s claim, there can be no
deprivation of property, because no effective protest can be made.20 The
haphazard shot at slapping an assessment, supposedly based on estate
taxation’s general provisions that are expected to be known by the taxpayer, is
utter chicanery.

Even a cursory review of the preliminary assessment notice, as well as the


demand letter sent, reveals the lack of basis for -- not to mention the
insufficiency of -- the gross figures and details of the itemized deductions
56

indicated in the notice and the letter. This Court cannot countenance an
assessment based on estimates that appear to have been arbitrarily or
capriciously arrived at. Although taxes are the lifeblood of the government,
their assessment and collection "should be made in accordance with law as
any arbitrariness will negate the very reason for government itself."21

Fifth, the rule against estoppel does not apply. Although the government
cannot be estopped by the negligence or omission of its agents, the obligatory
provision on protesting a tax assessment cannot be rendered nugatory by a
mere act of the CIR .

Tax laws are civil in nature.22 Under our Civil Code, acts executed against the
mandatory provisions of law are void, except when the law itself authorizes the
validity of those acts.23 Failure to comply with Section 228 does not only render
the assessment void, but also finds no validation in any provision in the Tax
Code. We cannot condone errant or enterprising tax officials, as they are
expected to be vigilant and law-abiding.

Second Issue:

Validity of Compromise

It would be premature for this Court to declare that the compromise on the
estate tax liability has been perfected and consummated, considering the
earlier determination that the assessment against the estate was void. Nothing
has been settled or finalized. Under Section 204(A) of the Tax Code, where the
basic tax involved exceeds one million pesos or the settlement offered is less
than the prescribed minimum rates, the compromise shall be subject to the
approval of the NEB composed of the petitioner and four deputy
commissioners.

Finally, as correctly held by the appellate court, this provision applies to all
compromises, whether government-initiated or not. Ubi lex non distinguit, nec
nos distinguere debemos. Where the law does not distinguish, we should not
distinguish.

WHEREFORE, the Petition is hereby DENIED and the assailed Decision AFFIRMED.
No pronouncement as to costs.

SO ORDERED.

G.R. NO. 159647 April 15, 2005]

COMMISSIONER OF INTERNAL REVENUE, Petitioners, v. CENTRAL LUZON DRUG


CORPORATION, Respondent.

DECISION

PANGANIBAN, J.:

The 20 percent discount required by the law to be given to senior citizens is


a tax credit, not merely a tax deduction from the gross income or gross sale of
57

the establishment concerned. A tax credit is used by a private establishment


only after the tax has been computed; a tax deduction, before the tax is
computed. RA 7432 unconditionally grants a tax credit to all covered entities.
Thus, the provisions of the revenue regulation that withdraw or modify such
grant are void. Basic is the rule that administrative regulations cannot amend or
revoke the law.

The Case

Before us is a Petition for Review1 under Rule 45 of the Rules of Court, seeking to
set aside the August 29, 2002 Decision2 and the August 11, 2003 Resolution3 of
the Court of Appeals (CA) in CA-GR SP No. 67439. The assailed Decision reads
as follows:

"WHEREFORE, premises considered, the Resolution appealed from is AFFIRMED in


toto. No costs."4

The assailed Resolution denied petitioner's Motion for Reconsideration.

The Facts

The CA narrated the antecedent facts as follows:

"Respondent is a domestic corporation primarily engaged in retailing of


medicines and other pharmaceutical products. In 1996, it operated six (6)
drugstores under the business name and style 'Mercury Drug.'

"From January to December 1996, respondent granted twenty (20%) percent


sales discount to qualified senior citizens on their purchases of medicines
pursuant to Republic Act No. [R.A.] 7432 and its Implementing Rules and
Regulations. For the said period, the amount allegedly representing the 20%
sales discount granted by respondent to qualified senior citizens
totaled P904,769.00.

"On April 15, 1997, respondent filed its Annual Income Tax Return for taxable
year 1996 declaring therein that it incurred net losses from its operations.

"On January 16, 1998, respondent filed with petitioner a claim for tax
refund/credit in the amount of P904,769.00 allegedly arising from the 20% sales
discount granted by respondent to qualified senior citizens in compliance with
[R.A.] 7432. Unable to obtain affirmative response from petitioner, respondent
elevated its claim to the Court of Tax Appeals [(CTA or Tax Court)] via a Petition
for Review.

"On February 12, 2001, the Tax Court rendered a Decision5 dismissing
respondent's Petition for lack of merit. In said decision, the [CTA] justified its
ruling with the following ratiocination:

'x x x, if no tax has been paid to the government, erroneously or illegally, or if no


amount is due and collectible from the taxpayer, tax refund or tax credit is
unavailing. Moreover, whether the recovery of the tax is made by means of a
claim for refund or tax credit, before recovery is allowed[,] it must be first
58

established that there was an actual collection and receipt by the government
of the tax sought to be recovered. x x x.

'x x x x x x x x x

'Prescinding from the above, it could logically be deduced that tax credit is
premised on the existence of tax liability on the part of taxpayer. In other words,
if there is no tax liability, tax credit is not available.'

"Respondent lodged a Motion for Reconsideration. The [CTA], in its assailed


resolution,6 granted respondent's motion for reconsideration and ordered
herein petitioner to issue a Tax Credit Certificate in favor of respondent citing
the decision of the then Special Fourth Division of [the CA] in CA G.R. SP No.
60057 entitled 'Central [Luzon] Drug Corporation v. Commissioner of Internal
Revenue' promulgated on May 31, 2001, to wit:

'However, Sec. 229 clearly does not apply in the instant case because the tax
sought to be refunded or credited by petitioner was not erroneously paid or
illegally collected. We take exception to the CTA's sweeping but unfounded
statement that 'both tax refund and tax credit are modes of recovering taxes
which are either erroneously or illegally paid to the government. 'Tax refunds or
credits do not exclusively pertain to illegally collected or erroneously paid taxes
as they may be other circumstances where a refund is warranted. The tax
refund provided under Section 229 deals exclusively with illegally collected or
erroneously paid taxes but there are other possible situations, such as the
refund of excess estimated corporate quarterly income tax paid, or that of
excess input tax paid by a VAT-registered person, or that of excise tax paid on
goods locally produced or manufactured but actually exported. The standards
and mechanics for the grant of a refund or credit under these situations are
different from that under Sec. 229. Sec. 4[.a)] of R.A. 7432, is yet another
instance of a tax credit and it does not in any way refer to illegally collected or
erroneously paid taxes, x x x. '"7

Ruling of the Court of Appeals

The CA affirmed in toto the Resolution of the Court of Tax Appeals (CTA)
ordering petitioner to issue a tax credit certificate in favor of respondent in the
reduced amount of P903,038.39. It reasoned that Republic Act No. (RA) 7432
required neither a tax liability nor a payment of taxes by private establishments
prior to the availment of a tax credit. Moreover, such credit is not tantamount
to an unintended benefit from the law, but rather a just compensation for the
taking of private property for public use.

Hence this Petition.8

The Issues

Petitioner raises the following issues for our consideration:

"Whether the Court of Appeals erred in holding that respondent may claim the
20% sales discount as a tax credit instead of as a deduction from gross income
or gross sales.
59

"Whether the Court of Appeals erred in holding that respondent is entitled to a


refund."9

These two issues may be summed up in only one: whether respondent, despite
incurring a net loss, may still claim the 20 percent sales discount as a tax credit.

The Court's Ruling

The Petition is not meritorious.

Sole Issue:

Claim of 20 Percent Sales Discount as Tax Credit Despite Net Loss

Section 4a) of RA 743210 grants to senior citizens the privilege of obtaining a 20


percent discount on their purchase of medicine from any private establishment
in the country.11 The latter may then claim the cost of the discount as a tax
credit.12 But can such credit be claimed, even though an establishment
operates at a loss?chanroblesvirtualawlibrary

We answer in the affirmative.

Tax Credit versus Tax Deduction

Although the term is not specifically defined in our Tax Code,13 tax
credit generally refers to an amount that is "subtracted directly from one's total
tax liability."14 It is an "allowance against the tax itself"15 or "a deduction from
what is owed"16 by a taxpayer to the government. Examples of tax credits are
withheld taxes, payments of estimated tax, and investment tax credits.17

Tax credit should be understood in relation to other tax concepts. One of these
is tax deduction - - defined as a subtraction "from income for tax purposes,"18 or
an amount that is "allowed by law to reduce income prior to [the] application
of the tax rate to compute the amount of tax which is due."19 An example of
a tax deduction is any of the allowable deductions enumerated in Section
3420 of the Tax Code.

A tax credit differs from a tax deduction. On the one hand, a tax
credit reduces the tax due, including - - whenever applicable - - the income
tax that is determined after applying the corresponding tax rates to taxable
income.21 A tax deduction, on the other, reduces the income that is subject to
tax22 in order to arrive at taxable income.23 To think of the former as the latter is
to avoid, if not entirely confuse, the issue. A tax credit is used only after the tax
has been computed; a tax deduction, before.

Tax Liability Required for Tax Credit

Since a tax credit is used to reduce directly the tax that is due, there ought to
be a tax liability before the tax credit can be applied. Without that liability,
any tax credit application will be useless. There will be no reason for deducting
the latter when there is, to begin with, no existing obligation to the government.
However, as will be presented shortly, the existence of a tax credit or
60

its grant by law is not the same as the availment or use of such credit. While the
grant is mandatory, the availment or use is not.

If a net loss is reported by, and no other taxes are currently due from, a business
establishment, there will obviously be no tax liability against which any tax
credit can be applied.24 For the establishment to choose the immediate
availment of a tax credit will be premature and impracticable. Nevertheless,
the irrefutable fact remains that, under RA 7432, Congress has granted without
conditions a tax credit benefit to all covered establishments.

Although this tax credit benefit is available, it need not be used by losing
ventures, since there is no tax liability that calls for its application. Neither can it
be reduced to nil by the quick yet callow stroke of an administrative pen,
simply because no reduction of taxes can instantly be effected. By its nature,
the tax credit may still be deducted from a future, not a present, tax liability,
without which it does not have any use. In the meantime, it need not move. But
it breathes.

Prior Tax Payments Not Required for Tax Credit

While a tax liability is essential to the availment or use of any tax credit, prior tax
payments are not. On the contrary, for the existence or grant solely of such
credit, neither a tax liability nor a prior tax payment is needed. The Tax Code is
in fact replete with provisions granting or allowing tax credits, even though no
taxes have been previously paid.

For example, in computing the estate tax due, Section 86(E) allows a tax
credit - - subject to certain limitations - - for estate taxes paid to a foreign
country. Also found in Section 101(C) is a similar provision for donor's taxes - -
again when paid to a foreign country - - in computing for the donor's tax due.
The tax credits in both instances allude to the prior payment of taxes, even if
not made to our government.

Under Section 110, a VAT (Value-Added Tax) - registered person engaging in


transactions - - whether or not subject to the VAT - - is also allowed a tax
credit that includes a ratable portion of any input tax not directly attributable
to either activity. This input tax may either be the VAT on the purchase or
importation of goods or services that is merely due from - - not necessarily paid
by - - such VAT-registered person in the course of trade or business; or the
transitional input tax determined in accordance with Section 111(A). The latter
type may in fact be an amount equivalent to only eight percent of the value of
a VAT-registered person's beginning inventory of goods, materials and supplies,
when such amount - - as computed - - is higher than the actual VAT paid on
the said items.25 Clearly from this provision, the tax credit refers to an input tax
that is either due only or given a value by mere comparison with the VAT
actually paid - - then later prorated. No tax is actually paid prior to the
availment of such credit.

In Section 111(B), a one and a half percent input tax credit that is merely
presumptive is allowed. For the purchase of primary agricultural products used
as inputs - - either in the processing of sardines, mackerel and milk, or in the
61

manufacture of refined sugar and cooking oil - - and for the contract price of
public work contracts entered into with the government, again, no prior tax
payments are needed for the use of the tax credit.

More important, a VAT-registered person whose sales are zero-rated or


effectively zero-rated may, under Section 112(A), apply for the issuance of
a tax credit certificate for the amount of creditable input taxes merely due - -
again not necessarily paid to - - the government and attributable to such sales,
to the extent that the input taxes have not been applied against output
taxes.26 Where a taxpayer
is engaged in zero-rated or effectively zero-rated sales and also in taxable or
exempt sales, the amount of creditable input taxes due that are not directly
and entirely attributable to any one of these transactions shall be
proportionately allocated on the basis of the volume of sales. Indeed, in
availing of such tax credit for VAT purposes, this provision - - as well as the one
earlier mentioned - - shows that the prior payment of taxes is not a requisite.

It may be argued that Section 28(B)(5)(b) of the Tax Code is another illustration
of a tax credit allowed, even though no prior tax payments are not required.
Specifically, in this provision, the imposition of a final withholding tax rate on
cash and/or property dividends received by a nonresident foreign corporation
from a domestic corporation is subjected to the condition that a foreign tax
credit will be given by the domiciliary country in an amount equivalent to taxes
that are merely deemed paid.27 Although true, this provision actually refers to
the tax credit as a condition only for the imposition of a lower tax rate, not as
a deduction from the corresponding tax liability. Besides, it is not our
government but the domiciliary country that credits against the income tax
payable to the latter by the foreign corporation, the tax to be foregone or
spared.28

In contrast, Section 34(C)(3), in relation to Section 34(C)(7)(b), categorically


allows as credits, against the income tax imposable under Title II, the amount of
income taxes merely incurred - - not necessarily paid - - by a domestic
corporation during a taxable year in any foreign country. Moreover, Section
34(C)(5) provides that for such taxes incurred but not paid, a tax credit may be
allowed, subject to the condition precedent that the taxpayer shall simply give
a bond with sureties satisfactory to and approved by petitioner, in such sum as
may be required; and further conditioned upon payment by the taxpayer of
any tax found due, upon petitioner's redetermination of it.

In addition to the above-cited provisions in the Tax Code, there are also tax
treaties and special laws that grant or allow tax credits, even though no prior
tax payments have been made.

Under the treaties in which the tax credit method is used as a relief to avoid
double taxation, income that is taxed in the state of source is also taxable in
the state of residence, but the tax paid in the former is merely allowed as a
credit against the tax levied in the latter.29 Apparently, payment is made to
the state of source, not the state of residence. No tax, therefore, has
been previously paid to the latter.
62

Under special laws that particularly affect businesses, there can also be tax
credit incentives. To illustrate, the incentives provided for in Article 48 of
Presidential Decree No. (PD) 1789, as amended by Batas Pambansa Blg. (BP)
391, include tax credits equivalent to either five percent of the net value
earned, or five or ten percent of the net local content of exports.30 In order to
avail of such credits under the said law and still achieve its objectives, no prior
tax payments are necessary.

From all the foregoing instances, it is evident that prior tax payments are not
indispensable to the availment of a tax credit. Thus, the CA correctly held that
the availment under RA 7432 did not require prior tax payments by private
establishments concerned.31 However, we do not agree with its finding32 that
the carry-over of tax credits under the said special law to succeeding taxable
periods, and even their application against internal revenue taxes, did not
necessitate the existence of a tax liability.

The examples above show that a tax liability is certainly important in


the availment or use, not the existence or grant, of a tax credit. Regarding this
matter, a private establishment reporting a net loss in its financial statements is
no different from another that presents a net income. Both are entitled to
the tax credit provided for under RA 7432, since the law itself accords that
unconditional benefit. However, for the losing establishment to immediately
apply such credit, where no tax is due, will be an improvident usance.

Sections 2.i and 4 of Revenue Regulations No. 2-94 Erroneous

RA 7432 specifically allows private establishments to claim as tax credit the


amount of discounts they grant.33 In turn, the Implementing Rules and
Regulations, issued pursuant thereto, provide the procedures for its
availment.34 To deny such credit, despite the plain mandate of the law and the
regulations carrying out that mandate, is indefensible.

First, the definition given by petitioner is erroneous. It refers to tax credit as the
amount representing the 20 percent discount that "shall be deducted by the
said establishments from their gross income for income tax purposes and from
their gross sales for value-added tax or other percentage tax purposes."35 In
ordinary business language, the tax credit represents the amount of such
discount. However, the manner by which the discount shall be credited against
taxes has not been clarified by the revenue regulations.

By ordinary acceptation, a discount is an "abatement or reduction made from


the gross amount or value of anything."36 To be more precise, it is in business
parlance "a deduction or lowering of an amount of money;"37 or "a reduction
from the full amount or value of something, especially a price."38 In business
there are many kinds of discount, the most common of which is that affecting
the income statement39 or financial report upon which the income tax is based.

Business Discounts Deducted from Gross Sales

A cash discount, for example, is one granted by business establishments


to credit customers for their prompt payment.40 It is a "reduction in price offered
63

to the purchaser if payment is made within a shorter period of time than the
maximum time specified."41 Also referred to as a sales discount on the part of
the seller and a purchase discount on the part of the buyer, it may be
expressed in such
terms as "5/10, n/30."42

A quantity discount, however, is a "reduction in price allowed for purchases


made in large quantities, justified by savings in packaging, shipping, and
handling."43 It is also called a volume or bulk discount.44

A "percentage reduction from the list price x x x allowed by manufacturers to


wholesalers and by wholesalers to retailers"45 is known as a trade discount. No
entry for it need be made in the manual or computerized books of accounts,
since the purchase or sale is already valued at the net price actually charged
the buyer.46 The purpose for the discount is to encourage trading or increase
sales, and the prices at which the purchased goods may be resold are also
suggested.47 Even a chain discount - - a series of discounts from one list price - -
is recorded at net.48

Finally, akin to a trade discount is a functional discount. It is "a supplier's price


discount given to a purchaser based on the [latter's] role in the [former's]
distribution system."49 This role usually involves warehousing or advertising.

Based on this discussion, we find that the nature of a sales discount is peculiar.
Applying generally accepted accounting principles (GAAP) in the country, this
type of discount is reflected in the income statement50 as a line item deducted
- - along with returns, allowances, rebates and other similar expenses - -
from gross sales to arrive at net sales.51 This type of presentation is resorted to,
because the accounts receivable and sales figures that arise from sales
discounts, - - as well as from quantity, volume or bulk discounts - - are recorded
in the manual and computerized books of accounts and reflected in the
financial statements at the gross amounts of the invoices.52 This manner of
recording credit sales - - known as the gross method - - is most widely used,
because it is simple, more convenient to apply than the net method, and
produces no material errors over time.53

However, under the net method used in recording trade, chain or functional
discounts, only the net amounts of the invoices - - after the discounts have
been deducted - - are recorded in the books of accounts54 and reflected in
the financial statements. A separate line item cannot be shown,55 because the
transactions themselves involving both accounts receivable and sales have
already been entered into, net of the said discounts.

The term sales discounts is not expressly defined in the Tax Code, but one
provision adverts to amounts whose sum - - along with sales
returns, allowances and cost of goods sold56 - - is deducted from gross sales to
come up with the gross income, profit or margin57 derived from business.58 In
another provision therein, sales discounts that are granted and indicated in the
invoices at the time of sale - - and that do not depend upon the happening of
any future event - - may be excluded from the gross sales within the same
quarter they were given.59 While determinative only of the VAT, the latter
64

provision also appears as a suitable reference point for income tax purposes
already embraced in the former. After all, these two provisions affirm that sales
discounts are amounts that are always deductible from gross sales.

Reason for the Senior Citizen Discount:

The Law, Not Prompt Payment

A distinguishing feature of the implementing rules of RA 7432 is the private


establishment's outright deduction of the discount from the invoice price of the
medicine sold to the senior citizen.60 It is, therefore, expected that for each
retail sale made under this law, the discount period lasts no more than a day,
because such discount is given - - and the net amount thereof collected - -
immediately upon perfection of the sale.61 Although prompt payment is made
for an arm's-length transaction by the senior citizen, the real and compelling
reason for the private establishment giving the discount is that the law itself
makes it mandatory.

What RA 7432 grants the senior citizen is a mere discount privilege, not a sales
discount or any of the above discounts in particular. Prompt payment is not the
reason for (although a necessary consequence of) such grant. To be sure, the
privilege enjoyed by the senior citizen must be equivalent to the tax
credit benefit enjoyed by the private establishment granting the discount. Yet,
under the revenue regulations promulgated by our tax authorities, this benefit
has been erroneously likened and confined to a sales discount.

To a senior citizen, the monetary effect of the privilege may be the same as
that resulting from a sales discount. However, to a private establishment, the
effect is different from a simple reduction in price that results from such
discount. In other words, the tax credit benefit is not the same as a sales
discount. To repeat from our earlier discourse, this benefit cannot and should
not be treated as a tax deduction.

To stress, the effect of a sales discount on the income statement and income
tax return of an establishment covered by RA 7432 is different from that resulting
from the availment or use of its tax credit benefit. While the former is a
deduction before, the latter is a deduction after, the income tax is computed.
As mentioned earlier, a discount is not necessarily a sales discount, and a tax
credit for a simple discount privilege should not be automatically treated like
a sales discount. Ubi lex non distinguit, nec nos distinguere debemus. Where the
law does not distinguish, we ought not to distinguish.

Sections 2.i and 4 of Revenue Regulations No. (RR) 2-94 define tax credit as the
20 percent discount deductible from gross income for income tax purposes, or
from gross sales for VAT or other percentage tax purposes. In effect, the tax
credit benefit under RA 7432 is related to a sales discount. This contrived
definition is improper, considering that the latter has to be deducted from gross
sales in order to compute the gross income in the income statement and
cannot be deducted again, even for purposes of computing the income tax.
65

When the law says that the cost of the discount may be claimed as a tax
credit, it means that the amount - - when claimed - - shall be treated as a
reduction from any tax liability, plain and simple. The option to avail of the tax
credit benefit depends upon the existence of a tax liability, but to limit the
benefit to a sales discount - - which is not even identical to the discount
privilege that is granted by law - - does not define it at all and serves no useful
purpose. The definition must, therefore, be stricken down.

Laws Not Amended by Regulations

Second, the law cannot be amended by a mere regulation. In fact, a


regulation that "operates to create a rule out of harmony with
the statute is a mere nullity";62 it cannot prevail.

It is a cardinal rule that courts "will and should respect the contemporaneous
construction placed upon a statute by the executive officers whose duty it is to
enforce it x x x."63 In the scheme of judicial tax administration, the need for
certainty and predictability in the implementation of tax laws is crucial.64 Our
tax authorities fill in the details that "Congress may not have the opportunity or
competence to provide."65 The regulations these authorities issue are relied
upon by taxpayers, who are certain that these will be followed by the
courts.66 Courts, however, will not uphold these authorities' interpretations when
clearly absurd, erroneous or improper.

In the present case, the tax authorities have given the term tax credit in
Sections 2.i and 4 of RR 2-94 a meaning utterly in contrast to what RA 7432
provides. Their interpretation has muddled up the intent of Congress in granting
a mere discount privilege, not a sales discount. The administrative agency
issuing these regulations may not enlarge, alter or restrict the provisions of the
law it administers; it cannot engraft additional requirements not contemplated
by the legislature.67

In case of conflict, the law must prevail.68 A "regulation adopted pursuant to


law is law."69 Conversely, a regulation or any portion thereof not adopted
pursuant to law is no law and has neither the force nor the effect of law.70

Availment of Tax Credit Voluntary

Third, the word may in the text of the statute71 implies that the
availability of the tax credit benefit is neither unrestricted nor
mandatory.72 There is no absolute right conferred upon respondent, or any
similar taxpayer, to avail itself of the tax credit remedy whenever it chooses;
"neither does it impose a duty on the part of the government to sit back and
allow an important facet of tax collection to be at the sole control and
discretion of the taxpayer."73 For the tax authorities to compel respondent to
deduct the 20 percent discount from either its gross income or its gross sales74 is,
therefore, not only to make an imposition without basis in law, but also to
blatantly contravene the law itself.
66

What Section 4.a of RA 7432 means is that the tax credit benefit is merely
permissive, not imperative. Respondent is given two options - - either to claim or
not to claim the cost of the discounts as a tax credit. In fact, it may even ignore
the credit and simply consider the gesture as an act of beneficence, an
expression of its social conscience.

Granting that there is a tax liability and respondent claims such cost as a tax
credit, then the tax credit can easily be applied. If there is none, the credit
cannot be used and will just have to be carried over and
revalidated75 accordingly. If, however, the business continues to operate at a
loss and no other taxes are due, thus compelling it to close shop, the credit can
never be applied and will be lost altogether.

In other words, it is the existence or the lack of a tax liability that determines
whether the cost of the discounts can be used as a tax credit. RA 7432 does
not give respondent the unfettered right to avail itself of the credit whenever it
pleases. Neither does it allow our tax administrators to expand or contract the
legislative mandate. "The 'plain meaning rule' or verba legis in statutory
construction is thus applicable x x x. Where the words of a statute are clear,
plain and free from ambiguity, it must be given its literal meaning and applied
without attempted interpretation."76

Tax Credit Benefit Deemed Just Compensation

Fourth, Sections 2.i and 4 of RR 2-94 deny the exercise by the State of its power
of eminent domain. Be it stressed that the privilege enjoyed by senior citizens
does not come directly from the State, but rather from the private
establishments concerned. Accordingly, the tax credit benefit granted to these
establishments can be deemed as their just compensation for private property
taken by the State for public use.77

The concept of public use is no longer confined to the traditional notion of use
by the public, but held synonymous with public interest, public benefit, public
welfare, and public convenience.78 The discount privilege to which our senior
citizens are entitled is actually a benefit enjoyed by the general public to which
these citizens belong. The discounts given would have entered the coffers and
formed part of the gross sales of the private establishments concerned, were it
not for RA 7432. The permanent reduction in their total revenues is a forced
subsidy corresponding to the taking of private property for public use or
benefit.

As a result of the 20 percent discount imposed by RA 7432, respondent


becomes entitled to a just compensation. This term refers not only to the
issuance of a tax credit certificate indicating the correct amount of the
discounts given, but also to the promptness in its release. Equivalent to the
payment of property taken by the State, such issuance - - when not done
within a reasonable time from the grant of the discounts - - cannot be
considered as just compensation. In effect, respondent is made to suffer the
consequences of being immediately deprived of its revenues while awaiting
actual receipt, through the certificate, of the equivalent amount it needs to
cope with the reduction in its revenues.79
67

Besides, the taxation power can also be used as an implement for the exercise
of the power of eminent domain.80 Tax measures are but "enforced
contributions exacted on pain of penal sanctions"81 and "clearly imposed for
a public purpose."82 In recent years, the power to tax has indeed become a
most effective tool to realize social justice, public welfare, and the equitable
distribution of wealth.83

While it is a declared commitment under Section 1 of RA 7432, social justice


"cannot be invoked to trample on the rights of property owners who under our
Constitution and laws are also entitled to protection. The social justice
consecrated in our [C]onstitution [is] not intended to take away rights from a
person and give them to another who is not entitled thereto."84 For this reason,
a just compensation for income that is taken away from respondent becomes
necessary. It is in the tax credit that our legislators find support to realize social
justice, and no administrative body can alter that fact.

To put it differently, a private establishment that merely breaks even85 - - without


the discounts yet - - will surely start to incur losses because of such discounts.
The same effect is expected if its mark-up is less than 20 percent, and if all its
sales come from retail purchases by senior citizens. Aside from the observation
we have already raised earlier, it will also be grossly unfair to an establishment if
the discounts will be treated merely as deductions from either its gross
income or its gross sales. Operating at a loss through no fault of its own, it will
realize that the tax credit limitation under RR 2-94 is inutile, if not improper.
Worse, profit-generating businesses will be put in a better position if they avail
themselves of tax credits denied those that are losing, because no taxes are
due from the latter.

Grant of Tax Credit Intended by the Legislature

Fifth, RA 7432 itself seeks to adopt measures whereby senior citizens are assisted
by the community as a whole and to establish a program beneficial to
them.86 These objectives are consonant with the constitutional policy of making
"health x x x services available to all the people at affordable cost"87 and of
giving "priority for the needs of the x x x elderly."88 Sections 2.i and 4 of RR 2-94,
however, contradict these constitutional policies and statutory objectives.

Furthermore, Congress has allowed all private establishments a simple tax


credit, not a deduction. In fact, no cash outlay is required from the government
for the availment or use of such credit. The deliberations on February 5, 1992 of
the Bicameral Conference Committee Meeting on Social Justice, which
finalized RA 7432, disclose the true intent of our legislators to treat the sales
discounts as a tax credit, rather than as a deduction from gross income. We
quote from those deliberations as follows:

"THE CHAIRMAN (Rep. Unico). By the way, before that ano, about deductions
from taxable income. I think we incorporated there a provision na - on the
responsibility of the private hospitals and drugstores, hindi
ba?chanroblesvirtualawlibrary

SEN. ANGARA. Oo.


68

THE CHAIRMAN. (Rep. Unico), So, I think we have to put in also a provision here
about the deductions from taxable income of that private hospitals, di ba
ganon 'yan?chanroblesvirtualawlibrary

MS. ADVENTO. Kaya lang po sir, and mga discounts po nila affecting
government and public institutions, so, puwede na po nating hindi isama yung
mga less deductions ng taxable income.

THE CHAIRMAN. (Rep. Unico). Puwede na. Yung about the private hospitals.
Yung isiningit natin?chanroblesvirtualawlibrary

MS. ADVENTO. Singit na po ba yung 15% on credit. (inaudible/did not use the
microphone).

SEN. ANGARA. Hindi pa, hindi pa.

THE CHAIRMAN. (Rep. Unico) Ah, 'di pa ba naisama


natin?chanroblesvirtualawlibrary

SEN. ANGARA. Oo. You want to insert that?chanroblesvirtualawlibrary

THE CHAIRMAN (Rep. Unico). Yung ang proposal ni Senator Shahani, e.

SEN. ANGARA. In the case of private hospitals they got the grant of 15%
discount, provided that, the private hospitals can claim the expense as a tax
credit.

REP. AQUINO. Yah could be allowed as deductions in the perpetrations of


(inaudible) income.

SEN. ANGARA. I-tax credit na lang natin para walang cash-out


ano?chanroblesvirtualawlibrary

REP. AQUINO. Oo, tax credit. Tama, Okay. Hospitals ba o lahat ng


establishments na covered.

THE CHAIRMAN. (Rep. Unico). Sa kuwan lang yon, as private hospitals lang.

REP. AQUINO. Ano ba yung establishments na


covered?chanroblesvirtualawlibrary

SEN. ANGARA. Restaurant lodging houses, recreation centers.

REP. AQUINO. All establishments covered siguro?chanroblesvirtualawlibrary

SEN. ANGARA. From all establishments. Alisin na natin 'Yung kuwan kung ganon.
Can we go back to Section 4 ha?chanroblesvirtualawlibrary

REP. AQUINO. Oho.

SEN. ANGARA. Letter A. To capture that thought, we'll say the grant of 20%
discount from all establishments et cetera, et cetera, provided that said
69

establishments - provided that private establishments may claim the cost as a


tax credit. Ganon ba 'yon?chanroblesvirtualawlibrary

REP. AQUINO. Yah.

SEN. ANGARA. Dahil kung government, they don't need to claim it.

THE CHAIRMAN. (Rep. Unico). Tax credit.

SEN. ANGARA. As a tax credit [rather] than a kuwan - deduction, Okay.

REP. AQUINO Okay.

SEN. ANGARA. Sige Okay. Di subject to style na lang sa Letter A".89

Special Law Over General Law

Sixth and last, RA 7432 is a special law that should prevail over the Tax Code - -
a general law. "x x x [T]he rule is that on a specific matter the special law shall
prevail over the general law, which shall
be resorted to only to supply deficiencies in the former."90 In addition, "[w]here
there are two statutes, the earlier special and the later general - - the terms of
the general broad enough to include the matter provided for in the special - -
the fact that one is special and the other is general creates a presumption that
the special is to be considered as remaining an exception to the general,91 one
as a general law of the land, the other as the law of a particular case."92 "It is a
canon of statutory construction that a later statute, general in its terms and not
expressly repealing a prior special statute, will ordinarily not affect the special
provisions of such earlier statute."93

RA 7432 is an earlier law not expressly repealed by, and therefore remains an
exception to, the Tax Code - - a later law. When the former states that a tax
credit may be claimed, then the requirement of prior tax payments under
certain provisions of the latter, as discussed above, cannot be made to apply.
Neither can the instances of or references to a tax deduction under the Tax
Code94 be made to restrict RA 7432. No provision of any revenue regulation
can supplant or modify the acts of Congress.

WHEREFORE, the Petition is hereby DENIED. The assailed Decision and Resolution
of the Court of Appeals AFFIRMED. No pronouncement as to costs.

SO ORDERED

G.R. No. 148083 July 21, 2006

COMMISSIONER OF INTERNAL REVENUE, petitioner,


vs.
BICOLANDIA DRUG CORPORATION (Formerly known as ELMAS DRUG
CO.), respondent.

DECISION

VELASCO, JR., J.:


70

In cases of conflict between the law and the rules and regulations
implementing the law, the law shall always prevail. Should Revenue Regulations
deviate from the law they seek to implement, they will be struck down.

The Facts

In 1992, Republic Act No. 7432, otherwise known as "An Act to Maximize the
Contribution of Senior Citizens to Nation Building, Grant Benefits and Special
Privileges and For Other Purposes," granted senior citizens several privileges, one
of which was obtaining a 20 percent discount from all establishments relative to
the use of transportation services, hotels and similar lodging establishments,
restaurants and recreation centers and purchase of medicines anywhere in the
country.1 The law also provided that the private establishments giving the
discount to senior citizens may claim the cost as tax credit.2 In compliance with
the law, the Bureau of Internal Revenue issued Revenue Regulations No. 2-94,
which defined "tax credit" as follows:

Tax Credit – refers to the amount representing the 20% discount granted
to a qualified senior citizen by all establishments relative to their utilization
of transportation services, hotels and similar lodging establishments,
restaurants, halls, circuses, carnivals and other similar places of culture,
leisure and amusement, which discount shall be deducted by the said
establishments from their gross income for income tax purposes and from
their gross sales for value-added tax or other percentage tax purposes.3

In 1995, respondent Bicolandia Drug Corporation, a corporation engaged in


the business of retailing pharmaceutical products under the business style of
"Mercury Drug," granted the 20 percent sales discount to qualified senior
citizens purchasing their medicines in compliance with R.A. No.
7432.4 Respondent treated this discount as a deduction from its gross income in
compliance with Revenue Regulations No. 2-94, which implemented R.A. No.
7432.5 On April 15, 1996, respondent filed its 1995 Corporate Annual Income Tax
Return declaring a net loss position with nil income tax liability.6

On December 27, 1996, respondent filed a claim for tax refund or credit in the
amount of PhP 259,659.00 with the Appellate Division of the Bureau of Internal
Revenue—because its net losses for the year 1995 prevented it from benefiting
from the treatment of sales discounts as a deduction from gross sales during the
said taxable year.7 It alleged that the petitioner Commissioner of Internal
Revenue erred in treating the 20 percent sales discount given to senior citizens
as a deduction from its gross income for income tax purposes or other
percentage tax purposes rather than as a tax credit.8

On April 6, 1998, respondent appealed to the Court of Tax Appeals in order to


toll the running of two (2)-year prescriptive period to file a claim for refund
pursuant to Section 230 of the Tax Code then.9 Respondent argued that since
Section 4 of R.A. No. 7432 provided that discounts granted to senior citizens
may be claimed as tax credit, Section 2(i) of Revenue Regulations No. 2-94,
which referred to the tax credit as the amount representing the 20 percent
discount that "shall be deducted by the said establishments from their gross
income for income tax purposes and from their gross sales for value-added tax
71

or other percentage tax purposes,"10 is illegal, void and without effect for being
inconsistent with the statute it implements.

Petitioner maintained that Revenue Regulations No. 2-94 is valid since the law
tasked the Department of Finance, among other government offices, with the
issuance of the necessary rules and regulations to carry out the objectives of
the law.11

Ruling of the Court of Tax Appeals

The Court of Tax Appeals declared that the provisions of R.A. No. 7432 would
prevail over Section 2(i) of Revenue Regulations No. 2-94, whose definition of
"tax credit" deviated from the intendment of the law; and as a result, partially
granted the respondent's claim for a refund. After examining the evidence on
record, the Court of Tax Appeals reduced the claimed 20 percent sales
discount, thus reducing the refund to be given. It ruled that "Respondent is
hereby ORDERED to REFUND in favor of Petitioner the amount of P236,321.52,
representing overpaid income tax for the year 1995."12

Ruling of the Court of Appeals

On appeal, the Court of Appeals modified the decision of the Court of Tax
Appeals as the law provided for a tax credit, not a tax refund. The fallo of the
Decision states:

WHEREFORE, premises considered, the present appeal is hereby GRANTED


and the Decision of the Court of Tax Appeals in C.T.A. Case No. 5599 is
hereby MODIFIED in the sense that the award of tax refund is ANNULLED
and SET ASIDE. Instead, the petitioner is hereby ORDERED to issue a tax
credit certificate in favor of the respondent in the amount of P 236,321.52.

No pronouncement as to costs.13

The Issue

Petitioner now argues that the Court of Appeals erred in holding that the 20
percent sales discount granted to qualified senior citizens by the respondent
pursuant to R.A. No. 7432 may be claimed as a tax credit, instead of a
deduction from gross income or gross sales.14

The Court's Ruling

The petition is not meritorious.

Redefining "Tax Credit" as "Tax Deduction"

The problem stems from the issuance of Revenue Regulations No. 2-94, which
was supposed to implement R.A. No. 7432, and the radical departure it made
when it defined the "tax credit" that would be granted to establishments that
give 20 percent discount to senior citizens. Under Revenue Regulations No. 2-94,
the tax credit is "the amount representing the 20 percent discount granted to a
qualified senior citizen by all establishments relative to their utilization of
72

transportation services, hotels and similar lodging establishments, restaurants,


drugstores, recreation centers, theaters, cinema houses, concert halls, circuses,
carnivals and other similar places of culture, leisure and amusement, which
discount shall be deducted by the said establishments from their gross income
for income tax purposes and from their gross sales for value-added tax or other
percentage tax purposes."15 It equated "tax credit" with "tax deduction,"
contrary to the definition in Black's Law Dictionary, which defined tax credit as:

An amount subtracted from an individual's or entity's tax liability to arrive


at the total tax liability. A tax credit reduces the taxpayer's liability x x x,
compared to a deduction which reduces taxable income upon which
the tax liability is calculated. A credit differs from deduction to the extent
that the former is subtracted from the tax while the latter is subtracted
from income before the tax is computed.16

The interpretation of an administrative government agency, which is tasked to


implement the statute, is accorded great respect and ordinarily controls the
construction of the courts.17 Be that as it may, the definition laid down in the
questioned Revenue Regulations can still be subjected to scrutiny. Courts will
not hesitate to set aside an executive interpretation when it is clearly erroneous.
There is no need for interpretation when there is no ambiguity in the rule, or
when the language or words used are clear and plain or readily
understandable to an ordinary reader.18 The definition of the term "tax credit" is
plain and clear, and the attempt of Revenue Regulations No. 2-94 to define it
differently is the root of the conflict.

Tax Credit is not Tax Refund

Petitioner argues that the tax credit is in the nature of a tax refund and should
be treated as a return for tax payments erroneously or excessively assessed
against a taxpayer, in line with Section 204(c) of Republic Act No. 8424, or the
National Internal Revenue Code of 1997. Petitioner claims that there should first
be payment of the tax before the tax credit can be claimed. However, in the
National Internal Revenue Code, we see at least one instance where this is not
the case. Any VAT-registered person, whose sales are zero-rated or effectively
zero-rated may, within two (2) years after the close of the taxable quarter when
the sales were made, apply for the issuance of a tax credit certificate or refund
of creditable input tax due or paid attributable to such sales, except
transitional input tax, to the extent that such input tax has not been applied
against output tax.19 It speaks of a tax credit for tax due, so payment of the tax
has not yet been made in that particular example.

The Court of Appeals expressly recognized the differences between a "tax


credit" and a "tax refund," and stated that the same are not synonymous with
each other, which is why it modified the ruling of the Court of Tax Appeals.

Revenue Regulations No. 2-94 vs. R.A. No. 7432 and


R.A. No. 7432 vs. the National Internal Revenue Code

Petitioner contends that since R.A. No. 7432 used the word "may," the
availability of the tax credit to private establishments is only permissive and not
73

absolute or mandatory. From that starting point, petitioner further argues that
the definition of the term "tax credit" in Revenue Regulations No. 2-94 was
validly issued under the authority granted by the law to the Department of
Finance to formulate the needed guidelines. It further explained that Revenue
Regulations No. 2-94 can be harmonized with R.A No. 7432, such that the
definition of the term "tax credit" in Revenue Regulations No. 2-94 is controlling.
It claims that to do otherwise would result in Section 4(a) of R.A. No. 7432
impliedly repealing Section 204 (c) of the National Internal Revenue Code.

These arguments must also fail.

Revenue Regulations No. 2-94 is still subordinate to R.A. No. 7432, and in cases
of conflict, the implementing rule will not prevail over the law it seeks to
implement. While seemingly conflicting laws must be harmonized as far as
practicable, in this particular case, the conflict cannot be resolved in the
manner the petitioner wishes. There is a great divide separating the idea of "tax
credit" and "tax deduction," as seen in the definition in Black's Law Dictionary.

The claimed absurdity of Section 4(a) of R.A. No. 7432 impliedly repealing
Section 204(c) of the National Internal Revenue Code could only come about if
it is accepted that a tax credit is akin to a tax refund wherein payment of taxes
must be made in order for it to be claimed. But as shown in Section 112(a) of
the National Internal Revenue Code, it is not always necessary for payment to
be made for a tax credit to be available.

Looking into R.A. No. 7432

Finally, petitioner argues that should private establishments, which count


respondent in their number, be allowed to claim tax credits for discounts given
to senior citizens, they would be earning and not just be reimbursed for the
discounts given.

It cannot be denied that R.A. No. 7432 has a laudable goal. Moreover, it
cannot be argued that it was the intent of lawmakers for private establishments
to be the primary beneficiaries of the law. However, while the purpose of the
law to benefit senior citizens is praiseworthy, the concerns of the affected
private establishments were also considered by the lawmakers. As in other
cases wherein private property is taken by the State for public use, there must
be just compensation. In this particular case, it took the form of the tax credit
granted to private establishments, purposely chosen by the lawmakers. In the
similar case of Commissioner of Internal Revenue v. Central Luzon Drug
Corporation,20 scrutinizing the deliberations of the Bicameral Conference
Committee Meeting on Social Justice on February 5, 1992 which finalized R.A.
No. 7432, the discussions of the lawmakers clearly showed the intent that the
cost of the 20 percent discount may be claimed by the private establishments
as a tax credit. An excerpt from the deliberations is as follows:

SEN. ANGARA. In the case of private hospitals they got the grant of 15%
discount, provided that, the private hospitals can claim the expense as a
tax credit.
74

REP. AQUINO. Yah could be allowed as deductions in the preparation of


(inaudible) income.

SEN. ANGARA. I-tax credit na lang natin para walang cash-out?

REP. AQUINO. Oo, tax credit. Tama. Okay. Hospitals ba o lahat ng


establishments na covered.

THE CHAIRMAN. Sa kuwan lang yon, as private hospitals lang.

REP. AQUINO. Ano ba yung establishments na covered?

SEN. ANGARA. Restaurant, lodging houses, recreation centers.

REP. AQUINO. All establishments covered siguro?

SEN. ANGARA. From all establishments. Alisin na natin `yung kuwan kung
ganon. Can we go back to Section 4 ha?

REP. AQUINO. Oho.

SEN. ANGARA. Letter A. To capture that thought, we'll say the grant of 20%
discount from all establishments et cetera, et cetera, provided that said
establishments may claim the cost as a tax credit. Ganon ba `yon?

REP. AQUINO. Yah.

SEN. ANGARA. Dahil kung government, they don't need to claim it.

THE CHAIRMAN. Tax credit.

SEN. ANGARA. As a tax credit [rather] than a kuwan – deduction, Okay.21

It is clear that the lawmakers intended the grant of a tax credit to complying
private establishments like the respondent.

If the private establishments appear to benefit more from the tax credit than
originally intended, it is not for petitioner to say that they shouldn't. The tax
credit may actually have provided greater incentive for the private
establishments to comply with R.A. No. 7432, or quicker relief from the cut into
profits of these businesses.

Revenue Regulations No. 2-94 Null and Void

From the above discussion, it must be concluded that Revenue Regulations No.
2-94 is null and void for failing to conform to the law it sought to implement. In
case of discrepancy between the basic law and a rule or regulation issued to
implement said law, the basic law prevails because said rule or regulation
cannot go beyond the terms and provisions of the basic law.22

Revenue Regulations No. 2-94 being null and void, it must be ruled then that
under R.A. No. 7432, which was effective at the time, respondent is entitled to
75

its claim of a tax credit, and the ruling of the Court of Appeals must be
affirmed.

But even as this particular case is decided in this manner, it must be noted that
the concerns of the petitioner regarding tax credits granted to private
establishments giving discounts to senior citizens have been addressed. R.A.
No. 7432 has been amended by Republic Act No. 9257, the "Expanded Senior
Citizens Act of 2003." In this, the term "tax credit" is no longer used. The 20
percent discount granted by hotels and similar lodging establishments,
restaurants and recreation centers, and in the purchase of medicines in all
establishments for the exclusive use and enjoyment of senior citizens is treated
in the following manner:

The establishment may claim the discounts granted under (a), (f), (g) and
(h) as tax deduction based on the net cost of the goods sold or services
rendered: Provided, That the cost of the discount shall be allowed as
deduction from gross income for the same taxable year that the discount
is granted. Provided, further, that the total amount of the claimed tax
deduction net of value added tax if applicable, shall be included in their
gross sales receipts for tax purposes and shall be subject to proper
documentation and to the provisions of the National Internal Revenue
Code, as amended.23

This time around, there is no conflict between the law and the implementing
Revenue Regulations. Under Revenue Regulations No. 4-2006, "(o)nly the actual
amount of the discount granted or a sales discount not exceeding 20% of the
gross selling price can be deducted from the gross income, net of value added
tax, if applicable, for income tax purposes, and from gross sales or gross
receipts of the business enterprise concerned, for VAT or other percentage tax
purposes."24 Under the new law, there is no tax credit to speak of, only
deductions.

Petitioner can find some vindication in the amendment made to R.A. No. 7432
by R.A. No. 9257, which may be more in consonance with the principles of
taxation, but as it was R.A. No. 7432 in force at the time this case arose, this law
controls the result in this particular case, for which reason the petition must fail.

This case should remind all heads of executive agencies which are given the
power to promulgate rules and regulations, that they assume the roles of
lawmakers. It is well-settled that a regulation should not conflict with the law it
implements. Thus, those drafting the regulations should study well the laws their
rules will implement, even to the extent of reviewing the minutes of the
deliberations of Congress about its intent when it drafted the law. They may
also consult the Secretary of Justice or the Solicitor General for their opinions on
the drafted rules. Administrative rules, regulations and orders have the efficacy
and force of law so long as they do not contravene any statute or the
Constitution.25 It is then the duty of the agencies to ensure that their rules do not
deviate from or amend acts of Congress, for their regulations are always
subordinate to law.
76

WHEREFORE, the Petition is hereby DENIED. The assailed Decision of the Court of
Appeals is AFFIRMED. There is no pronouncement as to costs.

SO ORDERED.

G.R. No. 162852 December 16, 2004

PHILIPPINE JOURNALISTS, INC., petitioner,


vs.
COMMISSIONER OF INTERNAL REVENUE, respondent.

DECISION

YNARES-SANTIAGO, J.:

This is a petition for review filed by Philippine Journalists, Incorporated (PJI)


assailing the Decision1 of the Court of Appeals dated August 5, 2003,2 which
ordered petitioner to pay the assessed tax liability of P111,291,214.46 and the
Resolution3 dated March 31, 2004 which denied the Motion for Reconsideration.

The case arose from the Annual Income Tax Return filed by petitioner for the
calendar year ended December 31, 1994 which presented a net income of
P30,877,387.00 and the tax due of P10,807,086.00. After deducting tax credits
for the year, petitioner paid the amount of P10,247,384.00.

On August 10, 1995, Revenue District Office No. 33 of the Bureau of Internal
Revenue (BIR) issued Letter of Authority No. 871204 for Revenue Officer Federico
de Vera, Jr. and Group Supervisor Vivencio Gapasin to examine petitioner’s
books of account and other accounting records for internal revenue taxes for
the period January 1, 1994 to December 31, 1994.

From the examination, the petitioner was told that there were deficiency taxes,
inclusive of surcharges, interest and compromise penalty in the following
amounts:

Value Added P 229,527.90


Tax
Income Tax 125,002,892.95
Withholding 2,748,012.35
Tax
Total P
127,980,433.20
In a letter dated August 29, 1997, Revenue District Officer Jaime Concepcion
invited petitioner to send a representative to an informal conference on
77

September 15, 1997 for an opportunity to object and present documentary


evidence relative to the proposed assessment. On September 22, 1997,
petitioner’s Comptroller, Lorenza Tolentino, executed a "Waiver of the Statute
of Limitation Under the National Internal Revenue Code (NIRC)".5 The document
"waive[d] the running of the prescriptive period provided by Sections 223 and
224 and other relevant provisions of the NIRC and consent[ed] to the
assessment and collection of taxes which may be found due after the
examination at any time after the lapse of the period of limitations fixed by said
Sections 223 and 224 and other relevant provisions of the NIRC, until the
completion of the investigation".6

On July 2, 1998, Revenue Officer De Vera submitted his audit report


recommending the issuance of an assessment and finding that petitioner had
deficiency taxes in the total amount of P136,952,408.97. On October 5, 1998,
the Assessment Division of the BIR issued Pre-Assessment Notices which informed
petitioner of the results of the investigation. Thus, BIR Revenue Region No. 6,
Assessment Division/Billing Section, issued Assessment/Demand No. 33-1-000757-
947 on December 9, 1998 stating the following deficiency taxes, inclusive of
interest and compromise penalty:

Income Tax P108,743,694.88


Value 184,299.20
Added Tax

Expanded 2,363,220.38
Withholding
Tax
Total P111,291,214.46
On March 16, 1999, a Preliminary Collection Letter was sent by Deputy
Commissioner Romeo S. Panganiban to the petitioner to pay the assessment
within ten (10) days from receipt of the letter. On November 10, 1999, a Final
Notice Before Seizure8 was issued by the same deputy commissioner giving the
petitioner ten (10) days from receipt to pay. Petitioner received a copy of the
final notice on November 24, 1999. By letters dated November 26, 1999,
petitioner asked to be clarified how the tax liability of P111,291,214.46 was
reached and requested an extension of thirty (30) days from receipt of the
clarification within which to reply.9

The BIR received a follow-up letter from the petitioner asserting that its (PJI)
records do not show receipt of Tax Assessment/Demand No. 33-1-000757-
94.10 Petitioner also contested that the assessment had no factual and legal
basis. On March 28, 2000, a Warrant of Distraint and/or Levy No. 33-06-
04611 signed by Deputy Commissioner Romeo Panganiban for the BIR was
received by the petitioner.

Petitioner filed a Petition for Review12 with the Court of Tax Appeals (CTA) which
was amended on May 12, 2000. Petitioner complains: (a) that no assessment or
demand was received from the BIR; (b) that the warrant of distraint and/or levy
was without factual and legal bases as its issuance was premature; (c) that the
78

assessment, having been made beyond the 3-year prescriptive period, is null
and void; (d) that the issuance of the warrant without being given the
opportunity to dispute the same violates its right to due process; and (e) that
the grave prejudice that will be sustained if the warrant is enforced is enough
basis for the issuance of the writ of preliminary injunction.

On May 14, 2002, the CTA rendered its decision,13 to wit:

As to whether or not the assessment notices were received by the


petitioner, this Court rules in the affirmative.

To disprove petitioner’s allegation of non-receipt of the aforesaid


assessment notices, respondent presented a certification issued by the
Post Master of the Central Post Office, Manila to the effect that Registered
Letter No. 76134 sent by the BIR, Region No. 6, Manila on December 15,
1998 addressed to Phil. Journalists, Inc. at Journal Bldg., Railroad St.,
Manila was duly delivered to and received by a certain Alfonso Sanchez,
Jr. (Authorized Representative) on January 8, 1999. Respondent also
showed proof that in claiming Registered Letter No. 76134, Mr. Sanchez
presented three identification cards, one of which is his company ID with
herein petitioner.

However, as to whether or not the Waiver of the Statute of Limitations is


valid and binding on the petitioner is another question. Since the subject
assessments were issued beyond the three-year prescriptive period, it
becomes imperative on our part to rule first on the validity of the waiver
allegedly executed on September 22, 1997, for if this court finds the same
to be ineffective, then the assessments must necessarily fail.

After carefully examining the questioned Waiver of the Statute of


Limitations, this Court considers the same to be without any binding effect
on the petitioner for the following reasons:

The waiver is an unlimited waiver. It does not contain a definite expiration


date. Under RMO No. 20-90, the phrase indicating the expiry date of the
period agreed upon to assess/collect the tax after the regular three-year
period of prescription should be filled up…

Secondly, the waiver failed to state the date of acceptance by the


Bureau which under the aforequoted RMO should likewise be indicated…

Finally, petitioner was not furnished a copy of the waiver. It is to be noted


that under RMO No. 20-90, the waiver must be executed in three (3)
copies, the second copy of which is for the taxpayer. It is likewise required
79

that the fact of receipt by the taxpayer of his/her file copy be indicated in
the original copy. Again, respondent failed to comply.

It bears stressing that RMO No. 20-90 is directed to all concerned internal
revenue officers. The said RMO even provides that the procedures found
therein should be strictly followed, under pain of being administratively
dealt with should non-compliance result to prescription of the right to
assess/collect…

Thus, finding the waiver executed by the petitioner on September 22, 1997
to be suffering from legal infirmities, rendering the same invalid and
ineffective, the Court finds Assessment/Demand No. 33-1-000757-94 issued
on December 5, 1998 to be time-barred. Consequently, the Warrant of
Distraint and/or Levy issued pursuant thereto is considered null and void.

WHEREFORE, in view of all the foregoing, the instant Petition for Review is
hereby GRANTED. Accordingly, the deficiency income, value-added and
expanded withholding tax assessments issued by the respondent against
the petitioner on December 9, 1998, in the total amount of
P111,291,214.46 for the year 1994 are hereby declared CANCELLED,
WITHDRAWN and WITH NO FORCE AND EFFECT. Likewise, Warrant of
Distraint and/or Levy No. 33-06-046 is hereby declared NULL and VOID.

SO ORDERED.14

After the motion for reconsideration of the Commissioner of Internal Revenue


was denied by the CTA in a Resolution dated August 2, 2002, an appeal was
filed with the Court of Appeals on August 12, 2002.

In its decision dated August 5, 2003, the Court of Appeals disagreed with the
ruling of the CTA, to wit:

… The petition for review filed on 26 April 2000 with CTA was neither timely
filed nor the proper remedy. Only decisions of the BIR, denying the request
for reconsideration or reinvestigation may be appealed to the CTA. Mere
assessment notices which have become final after the lapse of the thirty
(30)-day reglementary period are not appealable. Thus, the CTA should
not have entertained the petition at all.

… [T]he CTA found the waiver executed by Phil. Journalists to be invalid


for the following reasons: (1) it does not indicate a definite expiration
date; (2) it does not state the date of acceptance by the BIR; and (3) Phil.
Journalist, the taxpayer, was not furnished a copy of the waiver. These
grounds are merely formal in nature. The date of acceptance by the BIR
does not categorically appear in the document but it states at the
bottom page that the BIR "accepted and agreed to:"…, followed by the
signature of the BIR’s authorized representative. Although the date of
acceptance was not stated, the document was dated 22 September
1997. This date could reasonably be understood as the same date of
80

acceptance by the BIR since a different date was not otherwise


indicated. As to the allegation that Phil. Journalists was not furnished a
copy of the waiver, this requirement appears ridiculous. Phil. Journalists,
through its comptroller, Lorenza Tolentino, signed the waiver. Why would it
need a copy of the document it knowingly executed when the reason
why copies are furnished to a party is to notify it of the existence of a
document, event or proceeding? …

As regards the need for a definite expiration date, this is the biggest flaw
of the decision. The period of prescription for the assessment of taxes may
be extended provided that the extension be made in writing and that it
be made prior to the expiration of the period of prescription. These are
the requirements for a valid extension of the prescriptive period. To these
requirements provided by law, the memorandum order adds that the
length of the extension be specified by indicating its expiration date. This
requirement could be reasonably construed from the rule on extension of
the prescriptive period. But this requirement does not apply in the instant
case because what we have here is not an extension of the prescriptive
period but a waiver thereof. These are two (2) very different things. What
Phil. Journalists executed was a renunciation of its right to invoke the
defense of prescription. This is a valid waiver. When one waives the
prescriptive period, it is no longer necessary to indicate the length of the
extension of the prescriptive period since the person waiving may no
longer use this defense.

WHEREFORE, the 02 August 2002 resolution and 14 May 2002 decision of


the CTA are hereby SET ASIDE. Respondent Phil. Journalists is ordered [to]
pay its assessed tax liability of P111,291,214.46.

SO ORDERED.15

Petitioner’s Motion for Reconsideration was denied in a Resolution dated March


31, 2004. Hence, this appeal on the following assignment of errors:

I.

The Honorable Court of Appeals committed grave error in ruling that it is


outside the jurisdiction of the Court of Tax Appeals to entertain the Petition
for Review filed by the herein Petitioner at the CTA despite the fact that
such case inevitably rests upon the validity of the issuance by the BIR of
warrants of distraint and levy contrary to the provisions of Section 7(1) of
Republic Act No. 1125.

II.

The Honorable Court of Appeals gravely erred when it ruled that failure to
comply with the provisions of Revenue Memorandum Order (RMO) No. 20-
90 is merely a formal defect that does not invalidate the waiver of the
statute of limitations without stating the legal justification for such
conclusion. Such ruling totally disregarded the mandatory requirements of
Section 222(b) of the Tax Code and its implementing regulation, RMO No.
81

20-90 which are substantive in nature. The RMO provides that violation
thereof subjects the erring officer to administrative sanction. This directive
shows that the RMO is not merely cover forms.

III.

The Honorable Court of Appeals gravely erred when it ruled that the
assessment notices became final and unappealable. The assessment
issued is void and legally non-existent because the BIR has no power to
issue an assessment beyond the three-year prescriptive period where
there is no valid and binding waiver of the statute of limitation.

IV.

The Honorable Court of Appeals gravely erred when it held that the
assessment in question has became final and executory due to the failure
of the Petitioner to protest the same. Respondent had no power to issue
an assessment beyond the three year period under the mandatory
provisions of Section 203 of the NIRC. Such assessment should be held void
and non-existent, otherwise, Section 203, an expression of a public policy,
would be rendered useless and nugatory. Besides, such right to assess
cannot be validly granted after three years since it would arise from a
violation of the mandatory provisions of Section 203 and would go against
the vested right of the Petitioner to claim prescription of assessment.

V.

The Honorable Court of Appeals committed grave error when it HELD


valid a defective waiver by considering the latter a waiver of the right to
invoke the defense of prescription rather than an extension of the three
year period of prescription (to make an assessment) as provided under
Section 222 in relation to Section 203 of the Tax Code, an interpretation
that is contrary to law, existing jurisprudence and outside of the purpose
and intent for which they were enacted.16

We find merit in the appeal.

The first assigned error relates to the jurisdiction of the CTA over the issues in this
case. The Court of Appeals ruled that only decisions of the BIR denying a
request for reconsideration or reinvestigation may be appealed to the CTA.
Since the petitioner did not file a request for reinvestigation or reconsideration
within thirty (30) days, the assessment notices became final and unappealable.
The petitioner now argue that the case was brought to the CTA because the
warrant of distraint or levy was illegally issued and that no assessment was
issued because it was based on an invalid waiver of the statutes of limitations.

We agree with petitioner. Section 7(1) of Republic Act No. 1125, the Act
Creating the Court of Tax Appeals, provides for the jurisdiction of that special
court:

SEC. 7. Jurisdiction. – The Court of Tax Appeals shall exercise exclusive


appellate jurisdiction to review by appeal, as herein provided –
82

(1) Decisions of the Commissioner of Internal Revenue in cases involving


disputed assessments, refunds of internal revenue taxes, fees or other
charges, penalties imposed in relation thereto, or other matters arising
under the National Internal Revenue Code or other laws or part of law
administered by the Bureau of Internal Revenue; (Emphasis supplied).

The appellate jurisdiction of the CTA is not limited to cases which involve
decisions of the Commissioner of Internal Revenue on matters relating to
assessments or refunds. The second part of the provision covers other cases that
arise out of the NIRC or related laws administered by the Bureau of Internal
Revenue. The wording of the provision is clear and simple. It gives the CTA the
jurisdiction to determine if the warrant of distraint and levy issued by the BIR is
valid and to rule if the Waiver of Statute of Limitations was validly effected.

This is not the first case where the CTA validly ruled on issues that did not relate
directly to a disputed assessment or a claim for refund. In Pantoja v. David,17 we
upheld the jurisdiction of the CTA to act on a petition to invalidate and annul
the distraint orders of the Commissioner of Internal Revenue. Also,
in Commissioner of Internal Revenue v. Court of Appeals,18 the decision of the
CTA declaring several waivers executed by the taxpayer as null and void, thus
invalidating the assessments issued by the BIR, was upheld by this Court.

The second and fifth assigned errors both focus on Revenue Memorandum
Circular No. 20-90 (RMO No. 20-90) on the requisites of a valid waiver of the
statute of limitations. The Court of Appeals held that the requirements and
procedures laid down in the RMO are only formal in nature and did not
invalidate the waiver that was signed even if the requirements were not strictly
observed.

The NIRC, under Sections 203 and 222,19 provides for a statute of limitations on
the assessment and collection of internal revenue taxes in order to safeguard
the interest of the taxpayer against unreasonable investigation.20 Unreasonable
investigation contemplates cases where the period for assessment extends
indefinitely because this deprives the taxpayer of the assurance that it will no
longer be subjected to further investigation for taxes after the expiration of a
reasonable period of time. As was held in Republic of the Phils. v. Ablaza:21

The law prescribing a limitation of actions for the collection of the income
tax is beneficial both to the Government and to its citizens; to the
Government because tax officers would be obliged to act promptly in the
making of assessment, and to citizens because after the lapse of the
period of prescription citizens would have a feeling of security against
unscrupulous tax agents who will always find an excuse to inspect the
books of taxpayers, not to determine the latter’s real liability, but to take
advantage of every opportunity to molest peaceful, law-abiding citizens.
Without such a legal defense taxpayers would furthermore be under
obligation to always keep their books and keep them open for inspection
subject to harassment by unscrupulous tax agents. The law on prescription
being a remedial measure should be interpreted in a way conducive to
bringing about the beneficent purpose of affording protection to the
83

taxpayer within the contemplation of the Commission which recommend


the approval of the law. (Emphasis supplied)

RMO No. 20-90 implements these provisions of the NIRC relating to the period of
prescription for the assessment and collection of taxes. A cursory reading of the
Order supports petitioner’s argument that the RMO must be strictly followed,
thus:

In the execution of said waiver, the following procedures should be followed:

1. The waiver must be in the form identified hereof. This form may be
reproduced by the Office concerned but there should be no deviation
from such form. The phrase "but not after __________ 19___" should be filled
up…

2. …

Soon after the waiver is signed by the taxpayer, the Commissioner of


Internal Revenue or the revenue official authorized by him, as hereinafter
provided, shall sign the waiver indicating that the Bureau has accepted
and agreed to the waiver. The date of such acceptance by the Bureau
should be indicated…

3. The following revenue officials are authorized to sign the waiver.

A. In the National Office

3. Commissioner For tax cases


involving more
than P1M
B. In the Regional Offices

1. The Revenue District Officer with respect to tax cases still


pending investigation and the period to assess is about to
prescribe regardless of amount.

5. The foregoing procedures shall be strictly followed. Any


revenue official found not to have complied with this Order
resulting in prescription of the right to assess/collect shall be
administratively dealt with. (Emphasis supplied)22

A waiver of the statute of limitations under the NIRC, to a certain extent, is a


derogation of the taxpayers’ right to security against prolonged and
unscrupulous investigations and must therefore be carefully and strictly
construed.23 The waiver of the statute of limitations is not a waiver of the right to
invoke the defense of prescription as erroneously held by the Court of Appeals.
It is an agreement between the taxpayer and the BIR that the period to issue
an assessment and collect the taxes due is extended to a date certain. The
84

waiver does not mean that the taxpayer relinquishes the right to invoke
prescription unequivocally particularly where the language of the document is
equivocal. For the purpose of safeguarding taxpayers from any unreasonable
examination, investigation or assessment, our tax law provides a statute of
limitations in the collection of taxes. Thus, the law on prescription, being a
remedial measure, should be liberally construed in order to afford such
protection. As a corollary, the exceptions to the law on prescription should
perforce be strictly construed.24 RMO No. 20-90 explains the rationale of a
waiver:

... The phrase "but not after _________ 19___" should be filled up. This
indicates the expiry date of the period agreed upon to assess/collect the
tax after the regular three-year period of prescription. The period agreed
upon shall constitute the time within which to effect the
assessment/collection of the tax in addition to the ordinary prescriptive
period. (Emphasis supplied)

As found by the CTA, the Waiver of Statute of Limitations, signed by petitioner’s


comptroller on September 22, 1997 is not valid and binding because it does not
conform with the provisions of RMO No. 20-90. It did not specify a definite
agreed date between the BIR and petitioner, within which the former may
assess and collect revenue taxes. Thus, petitioner’s waiver became unlimited in
time, violating Section 222(b) of the NIRC.

The waiver is also defective from the government side because it was signed
only by a revenue district officer, not the Commissioner, as mandated by the
NIRC and RMO No. 20-90. The waiver is not a unilateral act by the taxpayer or
the BIR, but is a bilateral agreement between two parties to extend the period
to a date certain. The conformity of the BIR must be made by either the
Commissioner or the Revenue District Officer. This case involves taxes
amounting to more than One Million Pesos (P1,000,000.00) and executed
almost seven months before the expiration of the three-year prescription
period. For this, RMO No. 20-90 requires the Commissioner of Internal Revenue
to sign for the BIR.

The case of Commissioner of Internal Revenue v. Court of Appeals,25 dealt with


waivers that were not signed by the Commissioner but were argued to have
been given implied consent by the BIR. We invalidated the subject waivers and
ruled:

Petitioner’s submission is inaccurate…

The Court of Appeals itself also passed upon the validity of the waivers
executed by Carnation, observing thus:

We cannot go along with the petitioner’s theory. Section 319 of the


Tax Code earlier quoted is clear and explicit that the waiver of the
five-year26 prescriptive period must be in writing and signed by both
the BIR Commissioner and the taxpayer.
85

Here, the three waivers signed by Carnation do not bear the written
consent of the BIR Commissioner as required by law.

We agree with the CTA in holding "these ‘waivers’ to be invalid and


without any binding effect on petitioner (Carnation) for the reason
that there was no consent by the respondent (Commissioner of
Internal Revenue)."

For sure, no such written agreement concerning the said three


waivers exists between the petitioner and private respondent
Carnation.

What is more, the waivers in question reveal that they are in no wise
unequivocal, and therefore necessitates for its binding effect the
concurrence of the Commissioner of Internal Revenue…. On this basis
neither implied consent can be presumed nor can it be contended that
the waiver required under Sec. 319 of the Tax Code is one which is
unilateral nor can it be said that concurrence to such an agreement is a
mere formality because it is the very signatures of both the Commissioner
of Internal Revenue and the taxpayer which give birth to such a valid
agreement.27 (Emphasis supplied)

The other defect noted in this case is the date of acceptance which makes it
difficult to fix with certainty if the waiver was actually agreed before the
expiration of the three-year prescriptive period. The Court of Appeals held that
the date of the execution of the waiver on September 22, 1997 could
reasonably be understood as the same date of acceptance by the BIR.
Petitioner points out however that Revenue District Officer Sarmiento could not
have accepted the waiver yet because she was not the Revenue District
Officer of RDO No. 33 on such date. Ms. Sarmiento’s transfer and assignment to
RDO No. 33 was only signed by the BIR Commissioner on January 16, 1998 as
shown by the Revenue Travel Assignment Order No. 14-98.28 The Court of Tax
Appeals noted in its decision that it is unlikely as well that Ms. Sarmiento made
the acceptance on January 16, 1998 because "Revenue Officials normally
have to conduct first an inventory of their pending papers and property
responsibilities."29

Finally, the records show that petitioner was not furnished a copy of the waiver.
Under RMO No. 20-90, the waiver must be executed in three copies with the
second copy for the taxpayer. The Court of Appeals did not think this was
important because the petitioner need not have a copy of the document it
knowingly executed. It stated that the reason copies are furnished is for a party
to be notified of the existence of a document, event or proceeding.

The flaw in the appellate court’s reasoning stems from its assumption that the
waiver is a unilateral act of the taxpayer when it is in fact and in law an
agreement between the taxpayer and the BIR. When the petitioner’s
86

comptroller signed the waiver on September 22, 1997, it was not yet complete
and final because the BIR had not assented. There is compliance with the
provision of RMO No. 20-90 only after the taxpayer received a copy of the
waiver accepted by the BIR. The requirement to furnish the taxpayer with a
copy of the waiver is not only to give notice of the existence of the document
but of the acceptance by the BIR and the perfection of the agreement.

The waiver document is incomplete and defective and thus the three-year
prescriptive period was not tolled or extended and continued to run until April
17, 1998. Consequently, the Assessment/Demand No. 33-1-000757-94 issued on
December 9, 1998 was invalid because it was issued beyond the three (3) year
period. In the same manner, Warrant of Distraint and/or Levy No. 33-06-046
which petitioner received on March 28, 2000 is also null and void for having
been issued pursuant to an invalid assessment.

WHEREFORE, premises considered, the instant petition for review


is GRANTED. The Decision of the Court of Appeals dated August 5, 2003 and its
Resolution dated March 31, 2004 are REVERSED and SET ASIDE. The Decision of
the Court of Tax Appeals in CTA Case No. 6108 dated May 14, 2002, declaring
Warrant of Distraint and/or Levy No. 33-06-046 null and void, is REINSTATED.

SO ORDERED.

BANK OF THE PHILIPPINE G.R. No. 139736


ISLANDS,
Present:
P e t i t i o n e r,

PUNO,

Chairman

AUSTRIA-MARTINEZ,

CALLEJO, SR.,
- versus-
TINGA, and

CHICO-NAZARIO, JJ .

Promulgated:

COMMISSIONER OF
INTERNAL REVENUE, October 17, 2005

Respondent.
x- - - - - - - - - - - - - - - - - - - - - - - - - - - - - - - - - - - - - - - - - - - - - - - - - - -x
87

DECISION

CHICO-NAZARIO, J .:

This Petition for Review on Certiorari, under Rule 45 of the 1997 Rules of Civil
Procedure, assails the Decision of the Court of Appeals in CA-G.R. SP No. 51271,
dated 11 August 1999, [1] which reversed and set aside the Decision of the
Court of Tax Appeals (CTA), dated 02 February 1999, [2] and which reinstated
Assessment No. FAS-5-85-89-002054 requiring petitioner Bank of the Philippine
Islands (BPI) to pay the amount of P28,020.00 as deficiency documentary stamp
tax (DST) for the taxable year 1985, inclusive of the compromise penalty.

There is hardly any controversy as to the factual antecedents of this Petition.

Petitioner BPI is a commercial banking corporation organized and existing


under the laws of the Philippines. On two separate occasions, particularly on 06
June 1985 and 14 June 1985, it sold United States (US) $500,000.00 to the Central
Bank of the Philippines (Central Bank), for the total sales amount of
US$1,000,000.00.

On 10 October 1989, the Bureau of Internal Revenue (BIR) issued Assessment


No. FAS-5-85-89-002054, [3] finding petitioner BPI liable for deficiency DST on its
afore-mentioned sales of foreign bills of exchange to the Central Bank,
computed as follows '

1985 Deficiency Documentary Stamp Tax

Foreign Bills of Exchange.. P 18,480,000.00

Tax Due Thereon:


88

P 18,480,000.00 x P0.30 (Sec. 182 NIRC). 27,720.00

P200.00

Add: Suggested compromise penalty. 300.00

TOTAL AMOUNT DUE AND COLLECTIBLE. P 28,020.00

Petitioner BPI received the Assessment, together with the attached Assessment
Notice, [4] on 20 October 1989.

Petitioner BPI, through its counsel, protested the Assessment in a letter dated 16
November 1989, and filed with the BIR on 17 November 1989. The said protest
letter is reproduced in full below '

November 16, 1989

The Commissioner of Internal Revenue

Quezon City

Attention of: Mr. Pedro C. Aguillon

Asst. Commissioner for Collection

Sir:

On behalf of our client, Bank of the Philippine Islands (BPI), we have


the honor to protest your assessment against it for deficiency
documentary stamp tax for the year 1985 in the amount
of P28,020.00, arising from its sale to the Central Bank of U.S.
$500,000.00 on June 6, 1985 and another U.S. $500,000.00 on June 14,
1985.

1. ' Under established market practice, the documentary stamp tax


on telegraphic transfers or sales of foreign exchange is paid by the
buyer. Thus, when BPI sells to any party, the cost of documentary
stamp tax is added to the total price or charge to the buyer and the
seller affixes the corresponding documentary stamp on the
document. Similarly, when the Central Bank sells foreign exchange to
89

BPI, it charges BPI for the cost of the documentary stamp on the
transaction.

2. In the two transactions subject of your assessment, no


documentary stamps were affixed because the buyer,
Central Bank of the Philippines, was exempt from such tax. And while
it is true that under P.D. 1994, a proviso was added to sec. 222 (now
sec. 186) of the Tax Code 'that whenever one party to a taxable
document enjoys exemption from the tax herein imposed, the other
party thereto who is not exempt shall be the one directly liable for
the tax, this proviso (and the other amendments of P.D. 1994) took
effect only on January 1, 1986, according to sec. 49 of P.D. 1994.
Hence, the liability for the documentary stamp tax could not be
shifted to the seller.

In view of the foregoing, we request that the assessment be revoked


and cancelled.

Very truly yours,


PADILLA LAW OFFICE
By:

(signed)
SABINO PADILLA, JR. [5]

Petitioner BPI did not receive any immediate reply to its protest letter. However,
on 15 October 1992, the BIR issued a Warrant of Distraint and/or Levy [6] against
petitioner BPI for the assessed deficiency DST for taxable year 1985, in the
amount of P27,720.00 (excluding the compromise penalty of P300.00). It served
the Warrant on petitioner BPI only on 23 October 1992. [7]

Then again, petitioner BPI did not hear from the BIR until 11 September 1997,
when its counsel received a letter, dated 13 August 1997, signed by then BIR
Commissioner Liwayway Vinzons-Chato, denying its' 'request for
reconsideration, and addressing the points raised by petitioner BPI in its protest
letter, dated 16 November 1989, thus '

In reply, please be informed that after a thorough and careful study


of the facts of the case as well as the law and jurisprudence
pertinent thereto, this Office finds the above argument to be legally
untenable. It is admitted that while industry practice or market
convention has the force of law between the members of a
particular industry, it is not binding with the BIR since it is not a party
thereto. The same should, therefore, not be allowed to prejudice the
Bureau of its lawful task of collecting revenues necessary to defray
90

the expenses of the government. (Art. 11 in relation to Art. 1306 of


the New Civil Code.)

Moreover, let it be stated that even before the amendment of Sec.


222 (now Sec. 173) of the Tax Code, as amended, the same was
already interpreted to hold that the other party who is not exempt
from the payment of documentary stamp tax liable from the tax. This
interpretation was further strengthened by the following BIR Rulings
which in substance state:

1. BIR Unnumbered Ruling dated May 30, 1977 '

'x x x Documentary stamp taxes are payable by either person,


signing, issuing, accepting, or transferring the instrument, document
or paper. It is now settled that where one party to the instrument is
exempt from said taxes, the other party who is not exempt should be
liable.

2. BIR Ruling No. 144-84 dated September 3, 1984 '

'x x x Thus, where one party to the contract is exempt from said tax,
the other party, who is not exempt, shall be liable therefore.
Accordingly, since A.J.L. Construction Corporation, the other party to
the contract and the one assuming the payment of the expenses
incidental to the registration in the vendee's name of the property
sold, is not exempt from said tax, then it is the one liable therefore,
pursuant to Sec. 245 (now Sec. 196), in relation to Sec. 222 (now Sec.
173), both of the Tax Code of 1977, as amended.

Premised on all the foregoing considerations, your request for


reconsideration is hereby DENIED. [8]

Upon receipt of the above-cited letter from the BIR, petitioner BPI proceeded
to file a Petition for Review with the CTA on 10 October 1997; [9] to which
respondent BIR Commissioner, represented by the Office of the Solicitor
General, filed an Answer on 08 December 1997. [10]

Petitioner BPI raised in its Petition for Review before the CTA, in addition to the
arguments presented in its protest letter, dated 16 November 1989, the defense
of prescription of the right of respondent BIR Commissioner to enforce
collection of the assessed amount. It alleged that respondent BIR Commissioner
only had three years to collect on Assessment No. FAS-5-85-89-002054, but she
waited for seven years and nine months to deny the protest. In her Answer and
subsequent Memorandum, respondent BIR Commissioner merely reiterated her
position, as stated in her letter to petitioner BPI, dated 13 August 1997, which
91

denied the latter's protest; and remained silent as to the expiration of the
prescriptive period for collection of the assessed deficiency DST.

After due trial, the CTA rendered a Decision on 02 February 1999, in which it
identified two primary issues in the controversy between petitioner BPI and
respondent BIR Commissioner: (1) whether or not the right of respondent BIR
Commissioner to collect from petitioner BPI the alleged deficiency DST for
taxable year 1985 had prescribed; and (2) whether or not the sales of
US$1,000,000.00 on 06 June 1985 and 14 June 1985 by petitioner BPI to the
Central Bank were subject to DST.

The CTA answered the first issue in the negative and held that the statute of
limitations for respondent BIR Commissioner to collect on the Assessment had
not yet prescribed. In resolving the issue of prescription, the CTA reasoned that '

In the case of Commissioner of Internal Revenue vs. Wyeth Suaco


Laboratories, Inc., G.R. No. 76281, September 30, 1991, 202 SCRA
125, the Supreme Court laid to rest the first issue. It categorically
ruled that a 'protest is to be treated as request for reinvestigation or
reconsideration and a mere request for reexamination or
reinvestigation tolls the prescriptive period of the Commissioner to
collect on an assessment. . .
...

In the case at bar, there being no dispute that petitioner filed its
protest on the subject assessment on November 17, 1989, there can
be no conclusion other than that said protest stopped the running of
the prescriptive period of the Commissioner to collect.

Section 320 (now 223) of the Tax Code, clearly states that a request
for reinvestigation which is granted by the Commissioner, shall
suspend the prescriptive period to collect. The underscored portion
above does not mean that the Commissioner will cancel the subject
assessment but should be construed as when the same was
entertained by the Commissioner by not issuing any warrant of
distraint or levy on the properties of the taxpayer or any action
prejudicial to the latter unless and until the request for reinvestigation
is finally given due course. Taking into consideration this provision of
law and the aforementioned ruling of the Supreme Court in Wyeth
Suaco which specifically and categorically states that a protest
could be considered as a request for reinvestigation, We rule that
prescription has not set in against the government. [11]

The CTA had likewise resolved the second issue in the negative. Referring to its
own decision in an earlier case, Consolidated Bank & Trust Co. v. The
92

Commissioner of Internal Revenue, [12] the CTA reached the conclusion that
the sales of foreign currency by petitioner BPI to the Central Bank in taxable
year 1985 were not subject to DST '

From the abovementioned decision of this Court, it can be gleaned


that the Central Bank, during the period June 11, 1984 to March 9,
1987 enjoyed tax exemption privilege, including the payment of
documentary stamp tax (DST) pursuant to Resolution No. 35-85
dated May 3, 1985 of the Fiscal Incentive Review Board. As such, the
Central Bank, as buyer of the foreign currency, is exempt from
paying the documentary stamp tax for the period above-
mentioned. This Court further expounded that said tax exemption of
the Central Bank was modified beginning January 1, 1986 when
Presidential Decree (P.D.) 1994 took effect. Under this decree, the
liability for DST on sales of foreign currency to the Central Bank is
shifted to the seller.

Applying the above decision to the case at bar, petitioner cannot


be held liable for DST on its 1985 sales of foreign currencies to the
Central Bank, as the latter who is the purchaser of the subject
currencies is the one liable thereof. However, since the Central Bank
is exempt from all taxes during 1985 by virtue of Resolution No. 35-85
of the Fiscal Incentive Review Board dated March 3, 1985, neither
the petitioner nor the Central Bank is liable for the payment of the
documentary stamp tax for the former's 1985 sales of foreign
currencies to the latter. This aforecited case of Consolidated Bank vs.
Commissioner of Internal Revenue was affirmed by the Court of
Appeals in its decision dated March 31, 1995, CA-GR Sp. No. 35930.
Said decision was in turn affirmed by the Supreme Court in its
resolution denying the petition filed by Consolidated Bank dated
November 20, 1995 with the Supreme Court under Entry of Judgment
dated March 1, 1996. [13]

In sum, the CTA decided that the statute of limitations for respondent BIR
Commissioner to collect on Assessment No. FAS-5-85-89-002054 had not yet
prescribed; nonetheless, it still ordered the cancellation of the said Assessment
because the sales of foreign currency by petitioner BPI to the Central Bank in
taxable year 1985 were tax-exempt.

Herein respondent BIR Commissioner appealed the Decision of the CTA to the
Court of Appeals. In its Decision dated 11 August 1999, [14] the Court of
Appeals sustained the finding of the CTA on the first issue, that the running of
the prescriptive period for collection on Assessment No. FAS-5-85-89-002054 was
suspended when herein petitioner BPI filed a protest on 17 November 1989 and,
therefore, the prescriptive period for collection on the Assessment had not yet
93

lapsed. In the same Decision, however, the Court of Appeals reversed the CTA
on the second issue and basically adopted the position of the respondent BIR
Commissioner that the sales of foreign currency by petitioner BPI to the Central
Bank in taxable year 1985 were subject to DST. The Court of Appeals, thus,
ordered the reinstatement of Assessment No. FAS-5-85-89-002054 which
required petitioner BPI to pay the amount of P28,020.00 as deficiency DST for
taxable year 1985, inclusive of the compromise penalty.

Comes now petitioner BPI before this Court in this Petition for Review
on Certiorari, seeking resolution of the same two legal issues raised and
discussed in the courts below, to reiterate: (1) whether or not the right of
respondent BIR Commissioner to collect from petitioner BPI the alleged
deficiency DST for taxable year 1985 had prescribed; and (2) whether or not
the sales of US$1,000,000.00 on 06 June 1985 and 14 June 1985 by petitioner BPI
to the Central Bank were subject to DST.

The efforts of respondent Commissioner to collect on Assessment No.


FAS-5-85-89-002054 were already barred by prescription.

Anent the question of prescription, this Court disagrees in the Decisions of the
CTA and the Court of Appeals, and herein determines the statute of limitations
on collection of the deficiency DST in Assessment No. FAS-5-85-89-002054 had
already prescribed.

The period for the BIR to assess and collect an internal revenue tax is limited to
three years by Section 203 of the Tax Code of 1977, as amended, [15] which
provides that '

SEC. 203. Period of limitation upon assessment and collection. '


Except as provided in the succeeding section, internal revenue taxes
shall be assessed within three years after the last day prescribed by
law for the filing of the return, and no proceeding in court without
assessment for the collection of such taxes shall be begun after the
expiration of such period: Provided, That in a case where a return is
filed beyond the period prescribed by law, the three-year period
shall be counted from the day the return was filed. For the purposes
of this section, a return filed before the last day prescribed by law for
the filing thereof shall be considered as filed on such last day. [16]
94

The three-year period of limitations on the assessment and collection of


national internal revenue taxes set by Section 203 of the Tax Code of 1977, as
amended, can be affected, adjusted, or suspended, in accordance with the
following provisions of the same Code '

SEC. 223. ' Exceptions as to period of limitation of assessment and


collection of taxes. ' (a) In the case of a false or fraudulent return
with intent to evade tax or of failure to file a return, the tax may be
assessed, or a proceeding in court for the collection of such tax may
be begun without assessment, at any time within ten years after the
discovery of the falsity, fraud, or omission: Provided, That in a fraud
assessment which has become final and executory, the fact of fraud
shall be judicially taken cognizance of in the civil or criminal action
for the collection thereof.

(b) If before the expiration of the time prescribed in the preceding


section for the assessment of the tax, both the Commissioner and the
taxpayer have agreed in writing to its assessment after such time the
tax may be assessed within the period agreed upon. The period so
agreed upon may be extended by subsequent written agreement
made before the expiration of the period previously agreed upon.

(c) Any internal revenue tax which has been assessed within the
period of limitation above-prescribed may be collected by distraint
or levy or by a proceeding in court within three years following the
assessment of the tax.

(d) Any internal revenue tax which has been assessed within the
period agreed upon as provided in paragraph (b) hereinabove may
be collected by distraint or levy or by a proceeding in court within
the period agreed upon in writing before the expiration of the three-
year period. The period so agreed upon may be extended by
subsequent written agreements made before the expiration of the
period previously agreed upon.

(e) Provided, however, That nothing in the immediately preceding


section and paragraph (a) hereof shall be construed to authorize
the examination and investigation or inquiry into any tax returns filed
in accordance with the provisions of any tax amnesty law or
decree. [17]

SEC. 224. Suspension of running of statute. ' The running of the statute
of limitation provided in Section[s] 203 and 223 on the making of
assessment and the beginning of distraint or levy or a proceeding in
court for collection, in respect of any deficiency, shall be suspended
for the period during which the Commissioner is prohibited from
making the assessment or beginning distraint or levy or a proceeding
95

in court and for sixty days thereafter; when the taxpayer requests for
a reinvestigation which is granted by the Commissioner; when the
taxpayer cannot be located in the address given by him in the
return filed upon which a tax is being assessed or
collected: Provided, That, if the taxpayer informs the Commissioner
of any change in address, the running of the statute of limitations will
not be suspended; when the warrant of distraint and levy is duly
served upon the taxpayer, his authorized representative, or a
member of his household with sufficient discretion, and no property
could be located; and when the taxpayer is out of the
Philippines. [18]

As enunciated in these statutory provisions, the BIR has three years, counted
from the date of actual filing of the return or from the last date prescribed by
law for the filing of such return, whichever comes later, to assess a national
internal revenue tax or to begin a court proceeding for the collection thereof
without an assessment. In case of a false or fraudulent return with intent to
evade tax or the failure to file any return at all, the prescriptive period for
assessment of the tax due shall be 10 years from discovery by the BIR of the
falsity, fraud, or omission. When the BIR validly issues an assessment, within either
the three-year or ten-year period, whichever is appropriate, then the BIR has
another three years [19] after the assessment within which to collect the
national internal revenue tax due thereon by distraint, levy, and/or court
proceeding. The assessment of the tax is deemed made and the three-year
period for collection of the assessed tax begins to run on the date the
assessment notice had been released, mailed or sent by the BIR to the
taxpayer. [20]

In the present Petition, there is no controversy on the timeliness of the issuance


of the Assessment, only on the prescription of the period to collect the
deficiency DST following its Assessment. While Assessment No. FAS-5-85-89-
002054 and its corresponding Assessment Notice were both dated 10 October
1989 and were received by petitioner BPI on 20 October 1989, there was no
showing as to when the said Assessment and Assessment Notice were released,
mailed or sent by the BIR. Still, it can be granted that the latest date the BIR
could have released, mailed or sent the Assessment and Assessment Notice to
petitioner BPI was on the same date they were received by the latter, on 20
October 1989. Counting the three-year prescriptive period, for a total of 1,095
days, [21] from 20 October 1989, then the BIR only had until 19 October 1992
within which to collect the assessed deficiency DST.

The earliest attempt of the BIR to collect on Assessment No. FAS-5-85-89-002054


was its issuance and service of a Warrant of Distraint and/or Levy on petitioner
BPI. Although the Warrant was issued on 15 October 1992, previous to the
96

expiration of the period for collection on 19 October 1992, the same was
served on petitioner BPI only on 23 October 1992.

Under Section 223(c) of the Tax Code of 1977, as amended, it is not essential
that the Warrant of Distraint and/or Levy be fully executed so that it can
suspend the running of the statute of limitations on the collection of the tax. It is
enough that the proceedings have validly began or commenced and that
their execution has not been suspended by reason of the voluntary desistance
of the respondent BIR Commissioner. Existing jurisprudence establishes that
distraint and levy proceedings are validly begun or commenced by the
issuance of the Warrant and service thereof on the taxpayer. [22] It is only
logical to require that the Warrant of Distraint and/or Levy be, at the very least,
served upon the taxpayer in order to suspend the running of the prescriptive
period for collection of an assessed tax, because it may only be upon the
service of the Warrant that the taxpayer is informed of the denial by the BIR of
any pending protest of the said taxpayer, and the resolute intention of the BIR
to collect the tax assessed.

If the service of the Warrant of Distraint and/or Levy on petitioner BPI on 23


October 1992 was already beyond the prescriptive period for collection of the
deficiency DST, which had expired on 19 October 1992, then what more the
letter of respondent BIR Commissioner, dated 13 August 1997 and received by
the counsel of the petitioner BPI only on 11 September 1997, denying the
protest of petitioner BPI and requesting payment of the deficiency DST? Even
later and more unequivocally barred by prescription on collection was the
demand made by respondent BIR Commissioner for payment of the deficiency
DST in her Answer to the Petition for Review of petitioner BPI before the CTA,
filed on 08 December 1997. [23]

II

There is no valid ground for the suspension of the running of the


prescriptive period for collection of the assessed DST under the Tax
Code of 1977, as amended.

In their Decisions, both the CTA and the Court of Appeals found that the filing
by petitioner BPI of a protest letter suspended the running of the prescriptive
period for collecting the assessed DST. This Court, however, takes the opposing
view, and, based on the succeeding discussion, concludes that there is no
valid ground for suspending the running of the prescriptive period for collection
of the deficiency DST assessed against petitioner BPI.
97

A. The statute of limitations on assessment and collection of taxes is for the


protection of the taxpayer and, thus, shall be construed liberally in his
favor.

Though the statute of limitations on assessment and collection of national


internal revenue taxes benefits both the Government and the taxpayer, it
principally intends to afford protection to the taxpayer against unreasonable
investigation. The indefinite extension of the period for assessment is
unreasonable because it deprives the said taxpayer of the assurance that he
will no longer be subjected to further investigation for taxes after the expiration
of a reasonable period of time. [24] As aptly explained in Republic of the
Philippines v. Ablaza [25]

The law prescribing a limitation of actions for the collection of the


income tax is beneficial both to the Government and to its citizens;
to the Government because tax officers would be obliged to act
promptly in the making of assessment, and to citizens because after
the lapse of the period of prescription citizens would have a feeling
of security against unscrupulous tax agents who will always find an
excuse to inspect the books of taxpayers, not to determine the
latter's real liability, but to take advantage of every opportunity to
molest peaceful, law-abiding citizens. Without such a legal defense
taxpayers would furthermore be under obligation to always keep
their books and keep them open for inspection subject to
harassment by unscrupulous tax agents. The law on prescription
being a remedial measure should be interpreted in a way
conducive to bringing about the beneficent purpose of affording
protection to the taxpayer within the contemplation of the
Commission which recommend the approval of the law.

In order to provide even better protection to the taxpayer against


unreasonable investigation, the Tax Code of 1977, as amended, identifies
specifically in Sections 223 and 224 [26] thereof the circumstances when the
prescriptive periods for assessing and collecting taxes could be suspended or
interrupted.

To give effect to the legislative intent, these provisions on the statute of


limitations on assessment and collection of taxes shall be construed and
applied liberally in favor of the taxpayer and strictly against the Government.
98

B. The statute of limitations on assessment and collection of national internal


revenue taxes may be waived, subject to certain conditions, under
paragraphs (b) and (d) of Section 223 of the Tax Code of 1977, as
amended, respectively. Petitioner BPI, however, did not execute any such
waiver in the case at bar.

According to paragraphs (b) and (d) of Section 223 of the Tax Code of 1977, as
amended, the prescriptive periods for assessment and collection of national
internal revenue taxes, respectively, could be waived by agreement, to wit '

SEC. 223. ' Exceptions as to period of limitation of assessment and


collection of taxes. '
...

(b) If before the expiration of the time prescribed in the preceding


section for the assessment of the tax, both the Commissioner and the
taxpayer have agreed in writing to its assessment after such time the
tax may be assessed within the period agreed upon. The period so
agreed upon may be extended by subsequent written agreement
made before the expiration of the period previously agreed upon.
...

(d) Any internal revenue tax which has been assessed within the
period agreed upon as provided in paragraph (b) hereinabove may
be collected by distraint or levy or by a proceeding in court within
the period agreed upon in writing before the expiration of the three-
year period. The period so agreed upon may be extended by
subsequent written agreements made before the expiration of the
period previously agreed upon. [27]

The agreements so described in the afore-quoted provisions are often referred


to as waivers of the statute of limitations. The waiver of the statute of limitations,
whether on assessment or collection, should not be construed as a waiver of
the right to invoke the defense of prescription but, rather, an agreement
between the taxpayer and the BIR to extend the period to a date certain,
within which the latter could still assess or collect taxes due. The waiver does
not mean that the taxpayer relinquishes the right to invoke prescription
unequivocally. [28]
99

A valid waiver of the statute of limitations under paragraphs (b) and (d) of
Section 223 of the Tax Code of 1977, as amended, must be: (1) in writing; (2)
agreed to by both the Commissioner and the taxpayer; (3) before the
expiration of the ordinary prescriptive periods for assessment and collection;
and (4) for a definite period beyond the ordinary prescriptive periods for
assessment and collection. The period agreed upon can still be extended by
subsequent written agreement, provided that it is executed prior to the
expiration of the first period agreed upon. The BIR had issued Revenue
Memorandum Order (RMO) No. 20-90 on 04 April 1990 to lay down an even
more detailed procedure for the proper execution of such a waiver. RMO No.
20-90 mandates that the procedure for execution of the waiver shall be strictly
followed, and any revenue official who fails to comply therewith resulting in the
prescription of the right to assess and collect shall be administratively dealt
with.

This Court had consistently ruled in a number of cases that a request for
reconsideration or reinvestigation by the taxpayer, without a valid waiver of the
prescriptive periods for the assessment and collection of tax, as required by the
Tax Code and implementing rules, will not suspend the running thereof. [29]

In the Petition at bar, petitioner BPI executed no such waiver of the statute of
limitations on the collection of the deficiency DST per Assessment No. FAS-5-85-
89-002054. In fact, an internal memorandum of the Chief of the Legislative,
Ruling & Research Division of the BIR to her counterpart in the Collection
Enforcement Division, dated 15 October 1992, expressly noted that, 'The
taxpayer fails to execute a Waiver of the Statute of Limitations extending the
period of collection of the said tax up to December 31, 1993 pending
reconsideration of its protest. . . [30] Without a valid waiver, the statute of
limitations on collection by the BIR of the deficiency DST could not have been
suspended under paragraph (d) of Section 223 of the Tax Code of 1977, as
amended.

C. The protest filed by petitioner BPI did not constitute a request for
reinvestigation, granted by the respondent BIR Commissioner, which could
have suspended the running of the statute of limitations on collection of
the assessed deficiency DST under Section 224 of the Tax Code of 1977, as
amended.

The Tax Code of 1977, as amended, also recognizes instances when the
running of the statute of limitations on the assessment and collection of
national internal revenue taxes could be suspended, even in the absence of a
waiver, under Section 224 thereof, which reads '
100

SEC. 224. Suspension of running of statute. ' The running of the statute
of limitation provided in Section[s] 203 and 223 on the making of
assessment and the beginning of distraint or levy or a proceeding in
court for collection, in respect of any deficiency, shall be suspended
for the period during which the Commissioner is prohibited from
making the assessment or beginning distraint or levy or a proceeding
in court and for sixty days thereafter; when the taxpayer requests for
a reinvestigation which is granted by the Commissioner; when the
taxpayer cannot be located in the address given by him in the
return filed upon which a tax is being assessed or
collected: Provided, That, if the taxpayer informs the Commissioner
of any change in address, the running of the statute of limitations will
not be suspended; when the warrant of distraint and levy is duly
served upon the taxpayer, his authorized representative, or a
member of his household with sufficient discretion, and no property
could be located; and when the taxpayer is out of the
Philippines. [31]

Of particular importance to the present case is one of the circumstances


enumerated in Section 224 of the Tax Code of 1977, as amended, wherein the
running of the statute of limitations on assessment and collection of taxes is
considered suspended 'when the taxpayer requests for a reinvestigation which
is granted by the Commissioner.

This Court gives credence to the argument of petitioner BPI that there is a
distinction between a request for reconsideration and a request for
reinvestigation. Revenue Regulations (RR) No. 12-85, issued on 27 November
1985 by the Secretary of Finance, upon the recommendation of the BIR
Commissioner, governs the procedure for protesting an assessment and
distinguishes between the two types of protest, as follows '

PROTEST TO ASSESSMENT

SEC. 6. Protest. The taxpayer may protest administratively an


assessment by filing a written request for reconsideration or
reinvestigation. . .
...

For the purpose of the protest herein '

(a) Request for reconsideration. ' refers to a plea for a re-evaluation


of an assessment on the basis of existing records without need of
101

additional evidence. It may involve both a question of fact or of law


or both.

(b) Request for reinvestigation. ' refers to a plea for re-evaluation of


an assessment on the basis of newly-discovered or additional
evidence that a taxpayer intends to present in the reinvestigation. It
may also involve a question of fact or law or both.

With the issuance of RR No. 12-85 on 27 November 1985 providing the above-
quoted distinctions between a request for reconsideration and a request for
reinvestigation, the two types of protest can no longer be used
interchangeably and their differences so lightly brushed aside. It bears to
emphasize that under Section 224 of the Tax Code of 1977, as amended, the
running of the prescriptive period for collection of taxes can only be suspended
by a request for reinvestigation , not a request for reconsideration.
Undoubtedly, a reinvestigation, which entails the reception and evaluation of
additional evidence, will take more time than a reconsideration of a tax
assessment, which will be limited to the evidence already at hand; this justifies
why the former can suspend the running of the statute of limitations on
collection of the assessed tax, while the latter can not.

The protest letter of petitioner BPI, dated 16 November 1989 and filed with the
BIR the next day, on 17 November 1989, did not specifically request for either a
reconsideration or reinvestigation. A close review of the contents thereof would
reveal, however, that it protested Assessment No. FAS-5-85-89-002054 based on
a question of law, in particular, whether or not petitioner BPI was liable for DST
on its sales of foreign currency to the Central Bank in taxable year 1985. The
same protest letter did not raise any question of fact; neither did it offer to
present any new evidence. In its own letter to petitioner BPI, dated 10
September 1992, the BIR itself referred to the protest of petitioner BPI as a
request for reconsideration. [32] These considerations would lead this Court to
deduce that the protest letter of petitioner BPI was in the nature of a request for
reconsideration, rather than a request for reinvestigation and, consequently,
Section 224 of the Tax Code of 1977, as amended, on the suspension of the
running of the statute of limitations should not apply.

Even if, for the sake of argument, this Court glosses over the distinction between
a request for reconsideration and a request for reinvestigation, and considers
the protest of petitioner BPI as a request for reinvestigation, the filing thereof
could not have suspended at once the running of the statute of limitations.
Article 224 of the Tax Code of 1977, as amended, very plainly requires that the
request for reinvestigation had been granted by the BIR Commissioner to
suspend the running of the prescriptive periods for assessment and collection.
102

That the BIR Commissioner must first grant the request for reinvestigation as a
requirement for suspension of the statute of limitations is even supported by
existing jurisprudence.

In the case of Republic of the Philippines v. Gancayco, [33] taxpayer


Gancayco requested for a thorough reinvestigation of the assessment against
him and placed at the disposal of the Collector of Internal Revenue all the
evidences he had for such purpose; yet, the Collector ignored the request, and
the records and documents were not at all examined. Considering the given
facts, this Court pronounced that '

. . .The act of requesting a reinvestigation alone does not suspend


the period. The request should first be granted, in order to effect
suspension. (Collector vs. Suyoc Consolidated, supra; also Republic
vs. Ablaza, supra). Moreover, the Collector gave appellee until April
1, 1949, within which to submit his evidence, which the latter did one
day before. There were no impediments on the part of the Collector
to file the collection case from April 1, 1949. . . . [34]

In Republic of the Philippines v. Acebedo, [35] this Court similarly found that '

. . . [T]he defendant, after receiving the assessment notice of


September 24, 1949, asked for a reinvestigation thereof on October
11, 1949 (Exh. A). There is no evidence that this request was
considered or acted upon. In fact, on October 23, 1950 the then
Collector of Internal Revenue issued a warrant of distraint and levy
for the full amount of the assessment (Exh. D), but there was no
follow-up of this warrant. Consequently, the request for
reinvestigation did not suspend the running of the period for filing an
action for collection.

The burden of proof that the taxpayer's request for reinvestigation had been
actually granted shall be on respondent BIR Commissioner. The grant may be
expressed in communications with the taxpayer or implied from the actions of
the respondent BIR Commissioner or his authorized BIR representatives in
response to the request for reinvestigation.
103

In Querol v. Collector of Internal Revenue, [36] the BIR, after receiving the
protest letters of taxpayer Querol, sent a tax examiner to San Fernando,
Pampanga, to conduct the reinvestigation; as a result of which, the original
assessment against taxpayer Querol was revised by permitting him to deduct
reasonable depreciation. In another case, Republic of the Philippines v.
Lopez, [37] taxpayer Lopez filed a total of four petitions for reconsideration and
reinvestigation. The first petition was denied by the BIR. The second and third
petitions were granted by the BIR and after each reinvestigation, the assessed
amount was reduced. The fourth petition was again denied and, thereafter,
the BIR filed a collection suit against taxpayer Lopez. When the taxpayers
spouses Sison, in Commissioner of Internal Revenue v. Sison, [38] contested the
assessment against them and asked for a reinvestigation, the BIR ordered the
reinvestigation resulting in the issuance of an amended assessment. Lastly,
in Republic of the Philippines v. Oquias, [39] the BIR granted taxpayer Oquias's
request for reinvestigation and duly notified him of the date when such
reinvestigation would be held; only, neither taxpayer Oquias nor his counsel
appeared on the given date.

In all these cases, the request for reinvestigation of the assessment filed by the
taxpayer was evidently granted and actual reinvestigation was conducted by
the BIR, which eventually resulted in the issuance of an amended assessment.
On the basis of these facts, this Court ruled in the same cases that the period
between the request for reinvestigation and the revised assessment should be
subtracted from the total prescriptive period for the assessment of the tax; and,
once the assessment had been reconsidered at the taxpayer's instance, the
period for collection should begin to run from the date of the reconsidered or
modified assessment. [40]

The rulings of the foregoing cases do not apply to the present Petition because:
(1) the protest filed by petitioner BPI was a request for reconsideration, not a
reinvestigation, of the assessment against it; and (2) even granting that the
protest of petitioner BPI was a request for reinvestigation, there was no showing
that it was granted by respondent BIR Commissioner and that actual
reinvestigation had been conducted.

Going back to the administrative records of the present case, it would seem
that the BIR, after receiving a copy of the protest letter of petitioner BPI on 17
November 1989, did not attempt to communicate at all with the latter until 10
September 1992, less than a month before the prescriptive period for collection
on Assessment No. FAS-5-85-89-002054 was due to expire. There were internal
communications, mostly indorsements of the docket of the case from one BIR
division to another; but these hardly fall within the same sort of acts in the
previously discussed cases that satisfactorily demonstrated the grant of the
taxpayer's request for reinvestigation. Petitioner BPI, in the meantime, was left in
the dark as to the status of its protest in the absence of any word from the BIR.
104

Besides, in its letter to petitioner BPI, dated 10 September 1992, the BIR
unwittingly admitted that it had not yet acted on the protest of the former '

This refers to your protest against and/or request for reconsideration


of the assessment/s of this Office against you involving the amount
of P28,020.00 under FAS-5-85-89-002054 dated October 23, 1989 as
deficiency documentary stamp tax inclusive of compromise penalty
for the year 1985.

In this connection, it is requested that the enclosed waiver of the


statute of limitations extending the period of collection of the said
tax/es to December 31, 1993 be executed by you as a condition
precedent of our giving due course to your protest [41]

When the BIR stated in its letter, dated 10 September 1992, that the waiver of
the statute of limitations on collection was a condition precedent to its giving
due course to the request for reconsideration of petitioner BPI, then it was
understood that the grant of such request for reconsideration was being held
off until compliance with the given condition. When petitioner BPI failed to
comply with the condition precedent, which was the execution of the waiver,
the logical inference would be that the request was not granted and was not
given due course at all.

III

The suspension of the statute of limitations on collection of the


assessed deficiency DST from petitioner BPI does not find support in
jurisprudence.

It is the position of respondent BIR Commissioner, affirmed by the CTA and the
Court of Appeals, that the three-year prescriptive period for collecting on
Assessment No. FAS-5-85-89-002054 had not yet prescribed, because the said
prescriptive period was suspended, invoking the case of Commissioner of
Internal Revenue v. Wyeth Suaco Laboratories, Inc. [42] It was in this case in
which this Court ruled that the prescriptive period provided by law to make a
collection is interrupted once a taxpayer requests for reinvestigation or
reconsideration of the assessment.

Petitioner BPI, on the other hand, is requesting this Court to revisit the Wyeth
Suaco case contending that it had unjustifiably expanded the grounds for
105

suspending the prescriptive period for collection of national internal revenue


taxes.

This Court finds that although there is no compelling reason to abandon its
decision in the Wyeth Suaco case, the said case cannot be applied to the
particular facts of the Petition at bar.

A. The only exception to the statute of limitations on collection of taxes, other


than those already provided in the Tax Code, was recognized in the
Suyoc case.

As had been previously discussed herein, the statute of limitations on


assessment and collection of national internal revenue taxes may be
suspended if the taxpayer executes a valid waiver thereof, as provided in
paragraphs (b) and (d) of Section 223 of the Tax Code of 1977, as amended;
and in specific instances enumerated in Section 224 of the same Code, which
include a request for reinvestigation granted by the BIR Commissioner. Outside
of these statutory provisions, however, this Court also recognized one other
exception to the statute of limitations on collection of taxes in the case
of Collector of Internal Revenue v. Suyoc Consolidated Mining Co. [43]

In the said case, the Collector of Internal Revenue issued an assessment against
taxpayer Suyoc Consolidated Mining Co. on 11 February 1947 for deficiency
income tax for the taxable year 1941. Taxpayer Suyoc requested for at least a
year within which to pay the amount assessed, but at the same time, reserving
its right to question the correctness of the assessment before actual payment.
The Collector granted taxpayer Suyoc an extension of only three months to pay
the assessed tax. When taxpayer Suyoc failed to pay the assessed tax within
the extended period, the Collector sent it a demand letter, dated 28
November 1950. Upon receipt of the demand letter, taxpayer Suyoc asked for
a reinvestigation and reconsideration of the assessment, but the Collector
denied the request. Taxpayer Suyoc reiterated its request for reconsideration
on 25 April 1952, which was denied again by the Collector on 06 May 1953.
Taxpayer Suyoc then appealed the denial to the Conference Staff. The
Conference Staff heard the appeal from 02 September 1952 to 16 July 1955,
and the negotiations resulted in the reduction of the assessment on 26 July
1955. It was the collection of the reduced assessment that was questioned
before this Court for being enforced beyond the prescriptive period. [44]

In resolving the issue on prescription, this Court ratiocinated thus '


106

It is obvious from the foregoing that petitioner refrained from


collecting the tax by distraint or levy or by proceeding in court within
the 5-year period from the filing of the second amended final return
due to the several requests of respondent for extension to which
petitioner yielded to give it every opportunity to prove its claim
regarding the correctness of the assessment. Because of such
requests, several reinvestigations were made and a hearing was
even held by the Conference Staff organized in the collection office
to consider claims of such nature which, as the record shows, lasted
for several months. After inducing petitioner to delay collection as he
in fact did, it is most unfair for respondent to now take advantage of
such desistance to elude his deficiency income tax liability to the
prejudice of the Government invoking the technical ground of
prescription.

While we may agree with the Court of Tax Appeals that a mere
request for reexamination or reinvestigation may not have the effect
of suspending the running of the period of limitation for in such case
there is need of a written agreement to extend the period between
the Collector and the taxpayer, there are cases however where a
taxpayer may be prevented from setting up the defense of
prescription even if he has not previously waived it in writing as
when by his repeated requests or positive acts the Government has
been, for good reasons, persuaded to postpone collection to make
him feel that the demand was not unreasonable or that no
harassment or injustice is meant by the Government. And when such
situation comes to pass there are authorities that hold, based on
weighty reasons, that such an attitude or behavior should not be
countenanced if only to protect the interest of the Government. [45]

By the principle of estoppel, taxpayer Suyoc was not allowed to raise the
defense of prescription against the efforts of the Government to collect the tax
assessed against it. This Court adopted the following principle from American
jurisprudence: 'He who prevents a thing from being done may not avail himself
of the nonperformance which he has himself occasioned, for the law says to
him in effect 'this is your own act, and therefore you are not damnified. [46]

In the Suyoc case, this Court expressly conceded that a mere request for
reconsideration or reinvestigation of an assessment may not suspend the
running of the statute of limitations. It affirmed the need for a waiver of the
prescriptive period in order to effect suspension thereof. However, even without
such waiver, the taxpayer may be estopped from raising the defense of
prescription because by his repeated requests or positive acts, he had induced
Government authorities to delay collection of the assessed tax.
107

Based on the foregoing, petitioner BPI contends that the declaration made in
the later case of Wyeth Suaco, that the statute of limitations on collection is
suspended once the taxpayer files a request for reconsideration or
reinvestigation, runs counter to the ruling made by this Court in the Suyoc case.

B. Although this Court is not compelled to abandon its decision in the Wyeth
Suaco case, it finds that Wyeth Suaco is not applicable to the Petition at
bar because of the distinct facts involved herein.

In the case of Wyeth Suaco, taxpayer Wyeth Suaco was assessed for failing to
remit withholding taxes on royalties and dividend declarations, as well as, for
deficiency sales tax. The BIR issued two assessments, dated 16 December 1974
and 17 December 1974, both received by taxpayer Wyeth Suaco on 19
December 1974. Taxpayer Wyeth Suaco, through its tax consultant, SGV & Co.,
sent to the BIR two letters, dated 17 January 1975 and 08 February 1975,
protesting the assessments and requesting their cancellation or withdrawal on
the ground that said assessments lacked factual or legal basis. On 12
September 1975, the BIR Commissioner advised taxpayer Wyeth Suaco to avail
itself of the compromise settlement being offered under Letter of Instruction No.
308. Taxpayer Wyeth Suaco manifested its conformity to paying a compromise
amount, but subject to certain conditions; though, apparently, the said
compromise amount was never paid. On 10 December 1979, the BIR
Commissioner rendered a decision reducing the assessment for deficiency
withholding tax against taxpayer Wyeth Suaco, but maintaining the assessment
for deficiency sales tax. It was at this point when taxpayer Wyeth Suaco
brought its case before the CTA to enjoin the BIR from enforcing the
assessments by reason of prescription. Although the CTA decided in favor of
taxpayer Wyeth Suaco, it was reversed by this Court when the case was
brought before it on appeal. According to the decision of this Court '

Settled is the rule that the prescriptive period provided by law to


make a collection by distraint or levy or by a proceeding in court is
interrupted once a taxpayer requests for reinvestigation or
reconsideration of the assessment. . .
...

Although the protest letters prepared by SGV & Co. in behalf of


private respondent did not categorically state or use the words
reinvestigation and 'reconsideration, the same are to be treated as
letters of reinvestigation and reconsideration
108

These letters of Wyeth Suaco interrupted the running of the five-year


prescriptive period to collect the deficiency taxes. The Bureau of
Internal Revenue, after having reviewed the records of Wyeth Suaco,
in accordance with its request for reinvestigation, rendered a final
assessment It was only upon receipt by Wyeth Suaco of this final
assessment that the five-year prescriptive period started to run
again. [47]

The foremost criticism of petitioner BPI of the Wyeth Suaco decision is directed
at the statement made therein that, 'settled is the rule that the prescriptive
period provided by law to make a collection by distraint or levy or by a
proceeding in court is interrupted once a taxpayer requests for reinvestigation
or reconsideration of the assessment. [48] It would seem that both petitioner BPI
and respondent BIR Commissioner, as well as, the CTA and Court of Appeals,
take the statement to mean that the filing alone of the request for
reconsideration or reinvestigation can already interrupt or suspend the running
of the prescriptive period on collection. This Court therefore takes this
opportunity to clarify and qualify this statement made in the Wyeth
Suaco case. While it is true that, by itself, such statement would appear to be a
generalization of the exceptions to the statute of limitations on collection, it is
best interpreted in consideration of the particular facts of the Wyeth
Suaco case and previous jurisprudence.

The Wyeth Suaco case cannot be in conflict with the Suyoc case because
there are substantial differences in the factual backgrounds of the two cases.
The Suyoc case refers to a situation where there were repeated requests or
positive acts performed by the taxpayer that convinced the BIR to delay
collection of the assessed tax. This Court pronounced therein that the repeated
requests or positive acts of the taxpayer prevented or estopped it from setting
up the defense of prescription against the Government when the latter
attempted to collect the assessed tax. In the Wyeth Suaco case, taxpayer
Wyeth Suaco filed a request for reinvestigation, which was apparently granted
by the BIR and, consequently, the prescriptive period was indeed suspended as
provided under Section 224 of the Tax Code of 1977, as amended. [49]

To reiterate, Section 224 of the Tax Code of 1977, as amended, identifies


specific circumstances when the statute of limitations on assessment and
collection may be interrupted or suspended, among which is a request for
reinvestigation that is granted by the BIR Commissioner. The act of filing a
request for reinvestigation alone does not suspend the period; such request
must be granted. [50] The grant need not be express, but may be implied from
109

the acts of the BIR Commissioner or authorized BIR officials in response to the
request for reinvestigation. [51]

This Court found in the Wyeth Suaco case that the BIR actually conducted a
reinvestigation, in accordance with the request of the taxpayer Wyeth Suaco,
which resulted in the reduction of the assessment originally issued against it.
Taxpayer Wyeth Suaco was also aware that its request for reinvestigation was
granted, as written by its Finance Manager in a letter dated 01 July 1975,
addressed to the Chief of the Tax Accounts Division, wherein he admitted that,
'[a]s we understand, the matter is now undergoing review and consideration by
your Manufacturing Audit Division The statute of limitations on collection, then,
started to run only upon the issuance and release of the reduced assessment.

The Wyeth Suaco case, therefore, is correct in declaring that the prescriptive
period for collection is interrupted or suspended when the taxpayer files a
request for reinvestigation, provided that, as clarified and qualified herein, such
request is granted by the BIR Commissioner.

Thus, this Court finds no compelling reason to abandon its decision in the Wyeth
Suaco case. It also now rules that the said case is not applicable to the Petition
at bar because of the distinct facts involved herein. As already heretofore
determined by this Court, the protest filed by petitioner BPI was a request for
reconsideration, which merely required a review of existing evidence and the
legal basis for the assessment. Respondent BIR Commissioner did not require,
neither did petitioner BPI offer, additional evidence on the matter. After
petitioner BPI filed its request for reconsideration, there was no other
communication between it and respondent BIR Commissioner or any of the
authorized representatives of the latter. There was no showing that petitioner
BPI was informed or aware that its request for reconsideration was granted or
acted upon by the BIR.

IV

Conclusion

To summarize all the foregoing discussion, this Court lays down the following
rules on the exceptions to the statute of limitations on collection.

The statute of limitations on collection may only be interrupted or suspended by


a valid waiver executed in accordance with paragraph (d) of Section 223 of
the Tax Code of 1977, as amended, and the existence of the circumstances
110

enumerated in Section 224 of the same Code, which include a request for
reinvestigation granted by the BIR Commissioner.

Even when the request for reconsideration or reinvestigation is not


accompanied by a valid waiver or there is no request for reinvestigation that
had been granted by the BIR Commissioner, the taxpayer may still be held in
estoppel and be prevented from setting up the defense of prescription of the
statute of limitations on collection when, by his own repeated requests or
positive acts, the Government had been, for good reasons, persuaded to
postpone collection to make the taxpayer feel that the demand is not
unreasonable or that no harassment or injustice is meant by the Government,
as laid down by this Court in the Suyoc case.

Applying the given rules to the present Petition, this Court finds that '

(a) The statute of limitations for collection of the deficiency DST in Assessment
No. FAS-5-85-89-002054, issued against petitioner BPI, had already expired; and

(b) None of the conditions and requirements for exception from the statute of
limitations on collection exists herein: Petitioner BPI did not execute any waiver
of the prescriptive period on collection as mandated by paragraph (d) of
Section 223 of the Tax Code of 1977, as amended; the protest filed by
petitioner BPI was a request for reconsideration, not a request for
reinvestigation that was granted by respondent BIR Commissioner which could
have suspended the prescriptive period for collection under Section 224 of the
Tax Code of 1977, as amended; and, petitioner BPI, other than filing a request
for reconsideration of Assessment No. FAS-5-85-89-002054, did not make
repeated requests or performed positive acts that could have persuaded the
respondent BIR Commissioner to delay collection, and that would have
prevented or estopped petitioner BPI from setting up the defense of
prescription against collection of the tax assessed, as required in
the Suyoc case.

This is a simple case wherein respondent BIR Commissioner and other BIR
officials failed to act promptly in resolving and denying the request for
reconsideration filed by petitioner BPI and in enforcing collection on the
assessment. They presented no reason or explanation as to why it took them
almost eight years to address the protest of petitioner BPI. The statute on
limitations imposed by the Tax Code precisely intends to protect the taxpayer
from such prolonged and unreasonable assessment and investigation by the
BIR.

Considering that the right of the respondent BIR Commissioner to collect from
petitioner BPI the deficiency DST in Assessment No. FAS-5-85-89-002054 had
already prescribed, then, there is no more need for this Court to make a
111

determination on the validity and correctness of the said Assessment for the
latter would only be unenforceable.

WHEREFORE, BASED ON THE FOREGOING, THE INSTANT PETITION IS GRANTED. THE


DECISION OF THE COURT OF APPEALS IN CA-G.R. SP NO. 51271, DATED 11
AUGUST 1999, WHICH REINSTATED ASSESSMENT NO. FAS-5-85-89-002054
REQUIRING PETITIONER BPI TO PAY THE AMOUNT OF P28,020.00 AS DEFICIENCY
DOCUMENTARY STAMP TAX FOR THE TAXABLE YEAR 1985, INCLUSIVE OF THE
COMPROMISE PENALTY, IS REVERSED AND SET ASIDE. ASSESSMENT NO. FAS-5-85-
89-002054 IS HEREBY ORDERED CANCELED.

SO ORDERED.

G.R. No. L-19727 May 20, 1965

THE COMMISSIONER OF INTERNAL REVENUE, petitioner,


vs.
PHOENIX ASSURANCE CO., LTD., respondent.

-----------------------------

G.R. No. L-19903 May 20, 1965

PHOENIX ASSURANCE, CO., LTD., petitioner,


vs.
COMMISSIONER OF INTERNAL REVENUE, respondent.

Office of the Solicitor General for petitioner-respondent Commissioner of


Internal Revenue.
Sycip, Salazar, Luna & Associates and A. S. Monzon, B. V. Abela & J. M. Castillo
for respondent-petitioner Phoenix Assurance Co., Ltd.

BENGZON, J.P., J.:

From a judgment of the Court of Tax Appeals in C.T.A. Cases Nos. 305 and 543,
consolidated and jointly heard therein, these two appeals were taken. Since
they involve the same facts and interrelated issues, the appeals are herein
decided together.

Phoenix Assurance Co., Ltd., a foreign insurance corporation organized under


the laws of Great Britain, is licensed to do business in the Philippines with head
office in London. Through its head office, it entered in London into worldwide
reinsurance treaties with various foreign insurance companies. It agree to cede
a portion of premiums received on original insurances underwritten by its head
office, subsidiaries, and branch offices throughout the world, in consideration
for assumption by the foreign insurance companies of an equivalent portion of
the liability from such original insurances.1äwphï1.ñët
112

Pursuant to such reinsurance treaties, Phoenix Assurance Co., Ltd., ceded


portions of the premiums it earned from its underwriting business in the
Philippines, as follows:

Year Amount Ceded


1952 P316,526.75
1953 P246,082.04
1954 P203,384.69

upon which the Commissioner of Internal Revenue, by letter of May 6, 1958,


assessed the following withholding tax:

Year Withholding Tax


1952 P 75,966.42
1953 59,059.68
1954 48,812.32

Total P183,838.42
=============

On April 1, 1951, Phoenix Assurance Co., Ltd. filed its Philippine income tax
return for 1950, claiming therein, among others, a deduction of P37,147.04 as
net addition to marine insurance reserve equivalent to 40% of the gross marine
insurance premiums received during the year. The Commissioner of Internal
Revenue disallowed P11,772.57 of such claim for deduction and subsequently
assessed against Phoenix Assurance Co., Ltd. the sum of P1,884.00 as
deficiency income tax. The disallowance resulted from the fixing by the
Commissioner of the net addition to the marine insurance reserve at 100% of
the marine insurance premiums received during the last three months of the
year. The Commissioner assumed that "ninety and third, days are approximately
the length of time required before shipments reach their destination or before
claims are received by the insurance companies."

On April 1, 1953, Phoenix Assurance Co., Ltd. filed its Philippine income tax
return for 1952, declaring therein a deduction from gross income of P35,912.25
as part of the head office expenses incurred for its Philippine business,
computed at 5% on its gross Philippine income.

On August 30, 1955 it amended its income tax return for 1952 by excluding from
its gross income the amount of P316,526.75 representing reinsurance premiums
ceded to foreign reinsurers and further eliminating deductions corresponding to
the coded premiums. The amended return showed an income tax due in the
amount of P2,502.00. The Commissioner of Internal Revenue disallowed
P15,826.35 of the claimed deduction for head office expenses and assessed a
deficiency tax of P5,667.00 on July 24, 1958.
113

On April 30, 1954, Phoenix Assurance Co., Ltd. filed its Philippine income tax
return for 1953 and claimed therein a deduction from gross income of
P33,070.88 as head office expenses allocable to its Philippine business,
equivalent to 5%, of its gross Philippine income. On August 30, 1955 it amended
its 1953 income tax return to exclude from its gross income the amount of
P246,082.04 representing reinsurance premiums ceded to foreign reinsurers. At
the same time, it requested the refund of P23,409.00 as overpaid income tax for
1953. To avoid the prescriptive period provided for in Section 306 of the Tax
Code, it filed a petition for review on April 11, 1956 in the Court of Tax Appeals
praying for such refund. After verification of the amended income tax return
the Commissioner of Internal Revenue disallowed P12,304.10 of the deduction
representing head office expenses allocable to Philippine business thereby
reducing the refundable amount to P20,180.00.

On April 29, 1955, Phoenix Assurance Co., Ltd. filed its Philippine income tax
return for 1954 claiming therein, among others, a deduction from gross income
of P99,624.75 as head office expenses allocable to its Philippine business,
computed at 5% of its gross Philippine income. It also excluded from its gross
income the amount of P203,384.69 representing reinsurance premiums ceded
to foreign reinsurers not doing business in the Philippines.

On August 1, 1958 the Bureau of Internal Revenue released the following


assessment for deficiency income tax for the years 1952 and 1954 against
Phoenix Assurance Co., Ltd.:

1952
Net income per audited
P 12,511.61
return
Unallowable deduction & additional
income:
Overclaimed Head Office expenses:
Amount
P
claimed . . . . . .
35,912.25
......
Amount
allowed . . . . . . 20,085.90 P 15,826.35
......

Net income per


P 28,337.96
investigation

Tax due thereon P 5,667.00


===========
1954
Net income per audited P160,320.21
Unallowable deduction & additional
114

income:
Overclaimed Head Office expenses:
Amount
claimed . . . . . . P29,624.73
......
Amount
allowed . . . . . . 19,455.50 10,16.23
......

Net income per


P170,489.41
investigation

Tax due thereon P 39,737.00


Less: amount already
36,890.00
assessed

DEFICIENCY TAX DUE P 2,847.00


===========

The above assessment resulted from the disallowance of a portion of the


deduction claimed by Phoenix Assurance Co., Ltd. as head office expenses
allocable to its business in the Philippines fixed by the Commissioner at 5% of the
net Philippine income instead of 5% of the gross Philippine income as claimed in
the returns.

Phoenix Assurance Co., Ltd. protested against the aforesaid assessments for
withholding tax and deficiency income tax. However, the Commissioner of
Internal Revenue denied such protest. Subsequently, Phoenix Assurance Co.,
Ltd. appealed to the Court of Tax Appeals. In a decision dated February 14,
1962, the Court of Tax Appeals allowed in full the decision claimed by Phoenix
Assurance Co., Ltd. for 1950 as net addition to marine insurance reserve;
determined the allowable head office expenses allocable to Philippine
business to be 5% of the net income in the Philippines; declared the right of the
Commissioner of Internal Revenue to assess deficiency income tax for 1952 to
have prescribed; absolved Phoenix Assurance Co., Ltd. from payment of the
statutory penalties for non-filing of withholding tax return; and, rendered the
following judgment:

WHEREFORE, petitioner Phoenix Assurance Company, Ltd. is hereby


ordered to pay the Commissioner of Internal Revenue the respective
amounts of P75,966.42, P59,059.68 and P48,812.32, as withholding tax for
the years 1952, 1953 and 1954, and P2,847.00 as income tax for 1954, or
the total sum of P186,685.42 within thirty (30) days from the date this
decision becomes final. Upon the other hand, the respondent
Commissioner is ordered to refund to petitioner the sum of P20,180.00 as
overpaid income tax for 1953, which sum is to be deducted from the total
sum of P186,685.42 due as taxes.
115

If any amount of the tax is not paid within the time prescribed above,
there shall be collected a surcharge of 5% of the tax unpaid, plus interest
at the rate of 1% a month from the date of delinquency to the date of
payment, provided that the maximum amount that may be collected as
interest shall not exceed the amount corresponding to a period of three
(3) years. Without pronouncement as to costs.

Phoenix Assurance Co., Ltd. and the Commissioner of Internal Revenue have
appealed to this Court raising the following issues: (1) Whether or not
reinsurance premiums ceded to foreign reinsurers not doing business in the
Philippines pursuant to reinsurance contracts executed abroad are subject to
withholding tax; (2) Whether or not the right of the Commissioner of Internal
Revenue to assess deficiency income tax for the year 1952 against Phoenix
Assurance Co., Ltd., has prescribed; (3) Whether or not the deduction of
claimed by the Phoenix Assurance Co., Ltd.as net addition to reserve for the
year 1950 is excessive; (4) Whether or not the deductions claimed by Phoenix
Assurance Co., Ltd. for head office expenses allocable to Philippine business for
the years 1952, 1953 and 1954 are excessive.

The question of whether or not reinsurance premiums ceded to foreign


reinsurers not doing business in the Philippines pursuant to contracts executed
abroad are income from sources within the Philippines subject to withholding
tax under Sections 53 and 54 of the Tax Code has already been resolved in the
affirmative in British Traders' Insurance Co., Ltd.v. Commisioner of Internal
Revenue, L-20501, April 30, 1965. 1

We come to the issue of prescription. Phoenix Assurance Co., Ltd. filed its
income tax return for 1952 on April 1, 1953 showing a loss of P199,583.93. It
amended said return on August 30, 1955 reporting a tax liability of P2,502.00. On
July 24, 1958, after examination of the amended return, the Commissioner of
Internal Revenue assessed deficiency income tax in the sum of P5,667.00. The
Court of Tax Appeals found the right of the Commissioner of Internal Revenue
barred by prescription, the same having been exercised more than five years
from the date the original return was filed. On the other hand, the
Commissioner of Internal Revenue insists that his right to issue the assessment
has not prescribed inasmuch as the same was availed of before the 5-year
period provided for in Section 331 of the Tax Code expired, counting the
running of the period from August 30, 1955, the date when the amended return
was filed.

Section 331 of the Tax Code, which limits the right of the Commissioner of
Internal Revenue to assess income tax within five years from the Filipino of the
income tax return, states:

SEC. 331. Period of limitation upon assessment and collection. — Except as


provided in the succeeding section internal revenue taxes shall be
assessed within five years after the return was filed, and no proceeding in
court without assessment for the collection of such taxes shall be begun
after the expiration of such period. For the purposes of this section, a
return filed before the last day prescribed by law for the filing thereof shall
116

be considered as filed on such last day: Provided, That this limitation shall
not apply to cases already investigated prior to the approval of this Code.

The question is: Should the running of the prescriptive period commence from
the filing of the original or amended return?

The Court of Tax Appears that the original return was a complete return
containing "information on various items of income and deduction from which
respondent may intelligently compute and determine the tax liability of
petitioner, hence, the prescriptive period should be counted from the filing of
said original return. On the other hand, the Commissioner of Internal Revenue
maintains that:

"... the deficiency income tax in question could not possibly be


determined, or assessed, on the basis of the original return filed on April 1,
1953, for considering that the declared loss amounted to P199,583.93, the
mere disallowance of part of the head office expenses could not
probably result in said loss being completely wiped out and Phoenix being
liable to deficiency tax. Not until the amended return was filed on August
30, 1955 could the Commissioner assess the deficiency income tax in
question."

Accordingly, he would wish to press for the counting of the prescriptive period
from the filing of the amended return.

To our mind, the Commissioner's view should be sustained. The changes and
alterations embodied in the amended income tax return consisted of the
exclusion of reinsurance premiums received from domestic insurance
companies by Phoenix Assurance Co., Ltd.'s London head office, reinsurance
premiums ceded to foreign reinsurers not doing business in the Philippines and
various items of deduction attributable to such excluded reinsurance premiums
thereby substantially modifying the original return. Furthermore, although the
deduction for head office expenses allocable to Philippine business, whose
disallowance gave rise to the deficiency tax, was claimed also in the original
return, the Commissioner could not have possibly determined a deficiency tax
thereunder because Phoenix Assurance Co., Ltd. declared a loss of P199,583.93
therein which would have more than offset such disallowance of P15,826.35.
Considering that the deficiency assessment was based on the amended return
which, as aforestated, is substantially different from the original return, the
period of limitation of the right to issue the same should be counted from the
filing of the amended income tax return. From August 30, 1955, when the
amended return was filed, to July 24, 1958, when the deficiency assessment
was issued, less than five years elapsed. The right of the Commissioner to assess
the deficiency tax on such amended return has not prescribed.

To strengthen our opinion, we believe that to hold otherwise, we would be


paving the way for taxpayers to evade the payment of taxes by simply
reporting in their original return heavy losses and amending the same more
than five years later when the Commissioner of Internal Revenue has lost his
authority to assess the proper tax thereunder. The object of the Tax Code is to
117

impose taxes for the needs of the Government, not to enhance tax avoidance
to its prejudice.

We next consider Phoenix Assurance Co., Ltd.'s claim for deduction of


P37,147.04 for 1950 representing net addition to reserve computed at 40% of
the marine insurance premiums received during the year. Treating said said
deduction to be excessive, the Commissioner of Internal Revenue reduced the
same to P25,374.47 which is equivalent to 100% of all marine insurance
premiums received during the last months of the year.

Paragraph (a) of Section 32 of the Tax Code states:

SEC. 32. Special provisions regarding income and deductions of insurance


companies, whether domestic or foreign. — (a) Special deductions
allowed to insurance companies. — In the case of insurance companies,
except domestic life insurance companies and foreign life insurance
companies doing business in the Philippines, the net additions, if any,
required by law to be made within the year to reserve funds and the sums
other than dividends paid within the year on policy and annuity contracts
may be deducted from their gross income: Provided, however, That the
released reserve be treated as income for the year of release.

Section 186 of the Insurance Law requires the setting up of reserves for liability
on marine insurance:

SEC. 186. ... Provided, That for marine risks the insuring company shall be
required to charge as the liability for reinsurance fifty per centum of the
premiums written in the policies upon yearly risks, and the full
premiums written in the policies upon all other marine risks not
terminated (Emphasis supplied.)

The reserve required for marine insurance is determined on two bases: 50% of
premiums under policies on yearly risks and 100% of premiums under policies of
marine risks not terminated during the year. Section 32 (a) of the Tax Code
quoted above allows the full amount of such reserve to be deducted from
gross income.

It may be noteworthy to observe that the formulas for determining the marine
reserve employed by Phoenix Assurance Co., Ltd. and the Commissioner of
Internal Revenue — 40% of premiums received during the year and 100% of
premiums received during the last three months of the year, respectively — do
not comply with Section 186. Said determination runs short of the requirement.
For purposes of the Insurance Law, this Court therefore cannot countenance
the same. The reserve called for in Section 186 is a safeguard to the general
public and should be strictly followed not only because it is an express provision
but also as a matter of public policy. However, for income tax purposes a
taxpayer is free to deduct from its gross income a lesser amount, or not to claim
any deduction at all. What is prohibited by the income tax law is to claim a
deduction beyond the amount authorized therein.
118

Phoenix Assurance Co., Ltd.'s claim for deduction of P37,147.04 being less than
the amount required in Section 186 of the Insurance Law, the same cannot be
and is not excessive, and should therefore be fully allowed. *

We come now to the controversy on the taxpayer's claim for deduction on


head office expenses incurred during 1952, 1953, and 1954 allocable to its
Philippine business computed at 5% of its gross income in the Philippines The
Commissioner of Internal Revenue redetermined such deduction at 5% on
Phoenix Assurance Co., Ltd's net income thereby partially disallowing the
latter's claim. The parties are agreed as to the percentage — 5% — but differ as
to the basis of computation. Phoenix Assurance Co. Lt. insists that the 5% head
office expenses be determined from the gross income, while the Commissioner
wants the computation to be made on the net income. What, therefore, needs
to be resolved is: Should the 5% be computed on the gross or net income?

The record shows that the gross income of Phoenix Assurance Co., Ltd. consists
of income from its Philippine business as well as reinsurance premiums received
for its head office in London and reinsurance premiums ceded to foreign
reinsurance. Since the items of income not belonging to its Philippine business
are not taxable to its Philippine branch, they should be excluded in determining
the head office expenses allowable to said Philippine branch. This conclusion
finds support in paragraph 2, subsection (a), Section 30 of the Tax Code,
quoted hereunder:

(2) Expenses allowable to non-resident alien individuals and foreign


corporations. In the case of a non-resident alien individual or a foreign
corporation, the expenses deductible are the, necessary expenses paid
or incurred in carrying on any business or trade conducted within the
Philippines exclusively. (Emphasis supplied.)

Consequently, the deficiency assessments for 1952, 1953 and 1954, resulting
from partial disallowance of deduction representing head office expenses, are
sustained.

Finally, the Commissioner of Internal Revenue assails the dispositive portion of


the Tax Court's decision limiting the maximum amount of interest collectible for
deliquency of an amount corresponding to a period of three years. He
contends that since such limitation was incorporated into Section 51 of the Tax
Code by Republic Act 2343 which took effect only on June 20, 1959, it must not
be applied retroactively on withholding tax for the years 1952, 1953 and 1954.

The imposition of interest on unpaid taxes is one of the statutory penalties for
tax delinquency, from the payments of which the Court of Tax Appeals
absolved the Phoenix Assurance Co., Ltd. on the equitable ground that the
latter's failure to pay the withholding tax was due to the Commissioner's opinion
that no withholding tax was due. Consequently, the taxpayer could be held
liable for the payment of statutory penalties only upon its failure to comply with
the Tax Court's judgment rendered on February 14. 1962, after Republic Act
2343 took effect. This part of the ruling of the lower court ought not to be
disturbed.
119

WHEREFORE, the decision appealed from is modified, Phoenix Assurance Co.,


Ltd. is hereby ordered to pay the Commissioner, of Internal Revenue the
amount of P75,966.42, P59,059.68 and P48,812.32 as withholding tax for the
years 1952, 1953 and 1954, respectively, and the sums of P5,667.00 and
P2,847.00 as income tax for 1952 and 1954 or a total of P192,352.42. The
Commissioner of Internal Revenue is ordered to refund to Phoenix Assurance
Co., Ltd. the amount of P20,180.00 as overpaid income tax for 1953, which
should be deducted from the amount of P192,352.42.

If the amount of P192,352.42 or a portion thereof is not paid within thirty (30)
days from the date this judgment becomes final, there should be collected a
surcharge and interest as provided for in Section 51(c) (2) of the Tax Code. No
costs. It is so ordered.

G.R. No. 76281 September 30, 1991

COMMISSIONER OF INTERNAL REVENUE, petitioner,


vs.
WYETH SUACO LABORATORIES, INC. and THE COURT OF TAX
APPEALS, respondents.

FERNAN, C.J.:

The sole issue in this petition for review on certiorari is whether or not petitioner's
right to collect deficiency withholding tax at source and sales tax liabilities from
private respondent is barred by prescription.

The antecedent facts are as follows:

Private respondent Wyeth Suaco Laboratories, Inc. (Wyeth Suaco for brevity) is
a domestic corporation engaged in the manufacture and sale of assorted
pharmaceutical and nutritional products. Its accounting period is on a fiscal
year basis ending October 31 of every year.

By virtue of Letter of Authority No. 52415 dated June 17, 1974 issued by then
Commissioner of Internal Revenue Misael P. Vera, Revenue Examiner Dante
Kabigting conducted an investigation and examination of the books of
accounts of Wyeth Suaco.1 On October 15, 1974, he submitted a report
containing the result of his investigation. The report disclosed that Wyeth Suaco
was paying royalties to its foreign licensors as well as remuneration for technical
services to Wyeth International Laboratories of London. Wyeth Suaco was also
found to have declared cash dividends on September 27, 1973 and these were
paid on October 31, 1973. However, it allegedly failed to remit withholding tax
at source for the fourth (4th) quarter of 1973 on accrued royalties, remuneration
for technical services and cash dividends, resulting in a deficiency withholding
tax at source in the aggregate amount of P3,178,994.15.2

Moreover, it was reported that during the periods from November 1, 1972 to
December 31, 1972 and January 1, 1973 to October 31, 1973, Wyeth Suaco
120

deducted the cost of non-deductible raw materials, resulting in its alleged


failure to pay the correct amount of advance sales tax. There was reportedly
also a short payment of advance sales tax in its importation of "Mega
Polymycin D" on October 3, 1972. All these resulted in a deficiency sales tax in
the amount of P60,855.21 and compromise penalty in the amount of P300.00 or
a total amount of P61,155.21.3

Consequently, the Bureau of Internal Revenue assessed Wyeth Suaco on the


aforesaid tax liabilities in two (2) notices dated December 16, 1974 and
December 17, 1974. These assessment notices were both received by Wyeth
Suaco on December 19, 1974.4

Thereafter, Wyeth Suaco through its tax consultant SGV &Co., sent the Bureau
of Intemal Revenue two (2) letters dated January 17, 1975 and February 8,
1975, protesting the assessments and requesting their cancellation or
withdrawal on the ground that said assessments lacked factual or legal basis.

Wyeth Suaco argued that it was not liable to pay withholding tax at source on
the accrued royalties and dividends because they have yet to be remitted or
paid abroad. It claimed that it was not able to remit the balance of fifty
percent (50%) of the accrued royalties to its foreign licensors because of
Central Bank Circular No. 289 allowing remittance of royalties up to fifty percent
(50%) only. With regard to what the Bureau of Internal Revenue claimed as the
amount of P2,952,391.00 forming part of the cash dividends declared in 1973,
Wyeth Suaco alleged that the same was due its foreign stockholders. Again,
Wyeth Suaco was not able to remit these dividends because of the restriction
of the Central Bank in a memorandum implementing CB Circular No. 289 dated
February 21, 1970. Thus, Wyeth Suaco's contention was that a withholding tax at
source on royalties and dividends becomes due and payable only upon their
actual payment or remittance.

On the matter of the withholding tax at source on remuneration for technical


services, Wyeth Suaco insisted that it was up-to-date in remitting the
corresponding withholding tax on this income to the Bureau of Internal
Revenue.

As to the assessed deficiency sales tax, Wyeth Suaco maintained that the
difference between its landed cost figure (which is the basis for computing the
advancesales tax) and that of the revenue examiner, was due to the use of
estimated amounts by the Bureau of Customs and to foreign exchange
differential.

Wyeth Suaco however, admitted liability with respect to the short payment of
advance sales tax in the amount of P1,000.00 on its importation of "Mega
Polymycin D."5

On September 12, 1975, the Commissioner of Internal Revenue asked Wyeth


Suaco to avail itself of the compromise settlement under LOI 308. In its answer,
Wyeth Suaco manifested its conformity to a 10% compromise provided it be
applied only to the basic sales tax, excluding surcharge and interest. As to the
deficiency withholding tax at source, Wyeth took exception on the ground that
121

it involves purely a legal question and some of the amounts included in the
assessment have already bee paid.

On December 10, 1979, petitioner, thru then acting Commissioner of Internal


Revenue Ruben B. Ancheta, rendered a decision reducing the assessment of
the withholding tax at source for 1973 to P1,973,112.86. However, the amount of
P61,155.21 as deficiency sales tax remained the same.6

Thereafter, Wyeth Suaco filed a petition for review in Court of Tax Appeals on
January 18, 1980, praying that lpeti tioner be enjoined from enforcing the
assessments by reason of prescription and that the assessments be declared
null and void for lack of legal and factual basis.7

On February 7, 1980, petitioner issued a warrant of distrain of personal property


and warrant of levy of real property again private respondent to enforce
collection of the deficiency taxes. These were served on private respondent on
March 12, 1980.8 However, collection of the deficiency taxes by virtue of
warrants of distraint and levy was enjoined by respondent court upon motion of
Wyeth Suaco in a resolution dated May 22, 1980.9

On May 30, 1980, petitioner filed his answer to Wyeth Suaco's petition for review
praying, among others, that private respondent be declared liable to pay the
amount of P61,155.21 as deficiency sales tax for the periods November 1, 1972
to December 31, 1972 and January 1, 1973 to October 31, 1973, plus 14%
annual interest thereon from December 17, 1974 until payment thereof
pursuant to Section 183 (now Section 193) of the Tax Code, and the amount of
P1,973,112.86 as deficie withholding tax at source for the 4th quarter of 1973
plus 5% surcharge and 14% per annum interest thereon from December 16,
1974 to December 16, 1977, pursuant to Section 51 (e) of the Tax Code of 1977,
as amended.10

On August 29, 1986, the Court of Tax Appeals rendered a decision enjoining the
Commissioner of Internal Revenue from collecting the deficiency taxes, the
dispositive portion of which reads as follows:

WHEREFORE, the decision appealed from is hereby reversed and


respondent Commissioner of Internal Revenue is hereby enjoined from
collecting the deficiency withholding tax at source for the fourth quarter
of 1973 as well as the deficiency sales tax assessed against petitioner
(Wyeth Suaco). Without pronouncement as to costs.11

The basis of the above decision was the finding of the Tax Court that while the
assessments for the deficiency taxes were made within the five-year period of
limitation, the right of petitioner to collect the same has already prescribed, in
accordance with Section 319 (c) of the Tax Code of 1977. The said law
provides that an assessment of any internal revenue tax within the five-year
period of limitation may be collected by distraint or levy or by a proceeding in
court, but only if begun within five (5) years after the assessment of the tax.

Hence, this recourse by petitioner.


122

The applicable laws in the instant case are Sections 318 and 319 (c) of the
National Internal Revenue Code of 1977 (now Sections 203 and 224 of the
National Internal Revenue Code of 1986), to wit:

SEC. 318. Period of limitation upon assessment and collection — Except as


provided in the succeeding section, internal revenue taxes shall be
assessed within five years after the return was filed, and no proceeding in
court without assessment for the collection of such taxes shall be begun
after the expiration of such period. ...

SEC. 319. Exceptions as to period of limitations of assessment and


collection of taxes. —

xxx xxx xxx

(c) Where the assessment of any internal revenue tax has been made
within the period of limitation above-prescribed such tax may be
collected by distraint or levy by a proceeding in court, but only if begun
(1) within five years after the assessment of the tax, or (2) prior the
expiration of any period for collection agreed upon in writing by the
Commissioner and the taxpayer before the expiration of such five-year
period. The period so agreed upon may be extended by subsequent
agreements in writing made before the expiration of the period previously
agreed upon. (emphasis supplied)

The main thrust of petitioner for the allowance of this petition is that the five-
year prescriptive period provided by law to mak a collection by distraint or levy
or by a proceeding in court has not yet prescribed. Although he admits that
more than five (5) years have already lapsed from the time the assessment
notices were received by private respondent on December 19, 1974 up to the
time the warrants of distraint and levy were served on March 12, 1980, he avers
that the running of the prescriptive period was stayed or interrupted when
Wyeth Suaco protested the assessments. Petitioner argues that the protest
letters sent by SGV & Co. in behalf of Wyeth Suaco dated January 17, 1975 and
February 8, 1975, requesting for withdrawal and cancellation of the assessments
were actually requests for reinvestigation or reconsideration, which could
interrupt the running of the five-year prescriptive period.

Wyeth Suaco, on the other hand, maintains the position that it never asked for
a reinvestigation nor reconsideration of th assessments. What it requested was
the cancellation and with drawal of the assessments for lack of legal and
factual basis. Thus, its protest letters dated January 17, 1975 and February 8,
1975 did not suspend or interrupt the running of the five-year prescriptive
period.

Settled is the rule that the prescriptive period provided by law to make a
collection by distraint or levy or by a proceeding in court is interrupted once a
taxpayer requests for reinvestigation or reconsideration of the assessment. In
the case of Commissioner of Internal Revenue vs. Capitol Subdivision, Inc.,12 this
Court held:
123

The period of prescription of action to collect a taxpayer's deficiency


income tax assessment is interrupted when the taxpayer request for a
review or reconsideration of said assessment, and starts to run again when
said request is denied.

In another case, this Court stated that the statutory period of limitation for
collection may be interrupted if by the taxpayer's repeated requests or positive
acts the Government has been, for good reasons, persuaded to postpone
collection to make him feel that the demand was not unreasonable or that no
harassment or injustice is meant by the Goverrument.13 Also in the case
of Cordero vs. Gonda,14 we held:

Partial payment would not prevent the government from suing the
taxpayer. Because, by such act of payment, the government is not
thereby "persuaded to postpone collection to make him feel that the
demand was not unreasonable or that no harassment or injustice is
meant." This is the underlying reason behind the rule that the prescriptive
period is arrested by the taxpayer's request for re-examination or
reinvestigation — even if he "has not previously waived it (prescription in
writing)". ... (emphasis supplied)

Thus, the pivotal issue in this case is whether or not Wyeth Suaco sought
reinvestigation or reconsideration of the deficiency tax assessments issued by
the Bureau of Internal Revenue.

After carefully examining the records of the case, we find that Wyeth Suaco
admitted that it was seeking reconsideration of the tax assessments as shown in
a letter of James A. Gump, its President and General Manager, dated April 28,
1975, the relevant portion of which is quoted hereunder, to wit:

We submit this letter as a follow-up to our protest filed with your office,
through our tax advisers, Sycip, Gorres, Velayo & Co., on January 20 and
February 10, 1975 regarding alleged deficiency on withholding tax at
source of P3,178,994.15 and on percentage tax of P60,855.21, including
interest and surcharges, on which we are seeking
reconsideration.15 (emphasis supplied)

Furthermore, when Wyeth Suaco thru its tax consultant SGV & Co. sent the
letters protesting the assessments, the Bureau of Internal Revenue,
Manufacturing Audit Division, conducted a review and reinvestigation of the
assessments. This fact was admitted by Wyeth Suaco thru its Finance Manager
in a letter dated July 1, 1975 addressed to the Chief, Tax Accounts Division. The
pertinent portion of said letter reads as follows:

This will acknowledge receipt of your letter dated May 22, 1975 regarding
our alleged income and business tax deficiencies for fiscal year 1972/73.

xxx xxx xxx


124

Nevertheless, please be advised that the deficiency tax stated in your


letter is what we are protesting on pursuant to the letters we filed with the
Bureau of Internal Revenue on January 20, 1975 and on February 10, 1975.

xxx xxx xxx

As we understand, the matter is now undergoing review and


consideration by your Manufacturing Audit Division. Pending the outcome
of their decision, we regret our inability to make settlement. ...16 (Emphasis
supplied)

Although the protest letters prepared by SGV & Co. in behalf of private
respondent did not categorically state or use th words "reinvestigation" and
"reconsideration," the same are to be treated as letters of reinvestigation and
reconsideration. By virtue of these letters, the Bureau of Internal Revenue
ordered its Manufacturing Audit Division to review the assessment made.
Furthermore, private respondent's claim that it did not seek reinvestigation or
reconsideration of the assessments is belied by the subsequent
correspondence or letters written by its officers, as shown above.

These letters of Wyeth Suaco interrupted the running of the five-year


prescriptive period to collect the deficiency taxes. The Bureau of Internal
Revenue, after having reviewed the record of Wyeth Suaco, in accordance
with its request for reinvestigation, rendered a final assessment. This final
assessment issue by then Acting Commissioner Ruben B. Ancheta was date
December 10, 1979 and received by private respondent on January 2, 1980,
fixed its tax liability at P1,973,112.86 as deficiency withholding tax at source and
P61,155.21 as deficiency sales tax. It was only upon receipt by Wyeth Suaco of
this final assessment that the five-year prescriptive period started to run again.

Verily, the original assessments dated December 16 and 17, 1974 were both
received by Wyeth Suaco on December 19, 1974. However, when Wyeth
Suaco protested the assessments and sought its reconsideration in two (2)
letters received by the Bureau of Internal Revenue on January 20 and February
10, 1975, the prescriptive period was interrupted. This period started to run
again when the Bureau of Internal Revenue served the final assessment to
Wyeth Suaco on January 2, 1980. Since the warrants of distraint and levy were
served on Wyeth Suaco on March 12, 1980, then, only about four (4) months of
the five-year prescriptive period was used.

Having resolved the issue of prescription, we now come to the merits of the
case.

Wyeth Suaco questions the legality of the regulation imposed by the Bureau of
Intemal Revenue of requiring a withholding agent or taxpayer to remit the
taxes deducted and withheld at source on incomes which have not yet been
paid. It maintains the stand that withholding tax at source should only be
remitted to the Bureau of Internal Revenue once the incomes subject to
withholding tax at source have actually been paid. Thus, private respondent
avers that it was not liable to remit the taxes withheld at source on royalties and
125

dividends unless these incomes have been actually paid to its foreign licensors
and stockholders.

It is said that taxes are what we pay for civilized society. Without taxes, the
government would be paralyzed for lack of the motive power to activate and
operate it. ... It is the lifeblood of the government and so should be collected
without unnecessary hindrance ...17

In line with this principle, the Tax Code, particularly Section 54 (a) [now Section
51 (a)] provides that "the Commissioner of Internal Revenue may, with the
approval of the Secretary of Finance, require the withholding agents to pay or
deposit the taxes deducted and withheld at more frequent intervals when
necessary to protect the interest of the government. The return shall be filed
and the payment made within 25 days from the close of each calendar
quarter". Presently, Revenue Regulation No. 6-85 effective July 1, 1985, requires
the filing of monthly return and payment of taxes withheld at source within (10)
days after the end of each month.

Moreover, the records show that Wyeth Suaco adopted the accrual method of
accounting wherein the effect of transactions and other events on assets and
liabilities are recognized and reported in the time periods to which they relate
rather than only when cash is received or paid. The "Report of Investigation"
submitted by the tax examiner indicated that accrual was the basis of the
taxpayer's return.18 Thus, private respondent recorded accrued royalties and
dividends payable as well as the withholding tax at source payable on these
incomes. Having deducted and withheld the tax at source and having
recorded the withholding tax at source payable in its books of accounts,
private respondent was obligated to remit the same to the Bureau of Internal
Revenue.

With regard to the accuracy of the assessment on deficiency sales tax, we rule
that the examiner's assessment should be given full weight and credit, in the
absence of proof submitted by Wyeth Suaco to the contrary. This is in line with
our ruling in several cases wherein we said that tax assessments by tax
examiners are presumed correct and made in good faith. The taxpayer has the
duty to prove otherwise. In the absence of proof of any irregularities in the
performance of duties, an assessment duly made by a Bureau of Internal
Revenue examiner and approved by his superior officers will not be disturbed.
All presumptions are in favor of the correctness of tax assessments.19 The case
of Commissioner of Internal Revenue vs. Construction Resources of Asia,
Inc.,20 where this Court cited 51 Am. Jur. pp. 620-621, states the principle in
detail, thus:

All presumptions are in favor of the correctness of tax assessments. The


good faith of tax assessors and the validity of their actions are presumed.
They will be presumed to have taken into consideration all the facts to
which their attention was called. No presumption can be indulged that all
of the public officials of the State in the various counties who have to do
with the assessment of property for taxation will knowingly violate the
duties imposed upon them by law.
126

The final assessment issued by the Bureau of Internal Revenue declared the
issuance of deficiency sales tax assessments to be legal and valid. It was
ascertained that during the investigation, Wyeth Suaco deducted non-
deductible raw materials which were not subjected to advance sales tax
thereby resulting in its failure to pay the correct amount of sales tax under
Section 183, in relation to Section 186 and 186-B of the Tax Code, prior to and
after amendment by Presidential Decree No. 69. Wyeth Suaco was not able to
refute this by submitting supporting documents.21

WHEREFORE, the petition is GRANTED. Wyeth Suaco Laboratories, Inc, is hereby


ordered to pay the Bureau of Internal Revenue the amount of P1,973,112.86 as
deficiency withholding tax at source, with interest and surcharge in
accordance with law, without prejudice to any reduction brought about by
payments or remittance made. Wyeth Suaco Laboratories, Inc. is also ordered
to pay the Bureau of Internal Revenue the amount of P60,855.21 as deficiency
sales tax with interest and surcharge in accordance with law. Costs against
private respondent.

SO ORDERED

G.R. No. L-22734 September 15, 1967

COMMISSIONER OF INTERNAL REVENUE, petitioner,


vs.
MANUEL B. PINEDA, as one of the heirs of deceased ATANASIO
PINEDA, respondent.

Office of the Solicitor General for petitioner.


Manuel B. Pineda for and in his own behalf as respondent.

BENGZON, J.P., J.:

On May 23, 1945 Atanasio Pineda died, survived by his wife, Felicisima Bagtas,
and 15 children, the eldest of whom is Manuel B. Pineda, a lawyer. Estate
proceedings were had in the Court of First Instance of Manila (Case No. 71129)
wherein the surviving widow was appointed administratrix. The estate was
divided among and awarded to the heirs and the proceedings terminated on
June 8, 1948. Manuel B. Pineda's share amounted to about P2,500.00.

After the estate proceedings were closed, the Bureau of Internal Revenue
investigated the income tax liability of the estate for the years 1945, 1946, 1947
and 1948 and it found that the corresponding income tax returns were not filed.
Thereupon, the representative of the Collector of Internal Revenue filed said
returns for the estate on the basis of information and data obtained from the
aforesaid estate proceedings and issued an assessment for the following:

1. Deficiency income tax


1945 P135.83
127

1946 436.95
1947 1,206.91 P1,779.69
Add: 5%
surcharge 88.98
1% monthly
interest from
November
30, 1953 to
April 15,
1957 720.77
Compromise
for late filing 80.00
Compromise
for late
payment 40.00

Total amount due P2,707.44


===========
Additional
2. residence tax for P14.50
1945 ===========
3. Real Estate
dealer's tax for
the fourth quarter
of 1946 and the
whole year of P207.50
1947 ===========

Manuel B. Pineda, who received the assessment, contested the same.


Subsequently, he appealed to the Court of Tax Appeals alleging that he was
appealing "only that proportionate part or portion pertaining to him as one of
the heirs."

After hearing the parties, the Court of Tax Appeals rendered judgment
reversing the decision of the Commissioner on the ground that his right to assess
and collect the tax has prescribed. The Commissioner appealed and this Court
affirmed the findings of the Tax Court in respect to the assessment for income
tax for the year 1947 but held that the right to assess and collect the taxes for
1945 and 1946 has not prescribed. For 1945 and 1946 the returns were filed on
August 24, 1953; assessments for both taxable years were made within five
years therefrom or on October 19, 1953; and the action to collect the tax was
filed within five years from the latter date, on August 7, 1957. For taxable year
1947, however, the return was filed on March 1, 1948; the assessment was
made on October 19, 1953, more than five years from the date the return was
filed; hence, the right to assess income tax for 1947 had prescribed.
Accordingly, We remanded the case to the Tax Court for further appropriate
proceedings.1
128

In the Tax Court, the parties submitted the case for decision without additional
evidence.

On November 29, 1963 the Court of Tax Appeals rendered judgment holding
Manuel B. Pineda liable for the payment corresponding to his share of the
following taxes:

Deficiency income tax

P135.8
1945
3
1946 436.95
Real estate
dealer's
fixed tax 4th
quarter of
1946 and
whole year
of 1947 P187.50

The Commissioner of Internal Revenue has appealed to Us and has proposed


to hold Manuel B. Pineda liable for the payment of all the taxes found by the
Tax Court to be due from the estate in the total amount of P760.28 instead of
only for the amount of taxes corresponding to his share in the
estate.1awphîl.nèt

Manuel B. Pineda opposes the proposition on the ground that as an heir he is


liable for unpaid income tax due the estate only up to the extent of and in
proportion to any share he received. He relies on Government of the Philippine
Islands v. Pamintuan2 where We held that "after the partition of an estate, heirs
and distributees are liable individually for the payment of all lawful outstanding
claims against the estate in proportion to the amount or value of the property
they have respectively received from the estate."

We hold that the Government can require Manuel B. Pineda to pay the full
amount of the taxes assessed.

Pineda is liable for the assessment as an heir and as a holder-transferee of


property belonging to the estate/taxpayer. As an heir he is individually
answerable for the part of the tax proportionate to the share he received from
the inheritance.3 His liability, however, cannot exceed the amount of his share.4

As a holder of property belonging to the estate, Pineda is liable for he tax up to


the amount of the property in his possession. The reason is that the Government
has a lien on the P2,500.00 received by him from the estate as his share in the
inheritance, for unpaid income taxes4a for which said estate is liable, pursuant
to the last paragraph of Section 315 of the Tax Code, which we quote
hereunder:

If any person, corporation, partnership, joint-account (cuenta en


participacion), association, or insurance company liable to pay the
129

income tax, neglects or refuses to pay the same after demand, the
amount shall be a lien in favor of the Government of the Philippines from
the time when the assessment was made by the Commissioner of Internal
Revenue until paid with interest, penalties, and costs that may accrue in
addition thereto upon all property and rights to property belonging to the
taxpayer: . . .

By virtue of such lien, the Government has the right to subject the property in
Pineda's possession, i.e., the P2,500.00, to satisfy the income tax assessment in
the sum of P760.28. After such payment, Pineda will have a right of contribution
from his co-heirs,5 to achieve an adjustment of the proper share of each heir in
the distributable estate.

All told, the Government has two ways of collecting the tax in question. One,
by going after all the heirs and collecting from each one of them the amount
of the tax proportionate to the inheritance received. This remedy was adopted
in Government of the Philippine Islands v. Pamintuan, supra. In said case, the
Government filed an action against all the heirs for the collection of the tax.
This action rests on the concept that hereditary property consists only of that
part which remains after the settlement of all lawful claims against the estate,
for the settlement of which the entire estate is first liable.6 The reason why in
case suit is filed against all the heirs the tax due from the estate is levied
proportionately against them is to achieve thereby two results: first, payment of
the tax; and second, adjustment of the shares of each heir in the distributed
estate as lessened by the tax.

Another remedy, pursuant to the lien created by Section 315 of the Tax Code
upon all property and rights to property belonging to the taxpayer for unpaid
income tax, is by subjecting said property of the estate which is in the hands of
an heir or transferee to the payment of the tax due, the estate. This second
remedy is the very avenue the Government took in this case to collect the tax.
The Bureau of Internal Revenue should be given, in instances like the case at
bar, the necessary discretion to avail itself of the most expeditious way to
collect the tax as may be envisioned in the particular provision of the Tax Code
above quoted, because taxes are the lifeblood of government and their
prompt and certain availability is an imperious need.7 And as afore-stated in
this case the suit seeks to achieve only one objective: payment of the tax. The
adjustment of the respective shares due to the heirs from the inheritance, as
lessened by the tax, is left to await the suit for contribution by the heir from
whom the Government recovered said tax.

WHEREFORE, the decision appealed from is modified. Manuel B. Pineda is


hereby ordered to pay to the Commissioner of Internal Revenue the sum of
P760.28 as deficiency income tax for 1945 and 1946, and real estate dealer's
fixed tax for the fourth quarter of 1946 and for the whole year 1947, without
prejudice to his right of contribution for his co-heirs. No costs. So ordered.

G.R. No. L-66160 May 21, 1990


130

COMMISSIONER OF INTERNAL REVENUE, petitioner,


vs.
UNION SHIPPING CORPORATION and THE COURT OF TAX APPEALS, respondents.

Artemio M. Lobrin for private respondent.

PARAS, J.:

This is a petition for review on certiorari of the December 9, 1983 decision * of


the Court of Tax Appeals in CTA Case No. 2989 reversing the Commissioner of
Internal Revenue.

In a letter dated December 27, 1974 (Exhibit "A") herein petitioner Commissioner
of Internal Revenue assessed against Yee Fong Hong, Ltd. and/or herein private
respondent Union Shipping Corporation, the total sum of P583,155.22 as
deficiency income taxes due for the years 1971 and 1972. Said letter was
received on January 4, 1975, and in a letter dated January 10, 1975 (Exhibit "B"),
received by petitioner on January 13, 1975, private respondent protested the
assessment.

Petitioner, without ruling on the protest, issued a Warrant of Distraint and Levy
(Exhibit "C"), which was served on private respondent's counsel, Clemente
Celso, on November 25, 1976.

In a letter dated November 27, 1976 (Exhibit "D"), received by petitioner on


November 29, 1976 (Exhibit "D-1") private respondent reiterated its request for
reinvestigation of the assessment and for the reconsideration of the summary
collection thru the Warrant of Distraint and Levy.

Petitioner, again, without acting on the request for reinvestigation and


reconsideration of the Warrant of Distraint and Levy, filed a collection suit
before Branch XXI of the then Court of First Instance of Manila and docketed as
Civil Case No. 120459 against private respondent. Summons (Exhibit "E") in the
said collection case was issued to private respondent on December 28, 1978.

On January 10, 1979, private respondent filed with respondent court its Petition
for Review of the petitioner's assessment of its deficiency income taxes in a
letter dated December 27, 1974, docketed therein as CTA Case No. 2989
(Rollo, pp. 44-49), wherein it prays that after hearing, judgment be rendered
holding that it is not liable for the payment of the income tax herein involved, or
which may be due from foreign shipowner Yee Fong Hong, Ltd.; to which
petitioner filed his answer on March 29, 1979 (Rollo, pp. 50-53).

Respondent Tax Court, in a decision dated December 9, 1983, ruled in favor of


private respondent —

WHEREFORE, the decision of the Commissioner of Internal Revenue


appealed from, assessing against and demanding from petitioner
the payment of deficiency income tax, inclusive of 50% surcharge,
131

interest and compromise penalties, in the amounts of P73,958.76 and


P583,155.22 for the years 1971 and 1972, respectively, is reversed.

Hence, the instant petition.

The Second Division of this Court, after the filing of the required pleadings, in a
resolution dated January 28, 1985, resolved to give due course to the petition,
and directed petitioner therein, to file his brief (Rollo, p. 145). In compliance,
petitioner filed his brief on May 10, 1985 (Rollo, p. 151). Respondents, on the
other hand, filed their brief on June 6, 1985 (Rollo, p. 156).

The main issues in this case are: (a) on the procedural aspect, whether or not
the Court of Tax Appeals has jurisdiction over this case and (b) on the merits,
whether or not Union Shipping Corporation acting as a mere "husbanding
agent" of Yee Fong Hong Ltd. is liable for payment of taxes on the gross
receipts or earnings of the latter.

The main thrust of this petition is that the issuance of a warrant of distraint and
levy is proof of the finality of an assessment because it is the most drastic action
of all media of enforcing the collection of tax, and is tantamount to an outright
denial of a motion for reconsideration of an assessment. Among others,
petitioner contends that the warrant of distraint and levy was issued after
respondent corporation filed a request for reconsideration of subject
assessment, thus constituting petitioner's final decision in the disputed
assessments (Brief for petitioner, pp. 9 and 12).

Petitioner argues therefore that the period to appeal to the Court of Tax
Appeals commenced to run from receipt of said warrant on November 25,
1976, so that on January 10, 1979 when respondent corporation sought redress
from the Tax Court, petitioner's decision has long become final and executory.

On this issue, this Court had already laid down the dictum that the
Commissioner should always indicate to the taxpayer in clear and unequivocal
language what constitutes his final determination of the disputed assessment.

Specifically, this Court ruled:

. . . we deem it appropriate to state that the Commissioner of


Internal Revenue should always indicate to the taxpayer in clear and
unequivocal language whenever his action on an assessment
questioned by a taxpayer constitutes his final determination on the
disputed assessment, as contemplated by sections 7 and 11 of
Republic Act 1125, as amended. On the basis of this statement
indubitably showing that the Commissioner's communicated action
is his final decision on the contested assessment, the aggrieved
taxpayer would then be able to take recourse to the tax court at the
opportune time. Without needless difficulty, the taxpayer would be
able to determine when his right to appeal to the tax court accrues.
This rule of conduct would also obviate all desire and opportunity on
the part of the taxpayer to continually delay the finality of the
assessment — and, consequently, the collection of the amount
132

demanded as taxes — by repeated requests for recomputation and


reconsideration. On the part of the Commissioner, this would
encourage his office to conduct a careful and thorough study of
every questioned assessment and render a correct and definite
decision thereon in the first instance. This would also deter the
Commissioner from unfairly making the taxpayer grope in the dark
and speculate as to which action constitutes the decision
appealable to the tax court. Of greater import, this rule of conduct
would meet a pressing need for fair play, regularity, and orderliness
in administrative action. (Surigao Electric Co., Inc. v. C.T.A., 57 SCRA
523, 528, [1974]).

There appears to be no dispute that petitioner did not rule on private


respondent's motion for reconsideration but contrary to the above ruling of this
Court, left private respondent in the dark as to which action of the
Commissioner is the decision appealable to the Court of Tax Appeals. Had he
categorically stated that he denies private respondent's motion for
reconsideration and that his action constitutes his final determination on the
disputed assessment, private respondent without needless difficulty would have
been able to determine when his right to appeal accrues and the resulting
confusion would have been avoided.

Much later, this Court reiterated the above-mentioned dictum in a ruling


applicable on all fours to the issue in the case at bar, that the reviewable
decision of the Bureau of Internal Revenue is that contained in the letter of its
Commissioner, that such constitutes the final decision on the matter which may
be appealed to the Court of Tax Appeals and not the warrants of
distraint (Advertising Associates, Inc. v. Court of Appeals, 133 SCRA 769 [1984]
emphasis supplied). It was likewise stressed that the procedure enunciated is
demanded by the pressing need for fair play, regularity and orderliness in
administrative action.

Under the circumstances, the Commissioner of Internal Revenue, not having


clearly signified his final action on the disputed assessment, legally the period to
appeal has not commenced to run. Thus, it was only when private respondent
received the summons on the civil suit for collection of deficiency income on
December 28, 1978 that the period to appeal commenced to run.

The request for reinvestigation and reconsideration was in effect considered


denied by petitioner when the latter filed a civil suit for collection of deficiency
income. So. that on January 10, 1979 when private respondent filed the appeal
with the Court of Tax Appeals, it consumed a total of only thirteen (13) days
well within the thirty day period to appeal pursuant to Section 11 of R.A. 1125.

On the merits, it was found fully substantiated by the Court of Tax Appeals that,
respondent corporation is the husbanding agent of the vessel Yee Fong Hong,
Ltd. as follows:

Coming to the second issue, petitioner contended and was


substantiated by satisfactory uncontradicted testimonies of
Clemente Celso, Certified Public Accountant, and Rodolfo C.
133

Cabalquinto, President and General Manager, of petitioner that it is


actually and legally the husbanding agent of the vessel of Yee Fong
Hong, Ltd. as (1) it neither performed nor transacted any shipping
business, for and in representation, of Yee Fong Hong, Ltd. or its
vessels or otherwise negotiated or procured cargo to be loaded in
the vessels of Yee Fong Hong, Ltd. (p. 21, t.s.n., July 16, 1980); (2) it
never solicited or procured cargo or freight in the Philippines or
elsewhere for loading in said vessels of Yee Fong Hong, Ltd. (pp. 21 &
38, ibid.); (3) it had not collected any freight income or receipts for
the said Yee Fong Hong, Ltd. (pp. 22 & 38, ibid; pp. 46 & 48, t.s.n.,
Nov. 14, 1980.); (4) it never had possession or control, actual or
constructive, over the funds representing payment by Philippine
shippers for cargo loaded on said vessels (pp. 21 & 38, ibid; p.
48, ibid); petitioner never remitted to Yee Fong Hong, Ltd. any sum of
money representing freight incomes of Yee Fong Hong, Ltd. (p.
21, ibid.; p. 48, ibid); and (5) that the freight payments made for
cargo loaded in the Philippines for foreign destination were actually
paid directly by the shippers to the said Yee Fong Hong, Ltd. upon
arrival of the goods in the foreign ports. (Rollo, pp. 58-59).

On the same issue, the Commissioner of Internal Revenue Misael P. Vera, on


query of respondent's counsel, opined that respondent corporation being
merely a husbanding agent is not liable for the payment of the income taxes
due from the foreign ship owners loading cargoes in the Philippines (Rollo, p. 63;
Exhibit "I", Rollo, pp. 64-66).

Neither can private respondent be liable for withholding tax under Section 53
of the Internal Revenue Code since it is not in possession, custody or control of
the funds received by and remitted to Yee Fong Hong, Ltd., a non-resident
taxpayer. As correctly ruled by the Court of Tax Appeals, "if an individual or
corporation like the petitioner in this case, is not in the actual possession,
custody, or control of the funds, it can neither be physically nor legally liable or
obligated to pay the so-called withholding tax on income claimed by Yee
Fong Hong, Ltd." (Rollo, p. 67).

Finally, it must be stated that factual findings of the Court of Tax Appeals are
binding on this Court (Industrial Textiles Manufacturing Company of the Phil.,
Inc. (ITEMCOP) v. Commissioner of Internal Revenue, et al. (136 SCRA 549
[1985]). It is well-settled that in passing upon petitions for review of the decisions
of the Court of Tax Appeals, this Court is generally confined to questions of law.
The findings of fact of said Court are not to be disturbed unless clearly shown to
be unsupported by substantial evidence (Commissioner of Internal Revenue v.
Manila Machinery & Supply Company, 135 SCRA 8 [1985]).

A careful scrutiny of the records reveals no cogent reason to disturb the


findings of the Court of Tax Appeals.

PREMISES CONSIDERED, the instant petition is hereby DISMISSED and the


assailed decision of the Court of Tax Appeals is hereby AFFIRMED.

SO ORDERED
134

G.R. No. L-46954 July 20, 1982

ELPIDIO YABES and SEVERINO YABES, petitioners,


vs.
THE HON. NAPOLEON FLOJO, in his capacity as Presiding Judge of Branch II,
Court of First Instance of Cagayan and THE REPUBLIC OF THE
PHILIPPINES, respondents.

Bito, Misa & Lozada & Associates for petitioners.

Solicitor General Estelito P. Mendoza, Asst. Solicitor General Ruben E. Agpolo


and Solicitor Deusdedit B. Quinano for respondents.

CONCEPCION JR., J.:

The lone issue raised in this petition for certiorari and prohibition, which seeks to
annul the Order dated June 22, 1971 issued by the Court of First Instance of
Cagayan in Civil Case No. II-7, which denied the motion to dismiss said case
dated March 25, 1971, filed by petitioner; 1 the Order dated June 7, 1977 of the
respondent District Judge of said Court in the same civil case denying
petitioners' motion for reconsideration of the said Order of denial dated June
22, 1971; 2 and the Order dated July 21, 1977, issued by the said respondent
Judge of said Court in the same civil case denying petitioners' motion for leave
to file a second motion for reconsideration of the aforesaid order of denials; 3 is
whether or not respondent Court of First Instance can lawfully acquire
jurisdiction over a contested assessment made by the Commissioner of Internal
Revenue against the deceased taxpayer Doroteo Yabes, which has not yet
become final, executory and incontestable, and which assessment is being
contested by petitioners in the Court of Tax Appeals, Case No. 2216, and still
pending consideration.

After this Court required respondents to comment on the petition and issued a
temporary restraining order in the Resolution dated September 28, 1977, 4 the
Solicitor General, in his Comment dated November 21, 1977, submitted that the
petition be given due course, and thereafter judgment be rendered setting
aside the questioned orders issued by the respondent Court of First Instance of
Cagayan in Civil Case No. II-7, directing said lower Court to hold in abeyance
any action or proceeding in Civil Case No. II-7, until after the Court of Tax
Appeals shall have finally decided CTA Case No. 2216. 5 The Solicitor General
also filed a Manifestation dated November 22, 1977, stating that "in their
Comment dated November 21, 1977, they have limited their appearance as
counsel only for the Republic of the Philippines and not for the respondent
Judge on the ground that they do not agree with the latter's orders which are
being questioned in the instant petition." 6

Undisputed facts of record are as follows:

(1) Doroteo Yabes of Calamaniugan Cagayan, who was for


sometime an exclusive dealer of products of the International
135

Harvester Macleod, Inc., received on or about May 1, 1962, a letter


from the Commissioner of Internal Revenue dated March 27, 1962,
demanding payment of the amount of P15,976.81, as commercial
broker's fixed and percentage taxes plus surcharges and the sum of
P2,530 as compromise penalty alledgely due from Yabes for the
years 1956-1960; 7

(2) On May 11, 1962, Doroteo Yabes, through his counsel, filed with
the Commissioner's Office his letter dated May 10, 1962, protesting
the assessment of commercial broker's fixed and percentage taxes
plus penalties against him on the ground that his agreements with
the International Harvester Macleod, Inc. were of purchase and sale,
and not of agency, hence he claimed he was not able to pay such
kind of taxes; 8

(3) Thereafter, there ensued an exchange of correspondence


between the lawyers of Doroteo Yabes and the Commissioner; the
Commissioner in a letter dated August 3, 1962, informed Doroteo
Yabes that he acted as a commercial broker "in accordance with
the ruling of this Office in the case of Cirilo D. Constantino;" 9 in turn,
Doroteo Yabes, in a letter dated August 22, 1962, requested for the
reinvestigation, or review of the case by the appellate division of the
Bureau of Internal Revenue in accordance with standing rules,
regulations or practice on the matter; 10 Yabes also wrote the
Commissioner on August 24, 1962, requesting that the appeal be
held in abeyance pending final decision of the Case of Cirilo D.
Constantino; 11 in reply, the Commissioner informed Doroteo Yabes in
a letter dated September 18, 1962, that the latter's request for
reinvestigation was denied on the ground that he has "not submitted
any evidence to offset the findings of this Office as to warrant a
reinvestigation thereof"; 12 but eight days later or on September 26,
1962, the Commissioner wrote a letter advising Doroteo Yabes that
"the administrative appeal ... will be held in abeyance pending the
resolution of the issues in a similar case (obviously referring to the
aforesaid Constantino case)";

(4) To give time for the Commissioner to study the case and several
other cases similar thereto, the lawyers of Doroteo Yabes agreed to
file, and their client, Doroteo Yabes did file a tax waiver on October
20, 1962, extending the period of prescription to December 31,
1967; 13

(5) Doroteo Yabes died on March 13, 1963 and no estate


proceedings were instituted for the settlement of his estate; his
widow also died during the pendency of the case; the petitioners
are the children of the deceased taxpayer; 14

(6) On March 14, 1966, the Court of Tax Appeals decided the
Constantino "test" case. The Court of Tax Appeals ruled that
agreements entered into by Constantino with the International
Harvester Macleod, Inc. were of purchase and sale, and not of
136

agency, hence no commercial broker's fixed and percentage fees


could be collected from the said taxpayer; however this Court on
February 27, 1970, in G.R. No. L-25926 reversed the Court of Tax
Appeals and ruled in favor of the Commissioner of Internal
Revenue; 15

(7) After a lapse of about five years, the heirs of the deceased
Doroteo Yabes, through their lawyers, received on August 4, 1967, a
letter from the Commissioner dated July 27, 1967, requesting that
they "waive anew the Statute of Limitations" and further confirming
the previous understanding that the final resolution of the protest of
the deceased Doroteo Yabes was "being held in abeyance until the
Supreme Court renders its decision on a similar case involving the
same factual and legal issues brought to it on appeal" (referring to
the Constantino "test" case); 16 conformably with the request of the
Commissioner, the heirs of Doroteo Yabes filed a revised waiver
further extending the period of prescription to December 31, 1970; 17

(8) Thereafter, no word was received by the petitioners or their


lawyers during the interim of more than three (3) years, but on
January 20, 1971, petitioners as heirs of the deceased Doroteo Yabes
received the summons and a copy of the complaint filed by the
Commissioner on December 4, 1970 with the Court of First Instance
of Cagayan which seeks to collect from the petitioners the sum of P
15,976.82, as deficiency commercial broker's fixed and percentage
taxes, including surcharges and interest thereon, due from their
predecessor-in-interest, Doroteo Yabes, by reason of the latter's
income derived from transactions as dealer of the products of the
International Harvester Macleod, Inc.;

(9) Taking the complaint as the final decision of the Commissioner on


the disputed assessment against the deceased taxpayer Doroteo
Yabes, petitioners filed on February 12, 1971, a petition for review of
said disputed assessment with the Court of Tax Appeals; 18 later on
the same day, February 12, 1971, petitioners filed their answer to the
complaint of the Commissioner before the Court of First Instance of
Cagayan; 19 and alleged therein, by way of special defense, that
the Court of Tax Appeals has exclusive jurisdiction of the action and
that there is another action of the same nature between the parties
relating to the same assessment pending before the Court of Tax
Appeals;

(10) On the other hand, the Commissioner filed a motion to dismiss


dated March 24, 1971, with the Court of Tax Appeals in CTA Case
No. 2216, and subsequently filed a memorandum in support of said
motion to dismiss, on the ground that the assessment against
Doroteo Yabes had already become final, executory and
incontestable, and the Court of Tax Appeals had no jurisdiction over
the case;
137

(11) On March 25, 1971, petitioners filed a formal motion to dismiss


Civil Case No. II-7 with the Court of First Instance of Cagayan on the
grounds that said Court has no jurisdiction over the case and that
there is another action pending between the same parties for the
same cause before a competent court; 20

(12) On June 22, 1971, the respondent Court of First Instance of


Cagayan, through its former presiding judge issued the questioned
order in Civil Case No. II-7, which is the main subject of the instant
petition, denying the petitioners' motion to dismiss on the ground
that the petitioners "have already made a previous answer wherein
they categorically admitted the jurisdiction of the court over the
subject matter and the Court believes that, granting for the sake of
argument, there is a pending action between the same parties for
the same cause yet the judgment which may be rendered in the first
cited case does not necessarily bar the present action"; 21

(13) On September 1, 1972, the respondent Court of First Instance of


Cagayan issued an order holding the trial of Civil Case No. I I-7, in
abeyance upon the joint motion of the parties; 22

(14) On September 29, 1974, the Court of Tax Appeals denied the
Commissioner's motion to dismiss CTA Case No. 2216. 23 Accordingly,
on October 30, 1975, the Commissioner filed his Answer to the
petition for review. 24

(15) On December 17, 1976, however, the Court of First Instance of


Cagayan, this time presided by the respondent Judge Napoleon
Flojo, upon motion of counsel for the plaintiff therein, set Civil Case
No. II-7 for trial on January 27 and 28, 1977. 25

(16) On February 9, 1977, the respondent Judge Flojo denied the


petitioners' motion to suspend further proceedings and set the trial of
the case for March 5, 1977. 26

(17) On May 3, 1977, the herein petitioners filed a motion for the
reconsideration of the order issued on June 22, 1971 and for a ruling
on their affirmative defense that the Court of First Instance of
Cagayan has no jurisdiction over the case. 27

(18) On June 7, 1977, the respondent Judge denied the


aforementioned motion for reconsideration for lack of merit, and set
the trial of the case for June 23, 1977. 28

(19) On July 8, 1977, the petitioners filed a motion seeking leave to


file a second motion for reconsideration of the order issued on June
7, 1977, 29 attaching thereto a copy of their motion for
reconsideration. 30 The motions were denied on July 21, 1977, and
trial was set for August 18, and 19, 1977 31 which was postponed to
September 23, 1977. 32
138

Hence, the present recourse. As prayed for, a temporary restraining order was
issued on September 28, 1977. 33

As to the issue of whether or not the assessment made by the Commissioner of


Internal Revenue against the deceased taxpayer Doroteo Yabes, as contained
in the letter dated March 27, 1962, has become final, executory and
incontestable, after Doroteo Yabes had received the Commissioner's letter
dated August 3, 1962, denying the latter's protest against the said assessment
on September 18, 1962 and his failure to appeal therefrom within the 30-day
period contemplated under Section 11, of Republic Act 1125, We are
constrained to agree with the Court of Tax Appeals, when it denied the
Commissioner's motion to dismiss CTA Case No. 2216, that:

the period for appeal to this Court should not be counted from
September 18, 1962. In a letter of July 27, 1967, respondent informed
petitioners that a resolution of their protest was being held in
abeyance until the Supreme Court renders a decision on a similar
case "involving the same factual and legal issues". As a matter of
fact, in an earlier letter dated September 26, 1962, respondent also
informed petitioners' counsel that "administrative appeal for and in
behalf of their clients win be held in abeyance pending resolution of
the issues on a similar case which was appealed by you to the Court
of Tax Appeals". It is thus clear in these letters that respondent
reconsidered the finality of his decision of August 3, 1962,
assuming arguendo that the letter had a tenor of finality. 34

The Court of Tax Appeals in CTA Case No. 2216, stated further:

The records show that a warrant of distraint and levy was issued on
October 2, 1970. Had this been served on Doroteo Yabes, it would
have been equivalent to a final decision, ... There is, however,
nothing to show that it was ever served on Yabes. Neither is there
anything in the record to show that a formal decision of denial was
made after respondent's letter of July 27, 1967. 35

There is no reason for Us to disagree from or reverse the Court of Tax Appeals'
conclusion that under the circumstances of this case, what may be considered
as final decision or assessment of the Commissioner is the filing of the complaint
for collection in the respondent Court of First Instance of Cagayan, the
summons of which was served on petitioners on January 20, 1971, and that
therefore the appeal with the Court of Tax Appeals in CTA Case No. 2216 was
filed on time. 36 The respondent Court of First Instance of Cagayan can only
acquire jurisdiction over this case filed against the heirs of the taxpayer if the
assessment made by the Commissioner of Internal Revenue had become final
and incontestable. If the contrary is established, as this Court holds it to be,
considering the aforementioned conclusion of the Court of Tax Appeals on the
finality and incontestability of the assessment made by the Commissioner is
correct, then the Court of Tax Appeals has exclusive jurisdiction over this case.
Petitioners received the summons in Civil Case No. II-7 of the respondent Court
of First Instance of Cagayan on January 20, 1971, and petitioners filed their
appeal with the Court of Tax Appeals in CTA Case No. 2216, on February 12,
139

1971, well within the thirty-day prescriptive period under Section 11 of Republic
Act No. 1125. The Court of Tax Appeals has exclusive appellate jurisdiction to
review on appeal any decision of the Collector of Internal Revenue in cases
involving disputed assessments and other matters arising under the National
Internal Revenue Code. 37

For want of jurisdiction over the case, the Court of First Instance of Cagayan
should have dismissed the complaint filed in Civil Case No. II-7.

The recommendation of the Solicitor General that the lower court hold in
abeyance any action or proceeding in Civil Case No. II-7 until after the Court of
Tax Appeals shall have finally decided CTA Case No. 2216, is untenable since
the lower court has no jurisdiction over the case. Jurisdiction over an action
includes jurisdiction over all interlocutory matters incidental to the case and
deemed necessary to preserve the subject matter of the suit or protect interests
of the parties. Absent jurisdiction over the case, it would be improper for the
Court of First Instance of Cagayan to take cognizance over the case and act
upon interlocutory matters of the case, as well.

The dismissal of the complaint, however, is not sufficient. The ends of justice
would best be served by considering the complaint filed in Civil Case No. II-7
not only as a final notice of assessment but also as a counterclaim in CTA Case
No. 2216, in order to avoid mutiplicity of suits, as well as to expedite the
settlement of the controversy between the parties. After all, the two cases
involve the same parties, the same subject matter, and the same issue, which is
the liability of the heirs of the deceased Doroteo Yabes for commercial broker's
fixed and percentage taxes due from the said deceased.

WHEREFORE, the petition is granted and the writs prayed for are hereby issued.
The questioned orders dated June 22, 1971, June 7, 1977 and July 21, 1977 are
hereby annulled and set aside and the complaint filed in Civil Case No. II-7 of
the Court of First Instance of Cagayan, entitled: "Republic of the Philippines,
plaintiff, versus Nicolasa Jurado Yabes, et al., defendants," should be, as it is
hereby, dismissed, the same to be transferred to the Court of Tax Appeals to be
considered therein as a counterclaim in CTA Case No. 2216. The temporary
restraining order heretofore issued is hereby made permanent. Without costs.

SO ORDERED.

G.R. No. L-39387 June 29, 1982

PAMPANGA SUGAR DEVELOPMENT CO., INC., petitioner,


vs.
COURT OF INDUSTRIAL RELATIONS AND SUGAR WORKERS ASSOCIATION,
respondents.

MAKASIAR, J.:

Petitioner Pampanga Sugar Development Company, Inc. seeks the reversal of


the order dated June 6, 1974 of respondent Court of Industrial Relations
140

awarding to respondent Sugar Workers Association's (Union) counsel attorney's


fees equivalent to 20% of the judgment in CIR Case No. 4264- ULP and ordering
the lower court's Examining Division to compute the wage and fringe benefits
differentials due the 28 individual workers who did not execute quitclaims as
well as attorney's fees corresponding to 20% of the benefits due to 53 workers
who entered into agreements waiving their rights and benefits under the
decision dated December 4, 1972 in the aforecited case; also, the setting aside
of the CIR resolution of September 3, 1974 denying petitioner's motion for
reconsideration of the questioned order (pp. 15 & 57, rec.).

For a better appreciation of this case, certain prefatory facts must be recalled.
Sometime in February, 1956, the workers' affiliates of respondent Union staged a
strike against petitioner company. This labor dispute was certified by the
President to the Court of Industrial Relations which was docketed as Case No.
13-IPA. After six years, the said Court issued an order on November 8, 1962
directing petitioner company to reinstate the members of respondent union.
On March 12, 1963 some 88 union members were thus reinstated by petitioner.
However, petitioner discriminated against the reemployed workers with respect
to wage rates, off-season pay, cost of living allowance, milling bonus and
Christmas bonus by depriving them of aforesaid benefits or by granting to some
members benefits lesser than those given to members of the Pasudeco Workers
Union, another labor group in the service of petitioner. By reason of such denial
and/or grant of lower benefits to respondent's members because of their union
affiliation and union activities, respondent filed with the CIR a complaint dated
September 10, 1964 for unfair labor practice against petitioner which case was
docketed as Case No. 4264-ULP.

On December 4, 1972, the CIR handed down a decision adjudging herein


petitioner guilty of unfair labor practice acts as charged and finding the same
to have been committed, and thereby directing petitioner to cease and desist
from further committing the said unfair labor practice acts and directing
petitioner to pay wage differentials to certain workers and fringe benefits as
would be found due and payable to them and to readmitted seasonal and
casual members of respondent union totalling 88 with the exception of 7
workers.

In a resolution dated May 28, 1973, the CIR denied petitioner's motion for
reconsideration of aforesaid decision filed on December 14, 1972. Petitioner
appealed the above decision and resolution to this Court on June 15, 1973
praying in its petition for the nullification of said decision and motion for being
contrary to law, and for the rendition of a new judgment dismissing CIR Case
No. 4264-ULP.

This Court, in its resolution of July 31, 1973, denied the said petition for review
(docketed as G.R. No. L-36994) for lack of merit. Petitioner then moved for
reconsideration of aforesaid denial which was denied on October 4, 1973 for
lack of merit. Said resolution denying the motion for reconsideration thus
became final and executory on October 12, 1973.

With the finality of the December 4, 1972 decision having been settled,
respondent Union filed with the CIR a motion for computation of final judgment
141

and a petition for attorney's lien both dated October 17, 1973 (pp. 47 & 50,
rec.).

Petitioner company filed its answer to motion for computation of final judgment
and the petition for attorney's lien under date of November 20, 1973 (p. 52,
rec.).

The CIR, acting on the aforesaid motions of respondent Union, issued its order of
June 6, 1974 approving and granting to respondent's counsel, Atty. Ignacio
Lacsina, attorney's fees equivalent to 20% of the total amount of final judgment
or whatever recovery or settlement is made and directing its Examining Division
to compute the wage and fringe benefits differentials due the 28 individual
workers who did not waive or quitclaim their rights established by the decision
of December 4, 1972 as well as the attorney's fees equivalent to 20% of the
total wage and fringe benefits differentials due the fifty-three (53) individual
workers who executed agreements with the company waiving and
quitclaiming their rights, benefits and privileges under the aforesaid decision
(pp. 15 & 57, rec.).

Petitioner moved for reconsideration of aforecited order on June 26, 1974 and
on July 5, 1974, the arguments supporting said motion for reconsideration
followed (pp. 63 & 65, rec.).

Respondent Union then filed its motion to strike out the motion for
reconsideration dated July 23, 1974 (p. 72, rec.). In a resolution of September 3,
1974, respondent lower court denied petitioner's motion for reconsideration.

Thus, this appeal from the subject order and resolution of the CIR.

Petitioner alleges the following assignment of errors:

1. The Court of Industrial Relations erred in awarding attorney's fees to the


union's counsel equivalent to 20% of the total amount of final judgment or
whatever recovery or settlement is made thereunder; because, aside from
being inequitable, exorbitant, excessive and unconscionable, the same is
without legal basis.

2. The Court of Industrial Relations erred in ordering the Chief of its examining
division or his duly authorized representative to examine the payrolls, vouchers,
books of account and other pertinent documents of petitioner, and to
compute the wage and fringe-benefits differentials allegedly due the members
of respondent Union because such examination and computation have
become academic.

3. The Court of Industrial Relations erred in not denying or dismissing the two
motions filed by respondent union on October 18, 1973 praying therein that the
union's counsel be awarded attorney's fees and that an order be issued
directing the examining division of the court to compute the wage and fringe
benefits differentials allegedly due the members of the union under the
decision of December 4, 1972.

Respondents, however, contend that —


142

1. The issue of quitclaims is now res judicata;

2. The CIR finding that 81 members of respondent union are entitled


to adjudged benefits is no longer alterable after decision has
become final;

3. The CIR power to adjust unfair labor practices is unaffected by


individual settlements;

4. The rights of labor are unwaivable; quitclaims null and void; and

5. The question regarding alleged unreasonableness of award of


attorney's fees, not raised before Court a quo, is barred on appeal.

After a careful evaluation of the petitioners' and respondents' pleadings, this


Court, finds the allegations of petitioner to be without merit.

On the first assignment of error, paragraph (a), the petitioner failed to raise the
issue before the trial court. This Court notes that petitioner's answer to the
motion for computation of final judgment and to petition for attorney's lien filed
by the respondent in the trial court did not raise the foregoing issue. It is a well-
settled doctrine in this jurisdiction that issues not raised in the trial court may not
be raised on appeal. Otherwise, there will be no end to litigations thus
defeating the ends of justice.

Nevertheless, this Court finds the allegations to be devoid of merit. Petitioner's


contention that there is no basis for respondent's petition for attorney's lien filed
with the trial court containing allegations relative to attorney's fees as agreed
upon between him and his client, the complainant Sugar Workers' Association,
is untenable. The written conformity of the President of said Sugar Workers
Association on behalf thereof confirms the existence of such an agreement on
attorney's fees and constitutes an irrefutable evidence of such agreement. The
trial court, therefore, had sufficient evidence upon which it based its decision.
The petitioner did not contest the allegations contained in the respondent's
petition for attorney's lien before the trial court. This constitutes an implied
admission thereof. Moreover, it is evident from the tenor of the trial court's order
issued on June 6, 1974 that the said court carefully evaluated the respondent's
petition for attorney's lien and even reduced the percentage from 25 IC to 20
%.

On the first assignment of error, paragraph (b), this Court likewise finds the same
to be without merit. This issue has already been resolved by this Court when the
petitioner filed its first petition for certiorari (G.R. No. L- 36994) seeking
nullification of the trial court's judgment on the same issue. Petitioner's
allegations were rejected by this Court in said case. It may not now be
repeated and raised on appeal before this Court, the same being res judicata.

Be that as it may, the allegations of petitioner to the effect that by reason of


the quitclaims there is nothing upon which the attorney's lien attaches, is not
valid. This Court finds the quitclaims not valid. Firstly, said quitclaims were
secured on December 27, 1972 by petitioner after it lost its case in the lower
143

court when the latter promulgated its decision on the case on December 4,
1972. Obviously in its desire to deny what is due the sugar workers concerned
and frustrate the decision of the lower court awarding benefits to them, it used
its moral ascendancy as employer over said workers to secure said quitclaims.
Predicated on said quitclaims, petitioner filed a petition for certiorari before this
Court but the same was denied by the Court on July 31, 1973 and October 4,
1973. Petitioner now has the audacity to return before this Court still invoking
said quitclaims, which We again reject.

Secondly, while rights may be waived, the same must not be contrary to law,
public order, public policy, morals or good customs or prejudicial to a third
person with a right recognized by law (Art. 6, New Civil Code). The quitclaim
agreements contain the following provisions in paragraph I 1, No. 3, thereof:

3. Nothing herein stipulated shall be construed as an admission


and/or recognition by the Party of The Second Part of its failure
refusal and/or omission as employer, to faithfully comply with the
pertinent laws, rules and regulations and/or agreements, nor its
liability therefor and thereunder.

Needless to state, the foregoing provisions are contrary to law, It exempts the
petitioner from any legal liability. The above- quoted provision renders the
quitclaim agreements void ab initio in their entirety since they obligated the
workers concerned to forego their benefits, while at the same time, exempted
the petitioner from any liability that it may choose to reject. This runs counter to
Article 22 of the New Civil Code which provides that no one shall be unjustly
enriched at the expense of another.

Thirdly, the alleged quitclaim agreements are contrary to public policy. Once a
civil action is filed in court, the cause of action may not be the subject of
compromise unless the same is by leave of the court concerned. Otherwise, this
will render the entire judicial system irrelevant to the prejudice of the national
interest. Parties to litigations cannot be allowed to trifle with the judicial system
by coming to court and later on agreeing to a compromise without the
knowledge and approval of the court. This converts the judiciary into a mere
tool of party-litigants who act according to their whims and caprices. This is
more so when the court has already rendered its decision on the issues
submitted.

In the case at bar, the lower court has already rendered a decision on the
issues presented before the alleged quitclaims agreements were made. The
quitclaim agreements were secured by petitioner while it filed a petition for
certiorari before this Court for a review of the lower court's decision. The
quiclaim agreements taken together with the petitioner's petition for certiorari
of the trial court's decision clearly and unmistakably shows the bad faith of the
petitioner and its outright refusal to comply with its legal obligations. And now it
has the temerity to attempt to use this Court as its instrument for the purpose.

This Court rejects the contention of petitioner to the effect that the lien of an
attorney on the judgment or decree for the payment of money and the
preference thereof which he has secured in favor of his client takes legal effect
144

only from and after, but not before notice of said lien has been entered in the
record and served on the adverse party, citing the cases of Menzi and Co. vs.
Bastida (63 Phil. 16) and Macondray & Co. vs. Jose (66 Phil. 590) in support
thereof.

This Court finds the petitioner's contentions and citations applicable only when
the case has already been decided with finality. In the case at bar, the original
case was decided with finality only after this Court denied the petitioner's
motion for reconsideration of this Court's denial of its petition for certiorari on
the lower court's decision.

This Court is appalled by the attempt of petitioner to mislead it by alleging that


the lower court recognized the validity and effectivity of the 53 individual
agreements when it declared allegedly that "rights may be waived. " The
records show that the lower court qualified its statement to the effect that the
waiver must not be contrary to law, public order, public policy, morals or good
customs, or prejudicial to a third person with a right recognized by law citing
Article 6 of the New Civil Code. This attempt by petitioner casts a serious doubt
on the integrity and good faith not only of the petitioner but also of its counsel.

This Court rejects the allegation of petitioner to the effect that the 53
agreements gave substance to the policy of the Industrial Peace Act of
encouraging the parties to make all reasonable efforts to settle their
differences by mutual agreement, citing the case of Filomena Dionela, et al. vs.
CIR, et al. (L-18334, August 31, 1963).

Petitioner's contention and the case cited in support thereof apply only where
there is good faith on the part of the party litigants. In the case at bar,
petitioner acted with evident bad faith and malice. Petitioner secured the 53
quitclaim agreements individually with the 53 sugar workers without the
intervention of respondent's lawyer who was representing them before the
lower court. This subterfuge is tantamount to a sabotage of the interest of
respondent association. Needless to say, the means employed by petitioner in
dealing with the workers individually, instead of collectively through respondent
and its counsel, violates good morals as they undermine the unity of
respondent union and fuels industrial disputes, contrary to the declared policy
in the Industrial Peace Act.

This Court likewise rejects petitioner's allegation that the 53 quitclaim


agreements were in the nature of a compromise citing the case of Republic vs.
Estenzo, et al., (L-24656, September 25, 1968, 25 SCRA 122) and Articles 2028
and 2040 of the New Civil Code.

Petitioner's allegations and citations apply only to compromises between the


party-litigants done in good faith. In the case at bar, there was no compromise
between the petitioner and the respondent Sugar Workers Association. In
respect of the 53 quitclaims, these are not compromise agreements between
the petitioner and respondent union. They are separate documents of
renunciation of individual rights. Compromise involves the mutual renunciation
of rights by both parties on a parity basis. The quitclaims, however, bind the
workers to renounce their rights while the petitioner not only does not renounce
145

anything but also acquires exemption from any legal liability in connection
therewith.

On the First Assignment of Error, Paragraph (c), the petitioner anchors his
allegations on the technical procedural requirements of Section 37, Rule 138 of
the New Rules of Court. This Court, however, finds petitioner's allegation without
merit. Said provision of the Rules of Court is meant to protect the interest of an
attorney's client and the adverse party by seeing to it that they are given the
opportunity to contest the creation of the attorney's lien. It will be noted from
the records that the client Sugar Workers Union was not only notified but also
affixed its conformity to the respondents' motion for attorney's lien. With respect
to the adverse party, the petitioner in this case, said adverse party's interest was
amply protected by the lower court when the latter admitted petitioner's
answer to respondent's motion for computation of final judgment and to
respondent's counsel's petition for attorney's lien. Petitioner did not raise the
aforesaid technicality in its answer before the lower court. It cannot now raise it
for the first time on appeal.

On the First Assignment of Error, Paragraph (d), this Court finds petitioner's
allegations to the effect that the attorney's fees awarded are inequitable,
exorbitant, excessive and unconscionable, citing in the process the case of
Meralco Workers' Union vs. Gaerlan (32 SCRA 419), completely without basis nor
merit.

Again, petitioner did not raise this issue in the lower court. It cannot now raise
said issue for the first time on appeal before this Court. Nevertheless, petitioner
has failed to prove any of its allegations. Hence, this Court finds the same
worthless. The Meralco case does not apply in this case for the reason that the
facts and circusmtances are entirely different.

On the Second Assignment of Error, this Court finds petitioner's allegation to the
effect that the lower court erred in ordering the computation of judgment on
the ground that by reason of the quitclaim agreements the computation of
judgment has become academic, to be without merit and grossly inane.

The allegations of petitioner are premised on its previous allegations regarding


the quitclaims. This Court has earlier stated that the quitclaim agreements are
void ab initio. The lower court was correct in directing the computation of
judgment, there being a basis therefor.

On the Third Assignment of Error, this Court likewise finds petitioner's allegations
which are based on its allegations in support of the first and second
assignments of errors, without merit, as heretofore discussed.

WHEREFORE, THE PETITION IS HEREBY DISMISSED AND RESPONDENT CIR (NOW THE
NLRC) IS HEREBY DIRECTED TO IMPLEMENT ITS ORDER DATED JUNE 6,1974.

COSTS AGAINST PETITIONER.

SO ORDERED.

G.R. No. L-41919-24 May 30, 1980


146

QUIRICO P. UNGAB, petitioner,


vs.
HON. VICENTE N. CUSI, JR., in his capacity as Judge of the Court of First
Instance, Branch 1, 16TH Judicial District, Davao City, THE COMMISSIONER OF
INTERNAL REVENUE, and JESUS N. ACEBES, in his capacity as State
Prosecutor, respondents.

CONCEPCION JR., J:

Petition for certiorari and prohibition with preliminary injunction and restraining
order to annul and set aside the informations filed in Criminal Case Nos. 1960,
1961, 1962, 1963, 1964, and 1965 of the Court of First Instance of Davao, all
entitled: "People of the Philippines, plaintiff, versus Quirico Ungab, accused;"
and to restrain the respondent Judge from further proceeding with the hearing
and trial of the said cases.

It is not disputed that sometime in July, 1974, BIR Examiner Ben Garcia
examined the income tax returns filed by the herein petitioner, Quirico P.
Ungab, for the calendar year ending December 31, 1973. In the course of his
examination, he discovered that the petitioner failed to report his income
derived from sales of banana saplings. As a result, the BIR District Revenue
Officer at Davao City sent a "Notice of Taxpayer" to the petitioner informing him
that there is due from him (petitioner) the amount of P104,980.81, representing
income, business tax and forest charges for the year 1973 and inviting petitioner
to an informal conference where the petitioner, duly assisted by counsel, may
present his objections to the findings of the BIR Examiner. 1 Upon receipt of the
notice, the petitioner wrote the BIR District Revenue Officer protesting the
assessment, claiming that he was only a dealer or agent on commission basis in
the banana sapling business and that his income, as reported in his income tax
returns for the said year, was accurately stated. BIR Examiner Ben Garcia,
however, was fully convinced that the petitioner had filed a fraudulent income
tax return so that he submitted a "Fraud Referral Report," to the Tax Fraud Unit of
the Bureau of Internal Revenue. After examining the records of the case, the
Special Investigation Division of the Bureau of Internal Revenue found sufficient
proof that the herein petitioner is guilty of tax evasion for the taxable year 1973
and recommended his prosecution: têñ.£îhqwâ£

(1) For having filed a false or fraudulent income tax return for 1973
with intent to evade his just taxes due the government under Section
45 in relation to Section 72 of the National Internal Revenue Code;

(2) For failure to pay a fixed annual tax of P50.00 a year in 1973 and
1974, or a total of unpaid fixed taxes of P100.00 plus penalties of
175.00 or a total of P175.00, in accordance with Section 183 of the
National Internal Revenue Code;

(3) For failure to pay the 7% percentage tax, as a producer of


banana poles or saplings, on the total sales of P129,580.35 to the
147

Davao Fruit Corporation, depriving thereby the government of its


due revenue in the amount of P15,872.59, inclusive of surcharge. 2

In a second indorsement to the Chief of the Prosecution Division, dated


December 12, 1974, the Commissioner of Internal Revenue approved the
prosecution of the petitioner. 3

Thereafter, State Prosecutor Jesus Acebes who had been designated to assist
all Provincial and City Fiscals throughout the Philippines in the investigation and
prosecution, if the evidence warrants, of all violations of the National Internal
Revenue Code, as amended, and other related laws, in Administrative Order
No. 116 dated December 5, 1974, and to whom the case was assigned,
conducted a preliminary investigation of the case, and finding probable
cause, filed six (6) informations against the petitioner with the Court of First
Instance of Davao City, to wit: têñ.£îhqwâ£

(1) Criminal Case No. 1960 — Violation of Sec. 45, in relation to Sec.
72 of the National Internal-Revenue Code, for filing a fraudulent
income tax return for the calendar year ending December 31,
1973; 4

(2) Criminal Case No. 1961 — Violation of Sec. 182 (a), in relation to
Secs. 178, 186, and 208 of the National Internal Revenue Code, for
engaging in business as producer of saplings, from January, 1973 to
December, 1973, without first paying the annual fixed or privilege tax
thereof; 5

(3) Criminal Case No. 1962 — Violation of Sec. 183 (a), in relation to
Secs. 186 and 209 of the National Internal Revenue Code, for failure
to render a true and complete return on the gross quarterly sales,
receipts and earnings in his business as producer of banana saplings
and to pay the percentage tax due thereon, for the quarter ending
December 31, 1973; 6

(4) Criminal Case No. 1963 — Violation of Sec. 183 (a), in relation to
Secs. 186 and 209 of the National Internal Revenue Code, for failure
to render a true and complete return on the gross quarterly sales
receipts and earnings in his business as producer of saplings, and to
pay the percentage tax due thereon, for the quarter ending on
March 31, 1973; 7

(5) Criminal Case No. 1964 — Violation of Sec. 183 (a), in relation to
Secs. 186 and 209 of the National Internal Revenue Code, for failure
to render a true and complete return on the gross quarterly sales,
receipts and earnings in his business as producer of banana saplings
for the quarter ending on June 30, 1973, and to pay the percentage
tax due thereon; 8

(6) Criminal Case No. 1965 — Violation of Sec. 183 (a), in relation to
Secs. 186 and 209 of the National Internal Revenue Code, for failure
to render a true and complete return on the gross quarterly sales,
148

receipts and earnings as producer of banana saplings, for the


quarter ending on September 30, 1973, and to pay the percentage
tax due thereon. 9

On September 16, 1975, the petitioner filed a motion to quash the informations
upon the grounds that: (1) the informations are null and void for want of
authority on the part of the State Prosecutor to initiate and prosecute the said
cases; and (2) the trial court has no jurisdiction to take cognizance of the
above-entitled cases in view of his pending protest against the assessment
made by the BIR Examiner. 10 However, the trial court denied the motion on
October 22, 1975. 11 Whereupon, the petitioner filed the instant recourse. As
prayed for, a temporary restraining order was issued by the Court, ordering the
respondent Judge from further proceeding with the trial and hearing of
Criminal Case Nos. 1960, 1961, 1962, 1963, 1964, and 1965 of the Court of First
Instance of Davao, all entitled: "People of the Philippines, plaintiff, versus
Quirico Ungab, accused."

The petitioner seeks the annulment of the informations filed against him on the
ground that the respondent State Prosecutor is allegedly without authority to
do so. The petitioner argues that while the respondent State Prosecutor may
initiate the investigation of and prosecute crimes and violations of penal laws
when duly authorized, certain requisites, enumerated by this Court in its
decision in the case of Estrella vs. Orendain, 12 should be observed before such
authority may be exercised; otherwise, the provisions of the Charter of Davao
City on the functions and powers of the City Fiscal will be meaningless because
according to said charter he has charge of the prosecution of all crimes
committed within his jurisdiction; and since "appropriate circumstances are not
extant to warrant the intervention of the State Prosecution to initiate the
investigation, sign the informations and prosecute these cases, said
informations are null and void." The ruling adverted to by the petitioner reads,
as follows: têñ.£îhqwâ£

In view of all the foregoing considerations, it is the ruling of this Court


that under Sections 1679 and 1686 of the Revised Administrative
Code, in any instance where a provincial or city fiscal fails, refuses or
is unable, for any reason, to investigate or prosecute a case and, in
the opinion of the Secretary of Justice it is advisable in the public
interest to take a different course of action, the Secretary of Justice
may either appoint as acting provincial or city fiscal to handle the
investigation or prosecution exclusively and only of such case, any
practicing attorney or some competent officer of the Department of
Justice or office of any city or provincial fiscal, with complete
authority to act therein in all respects as if he were the provincial or
city fiscal himself, or appoint any lawyer in the government service,
temporarily to assist such city of provincial fiscal in the discharge of
his duties, with the same complete authority to act independently of
and for such city or provincial fiscal provided that no such
appointment may be made without first hearing the fiscal
concerned and never after the corresponding information has
already been filed with the court by the corresponding city or
149

provincial fiscal without the conformity of the latter, except when it


can be patently shown to the court having cognizance of the case
that said fiscal is intent on prejudicing the interests of justice. The
same sphere of authority is true with the prosecutor directed and
authorized under Section 3 of Republic Act 3783, as amended
and/or inserted by Republic Act 5184. The observation in Salcedo vs.
Liwag, supra, regarding the nature of the power of the Secretary of
Justice over fiscals as being purely over administrative matters only
was not really necessary, as indicated in the above relation of the
facts and discussion of the legal issues of said case, for the resolution
thereof. In any event, to any extent that the opinion therein may be
inconsistent herewith the same is hereby modified.

The contention is without merit. Contrary to the petitioner's claim, the rule
therein established had not been violated. The respondent State Prosecutor,
although believing that he can proceed independently of the City Fiscal in the
investigation and prosecution of these cases, first sought permission from the
City Fiscal of Davao City before he started the preliminary investigation of these
cases, and the City Fiscal, after being shown Administrative Order No. 116,
dated December 5, 1974, designating the said State Prosecutor to assist all
Provincial and City fiscals throughout the Philippines in the investigation and
prosecution of all violations of the National Internal Revenue Code, as
amended, and other related laws, graciously allowed the respondent State
Prosecutor to conduct the investigation of said cases, and in fact, said
investigation was conducted in the office of the City Fiscal. 13

The petitioner also claims that the filing of the informations was precipitate and
premature since the Commissioner of Internal Revenue has not yet resolved his
protests against the assessment of the Revenue District Officer; and that he was
denied recourse to the Court of Tax Appeals.

The contention is without merit. What is involved here is not the collection of
taxes where the assessment of the Commissioner of Internal Revenue may be
reviewed by the Court of Tax Appeals, but a criminal prosecution for violations
of the National Internal Revenue Code which is within the cognizance of courts
of first instance. While there can be no civil action to enforce collection before
the assessment procedures provided in the Code have been followed, there is
no requirement for the precise computation and assessment of the tax before
there can be a criminal prosecution under the Code. têñ.£îhqwâ£

The contention is made, and is here rejected, that an assessment of


the deficiency tax due is necessary before the taxpayer can be
prosecuted criminally for the charges preferred. The crime is
complete when the violator has, as in this case, knowingly and
willfully filed fraudulent returns with intent to evade and defeat a
part or all of the tax. 14

An assessment of a deficiency is not necessary to a criminal


prosecution for willful attempt to defeat and evade the income tax.
A crime is complete when the violator has knowingly and willfuly filed
a fraudulent return with intent to evade and defeat the tax. The
150

perpetration of the crime is grounded upon knowledge on the part


of the taxpayer that he has made an inaccurate return, and the
government's failure to discover the error and promptly to assess has
no connections with the commission of the crime. 15

Besides, it has been ruled that a petition for reconsideration of an assessment


may affect the suspension of the prescriptive period for the collection of taxes,
but not the prescriptive period of a criminal action for violation of
law. 16 Obviously, the protest of the petitioner against the assessment of the
District Revenue Officer cannot stop his prosecution for violation of the National
Internal Revenue Code. Accordingly, the respondent Judge did not abuse his
discretion in denying the motion to quash filed by the petitioner.

WHEREFORE, the petition should be, as it is hereby dismissed. The temporary


restraining order heretofore issued is hereby set aside. With costs against the
petitioner.

SO ORDERED.

G.R. No. L-17177-80 December 28, 1964

THE PEOPLE OF THE PHILIPPINES, Plaintiff-Appellee, vs. ILDEFONSO


TIERRA, Defendant-Appellant.

Ildefonso Tierra in his own behalf as defendant-appellant.


Office of the Solicitor General for plaintiff-appellee.

BENGZON, C.J.:chanrobles virtual law library

The Cases: From the judgment of the Manila Court of First Instance finding him
guilty of violations of the income tax law, Ildefonso Tierra brought this appeal to
the Court of Appeals. However, it is now before us upon certification of said
appellate court that it merely raises questions of
law.chanroblesvirtualawlibrarychanrobles virtual law library

Four separate informations were filed against this defendant on December 12,
1955. The first information alleged:

That on or about the 1st day of March, 1947, in the City of Manila, Philippines,
the said accused, having a net income of P93,886.55 for the year ending
December 31, 1946, willfully, unlawfully and feloniously filed a false and
fraudulent income tax return for the said year by stating therein that he had
only an income of P19,196.14 for the year 1946, and with deliberate intent did
then and there willfully, unlawfully and feloniously refuse and still refuses to pay
the Government of the Republic of the Philippines, the deficiency income tax in
the sum of P30,632.51, ... notwithstanding the repeated demands ... .

The second information substantially worded as the first, accused Tierra of


having filed in March, 1948, a false and fraudulent return by making it appear
that his net income for 1947 was P65,225.41 when, in truth, he actually earned
net income of P621,072.80.chanroblesvirtualawlibrarychanrobles virtual law
library
151

The third information charged him with having filed in March, 1950, in his
capacity as president of a corporation named "Ildefonso Tierra & Sons, Inc.", a
false and fraudulent income tax return for said corporation by making it appear
that the corporation had earned in 1949, a net income of P24,833.47 when as a
matter of fact, its true net income for that year was P359,310.18, and that he
refused and still refuses to pay the corresponding deficiency income
tax.chanroblesvirtualawlibrarychanrobles virtual law library

The fourth information charged appellant, in his own personal capacity and as
president of the "Ildefonso Tierra & Sons, Inc.", with the offense of failing to keep
and to preserve his own books of account and those of the corporation, for a
period of at least five years from the date of the last entry in each book, as
required by section 337 of the National Internal Revenue
Code.chanroblesvirtualawlibrarychanrobles virtual law library

The four cases were jointly tried by agreement between the prosecution and
the accused, who conducted his own
defense.chanroblesvirtualawlibrarychanrobles virtual law library

After trial, the Manila court found the accused guilty as charged and
sentenced him as follows:

In Criminal Case No. 33595, the accused shall pay a fine of two thousand pesos
(P2,000.00) and suffer imprisonment of four (4) months. He shall also indemnify
the Republic of the Philippines in the sum of P30,632.51 and pay the
costs.chanroblesvirtualawlibrarychanrobles virtual law library

In Criminal Case No. 33596, the accused shall pay a fine of two thousand pesos
(P2,000.00) and suffer imprisonment of four (4) months, pay the costs and
indemnify the Republic of the Philippines in the sum of
P376,303.16.chanroblesvirtualawlibrarychanrobles virtual law library

In Criminal Case No. 33597, the accused shall pay a fine of two thousand pesos
(P2,000.00) and suffer imprisonment of four (4) months, pay the costs and
indemnify the Republic of the Philippines in the sum of
P60,205.80.chanroblesvirtualawlibrarychanrobles virtual law library

In Criminal Case No. 33598, the accused shall pay a fine of three hundred
pesos (P 300.00), suffer imprisonment of six (6) months and pay the costs.

Defendant appealed the judgment to the Court of Appeals and filed a "Motion
for Dismissal" which, with his assent was considered as his appeal brief. As
already stated, the Court of Appeals elevated the record to this Court,
because it raises questions of law only.chanroblesvirtualawlibrarychanrobles
virtual law library

Statement of Facts: The facts of the case are not much disputed. It was
established that appellant was, during the years 1946 up to 1949, engaged in
the general merchandise business, including the buying and selling of school
and office supplies. For the years 1946, 1947 and 1949, he filed his income tax
returns, declaring in full all his gross sales, and paid income taxes due thereon in
152

the respective sums of P2,557.54,


P13,097.63 and P2,980.00.chanroblesvirtualawlibrarychanrobles virtual law
library

Said returns were later verified by Valerians Robles, an income tax examiner of
the Bureau of Internal Revenue. And on December 16, 1950, examiner Robles
reported his finding, that appellant had filed false and fraudulent returns for
said by overstating his purchases for 1946, 1947 and 1949, and over declaring
his expenses, for 1947, thereby reducing the, net income subject to
tax.chanroblesvirtualawlibrarychanrobles virtual law library

During the investigation, the appellant could not produce to examiner Robles,
all the pertinent vouchers, sales invoices and other accounting records for the
Purpose) of verifying the correctness of the returns. In view of the inability of the
appellant to produce said books and records, examiner Robles resorted to the
so-called "percentage basis" of computing net income. Under this method, net
income was computed by the use of an average percentage method based
on the returns of taxpayers engaged in the same line of
business.chanroblesvirtualawlibrarychanrobles virtual law library

On the basis of the reports of examiner Robles, the Collector of Internal


Revenue made a formal demand dated December 23, 1950, on the accused
for the payment of his deficiency income taxes for 1946, 1947 and 1949,
including surcharges thereon, in the respective sums of P30,632.51 P376,303.16,
and P60,205.80, and asked him to show cause why he should not be
prosecuted for his failure to preserve his books of accounts, vouchers and
invoices, for a period of five years from the date of the last entry therein. The
Collector found that the income tax returns should have stated that his income
was P93,886.55, P621,072.80 and P359,310.18 for the years 1946, 1947 and
1949.chanroblesvirtualawlibrarychanrobles virtual law library

On December 29, 1950, Income Tax Assessment Notices were sent to the
appellant, giving him up to January 29, 1951 to pay the aforementioned
deficiency income taxes plus surcharges. Appellant protested against the
assessments. On February 26, 1951, the Collector of Internal Revenue reiterated
the demand for payment by the appellant. Up to now, nothing was paid by,
the appellant on account of the above claims of the
government.chanroblesvirtualawlibrarychanrobles virtual law library

The Issues: Appellant urges in his appeal the following propositions:

1. The informations are void because they do not state any


offense.chanroblesvirtualawlibrarychanrobles virtual law library

2. The actions against appellant had already prescribed when the informations
were filed.chanroblesvirtualawlibrarychanrobles virtual law library

3. The criminal liability of the accused, if any in the first three informations (L-
17177-9) has been extinguished by reason of the extinguishment of his civil
liability to pay taxes.chanroblesvirtualawlibrarychanrobles virtual law library
153

4. Section 51(d) of the National Internal Revenue Code upon which the
informations in the first three cases were based, has already been repealed,
and, therefore, he can no longer be prosecuted for its violation.

Discussion: Appellant urges at some length, that each of the informations in the
above-entitled case, is fatally defective because it did not recite with definite
particularity that he belongs to that class of persons to whom the statutory
provisions violated are specially applicable. Thus, he argues, the first two
informations failed to allege that he was a Filipino citizen or resident, that he is
of lawful age, that he has a gross income of P1,800.00 or over or that he is a
non-resident, etc. The same objection he raises with respect to the third and
fourth informations.chanroblesvirtualawlibrarychanrobles virtual law library

This objection merits no serious consideration. The information alleged the


income. Age and residence or citizenship were not necessary to allege, the
defendant having submitted his return of income, which implied he was bound
to make it. It is next contended that the right of the government to prosecute
those actions has prescribed, since more than five years had elapsed from the
alleged dates of commission of the offenses
charged.chanroblesvirtualawlibrarychanrobles virtual law library

Again appellant's contention has no merit. Section 354 of the National Internal
Revenue Code provides:

All violations of any provision of this Code shall prescribe after five
years.chanroblesvirtualawlibrarychanrobles virtual law library

Prescription shall begin to run from the day of the commission of the violation of
the law and if the same be not known at the time, from the discovery thereof
and the institution of judicial proceedings for its investigation and punishment.
...

Evidence was adduced to show' and the trial court so found, that the falsity of
the returns filed by the appellant and his failure to preserve his books of
accounts for at least five years from the date of the last entry in each book
were all discovered only on December 16, 1950. Since the informations were
filed on December 12, 1955, the trial court correctly ruled that the actions were
all within the five-year period of limitation.chanroblesvirtualawlibrarychanrobles
virtual law library

Appellant argues, however, that since the informations make no allegation that
the offenses were not known at the time of the commission as to bring them
within the exception to the statute of limitations, then the informations were
necessarily defective for that reason, and this fatal defect cannot be cured by
the introduction of evidence. Prescription is a matter of defense and the
information does not need to anticipate arid meet it. The defendant could, at
most, object to the introduction of evidence to defeat his claim of prescription;
but he did not. Anyway, the law says that prescription begins to run from ... "the
institution of judicial proceedings, for its ... punishment" (See above See. 354
cited).chanroblesvirtualawlibrarychanrobles virtual law library
154

In the third issue he raises, appellant contends that his criminal liability in the first
three informations (L-171779) has been extinguished because of the failure of
the government to take any timely action, judicial or administrative, to collect
his income tax liabilities, and because of this failure, the right of the government
to collect the taxes was lost by prescription in accordance with section 332 of
the National Internal Revenue Code. On the premise that his criminal liability
arose from his failure to satisfy this civil liability, appellant argues that the
extinguishment of this civil liability by prescription ipso facto extinguished any
criminal action based thereon.chanroblesvirtualawlibrarychanrobles virtual law
library

We cannot uphold appellant's view. The filing of a false and fraudulent income
tax return and the failure to pay the tax necessarily makes the delinquent
taxpayer amenable to the penal provisions of Section 73 of the Code. Any
subsequent satisfaction of the tax liability, by payment or prescription, will not
operate to extinguish such criminal liability, since the duty to pay the tax is
imposed by statute independent of any attempt on the part of the taxpayer to
evade payment. Whether under the National Internal Revenue Code or under
the Revised Penal Code, the satisfaction of civil liability is not one of the
grounds for the extinction of criminal action. The failure of the government,
therefore, to enforce by appropriate civil remedies the collection of the taxes,
does not detract from its right criminally to prosecute violations of the Code.
The criminal actions subsist so long as there are no legal grounds that would bar
their prosecution.chanroblesvirtualawlibrarychanrobles virtual law library

However, although. appellant does not specifically assign this as error, we hold
that the lower court erred in sentencing him in the first three cases to indemnify
the government for the amounts of deficiency taxes plus surcharges which he
failed to pay. This question was already laid at rest in the case of People vs.
Arnault (G.R. No. 1,4288, November 20, 1952, 48 O.f Gaz. 4805) wherein we held
that there is no legal sanction for the imposition of payment of the civil
indemnity to the government in a criminal proceeding for violation of the
income tax laws. We said in that case:

.... While section 73 of the National Internal Revenue Code provides for the
imposition of the penalty for refusal or neglect to pay income tax or to make a
return thereof, by imprisonment or fine, or both, it fails to provide for the
collection of said tax in criminal proceedings. As well contended by counsel for
appellant, Chapters I and II of Title IX of the National Internal Revenue Code
provides only for civil remedies for the collection of the income tax, and under
section 316, the civil remedy is either by distraint of goods, chattels, etc., or by
judicial action. It is a commonly accepted principle of law that the method
prescribed by statute for the collection of taxes is generally exclusive, and
unless a contrary intent be gathered from the statute, it should be followed
strictly. (3 Cooley, Law on Taxation, Section 1326, pp. 621-623)

Finally, appellant argues that section 51(d) of the National Internal Revenue
Code upon which the informations in the first three cases were based, had
already been repealed by Republic Act No. 2343 which took effect on June 20,
1959, and that, therefore, he can no longer be prosecuted and convicted for
155

offenses based thereon. Prior to the present amendment, section 51(d)


provided:

Refusal or neglect to make returns; fraudulent returns, etc. - In cases of refusal


or neglect to make a return and in cases of erroneous, false or fraudulent
returns, the Collector of Internal Revenue shall, upon the discovery thereof, at
any time within three years after said return is due, or has been made, make a
return upon information obtained as provided for in this code or by existing law
or require the necessary corrections to be made and the assessment made by
the Collector of Internal Revenue thereon shall be paid by such person or
corporation immediately upon notification of the amount of such assessment.

It is not now necessary to discuss the effect of the suppression of this provision
by virtue of Republic Act No. 2343. Suffice it to say that the accused is charged
not only for failure to pay deficiency taxes as assessed under section 51 (d) of
the Revenue Code prior to its amendment, but also under sections 45 and 46 in
relation to section 73 for filing false and fraudulent returns. Even without alleging
a violation of section 51(d), the indictments can still
stand.chanroblesvirtualawlibrarychanrobles virtual law library

Conclusion: IN VIEW OF THE FOREGOING, and there being no assertion of error


in the penalty imposed, the decision appealed from is hereby affirmed, with this
modification: the portions regarding payment of monetary indemnity in favor of
the Government are eliminated. With costs

G.R. No. 175097

ALLIED BANKING CORPORATION, Petitioner,


vs.
COMMISSIONER OF INTERNAL REVENUE, Respondent.

DECISION

DEL CASTILLO, J.:

The key to effective communication is clarity.

The Commissioner of Internal Revenue (CIR) as well as his duly authorized


representative must indicate clearly and unequivocally to the taxpayer
whether an action constitutes a final determination on a disputed
assessment.1 Words must be carefully chosen in order to avoid any confusion
that could adversely affect the rights and interest of the taxpayer.

Assailed in this Petition for Review on Certiorari2 under Section 12 of Republic


Act (RA) No. 9282,3 in relation to Rule 45 of the Rules of Court, are the August
23, 2006 Decision4 of the Court of Tax Appeals (CTA) and its October 17, 2006
Resolution5 denying petitioner’s Motion for Reconsideration.

Factual Antecedents

On April 30, 2004, the Bureau of Internal Revenue (BIR) issued a Preliminary
Assessment Notice (PAN) to petitioner Allied Banking Corporation for deficiency
156

Documentary Stamp Tax (DST) in the amount of ₱12,050,595.60 and Gross


Receipts Tax (GRT) in the amount of ₱38,995,296.76 on industry issue for the
taxable year 2001.6 Petitioner received the PAN on May 18, 2004 and filed a
protest against it on May 27, 2004.7

On July 16, 2004, the BIR wrote a Formal Letter of Demand with Assessment
Notices to petitioner, which partly reads as follows:8

It is requested that the above deficiency tax be paid immediately upon receipt
hereof, inclusive of penalties incident to delinquency. This is our final decision
based on investigation. If you disagree, you may appeal the final decision
within thirty (30) days from receipt hereof, otherwise said deficiency tax
assessment shall become final, executory and demandable.

Petitioner received the Formal Letter of Demand with Assessment Notices on


August 30, 2004.9

Proceedings before the CTA First Division

On September 29, 2004, petitioner filed a Petition for Review10 with the CTA
which was raffled to its First Division and docketed as CTA Case No. 7062.11

On December 7, 2004, respondent CIR filed his Answer.12 On July 28, 2005, he
filed a Motion to Dismiss13 on the ground that petitioner failed to file an
administrative protest on the Formal Letter of Demand with Assessment Notices.
Petitioner opposed the Motion to Dismiss on August 18, 2005.14

On October 12, 2005, the First Division of the CTA rendered a


Resolution15 granting respondent’s Motion to Dismiss. It ruled:

Clearly, it is neither the assessment nor the formal demand letter itself that is
appealable to this Court. It is the decision of the Commissioner of Internal
Revenue on the disputed assessment that can be appealed to this Court
(Commissioner of Internal Revenue vs. Villa, 22 SCRA 3). As correctly pointed
out by respondent, a disputed assessment is one wherein the taxpayer or his
duly authorized representative filed an administrative protest against the formal
letter of demand and assessment notice within thirty (30) days from date [of]
receipt thereof. In this case, petitioner failed to file an administrative protest on
the formal letter of demand with the corresponding assessment notices. Hence,
the assessments did not become disputed assessments as subject to the Court’s
review under Republic Act No. 9282. (See also Republic v. Liam Tian Teng Sons
& Co., Inc., 16 SCRA 584.)

WHEREFORE, the Motion to Dismiss is GRANTED. The Petition for Review is hereby
DISMISSED for lack of jurisdiction.

SO ORDERED.16

Aggrieved, petitioner moved for reconsideration but the motion was denied by
the First Division in its Resolution dated February 1, 2006.17

Proceedings before the CTA En Banc


157

On February 22, 2006, petitioner appealed the dismissal to the CTA En


Banc.18 The case was docketed as CTA EB No. 167.

Finding no reversible error in the Resolutions dated October 12, 2005 and
February 1, 2006 of the CTA First Division, the CTA En Banc denied the Petition
for Review19as well as petitioner’s Motion for Reconsideration.20

The CTA En Banc declared that it is absolutely necessary for the taxpayer to file
an administrative protest in order for the CTA to acquire jurisdiction. It
emphasized that an administrative protest is an integral part of the remedies
given to a taxpayer in challenging the legality or validity of an assessment.
According to the CTA En Banc, although there are exceptions to the doctrine
of exhaustion of administrative remedies, the instant case does not fall in any of
the exceptions.

Issue

Hence, the present recourse, where petitioner raises the lone issue of whether
the Formal Letter of Demand dated July 16, 2004 can be construed as a final
decision of the CIR appealable to the CTA under RA 9282.

Our Ruling

The petition is meritorious.

Section 7 of RA 9282 expressly provides that the CTA exercises exclusive


appellate jurisdiction to review by appeal decisions of the CIR in cases involving
disputed assessments

The CTA, being a court of special jurisdiction, can take cognizance only of
matters that are clearly within its jurisdiction.21 Section 7 of RA 9282 provides:

Sec. 7. Jurisdiction. — The CTA shall exercise:

(a) Exclusive appellate jurisdiction to review by appeal, as herein provided:

(1) Decisions of the Commissioner of Internal Revenue in cases involving


disputed assessments, refunds of internal revenue taxes, fees or other
charges, penalties in relation thereto, or other matters arising under the
National Internal Revenue Code or other laws administered by the Bureau
of Internal Revenue;

(2) Inaction by the Commissioner of Internal Revenue in cases involving


disputed assessments, refunds of internal revenue taxes, fees or other
charges, penalties in relation thereto, or other matters arising under the
National Internal Revenue Code or other laws administered by the Bureau
of Internal Revenue, where the National Internal Revenue Code provides
a specific period of action, in which case the inaction shall be deemed a
denial; (Emphasis supplied)

xxxx
158

The word "decisions" in the above quoted provision of RA 9282 has been
interpreted to mean the decisions of the CIR on the protest of the taxpayer
against the assessments.22 Corollary thereto, Section 228 of the National Internal
Revenue Code (NIRC) provides for the procedure for protesting an assessment.
It states:

SECTION 228. Protesting of Assessment. – When the Commissioner or his duly


authorized representative finds that proper taxes should be assessed, he shall
first notify the taxpayer of his findings: Provided, however, That a preassessment
notice shall not be required in the following cases:

(a) When the finding for any deficiency tax is the result of mathematical
error in the computation of the tax as appearing on the face of the return;
or

(b) When a discrepancy has been determined between the tax withheld
and the amount actually remitted by the withholding agent; or

(c) When a taxpayer who opted to claim a refund or tax credit of excess
creditable withholding tax for a taxable period was determined to have
carried over and automatically applied the same amount claimed
against the estimated tax liabilities for the taxable quarter or quarters of
the succeeding taxable year; or

(d) When the excise tax due on excisable articles has not been paid; or

(e) When an article locally purchased or imported by an exempt person,


such as, but not limited to, vehicles, capital equipment, machineries and
spare parts, has been sold, traded or transferred to non-exempt persons.

The taxpayers shall be informed in writing of the law and the facts on which the
assessment is made; otherwise, the assessment shall be void.

Within a period to be prescribed by implementing rules and regulations, the


taxpayer shall be required to respond to said notice. If the taxpayer fails to
respond, the Commissioner or his duly authorized representative shall issue an
assessment based on his findings.

Such assessment may be protested administratively by filing a request for


reconsideration or reinvestigation within thirty (30) days from receipt of the
assessment in such form and manner as may be prescribed by implementing
rules and regulations. Within sixty (60) days from filing of the protest, all relevant
supporting documents shall have been submitted; otherwise, the assessment
shall become final.

If the protest is denied in whole or in part, or is not acted upon within one
hundred eighty (180) days from submission of documents, the taxpayer
adversely affected by the decision or inaction may appeal to the Court of Tax
Appeals within thirty (30) days from receipt of the said decision, or from the
lapse of the one hundred eighty (180)-day period; otherwise, the decision shall
become final, executory and demandable.
159

In the instant case, petitioner timely filed a protest after receiving the PAN. In
response thereto, the BIR issued a Formal Letter of Demand with Assessment
Notices. Pursuant to Section 228 of the NIRC, the proper recourse of petitioner
was to dispute the assessments by filing an administrative protest within 30 days
from receipt thereof. Petitioner, however, did not protest the final assessment
notices. Instead, it filed a Petition for Review with the CTA. Thus, if we strictly
apply the rules, the dismissal of the Petition for Review by the CTA was proper.

The case is an exception to the


rule on exhaustion of administrative remedies

However, a careful reading of the Formal Letter of Demand with Assessment


Notices leads us to agree with petitioner that the instant case is an exception to
the rule on exhaustion of administrative remedies, i.e., estoppel on the part of
the administrative agency concerned.

In the case of Vda. De Tan v. Veterans Backpay Commission,23 the respondent


contended that before filing a petition with the court, petitioner should have
first exhausted all administrative remedies by appealing to the Office of the
President. However, we ruled that respondent was estopped from invoking the
rule on exhaustion of administrative remedies considering that in its Resolution, it
said, "The opinions promulgated by the Secretary of Justice are advisory in
nature, which may either be accepted or ignored by the office seeking the
opinion, and any aggrieved party has the court for recourse". The statement of
the respondent in said case led the petitioner to conclude that only a final
judicial ruling in her favor would be accepted by the Commission.

Similarly, in this case, we find the CIR estopped from claiming that the filing of
the Petition for Review was premature because petitioner failed to exhaust all
administrative remedies.

The Formal Letter of Demand with Assessment Notices reads:

Based on your letter-protest dated May 26, 2004, you alleged the following:

1. That the said assessment has already prescribed in accordance with


the provisions of Section 203 of the Tax Code.

2. That since the exemption of FCDUs from all taxes found in the Old Tax
Code has been deleted, the wording of Section 28(A)(7)(b) discloses that
there are no other taxes imposable upon FCDUs aside from the 10% Final
Income Tax.

Contrary to your allegation, the assessments covering GRT and DST for taxable
year 2001 has not prescribed for [sic] simply because no returns were filed, thus,
the three year prescriptive period has not lapsed.

With the implementation of the CTRP, the phrase "exempt from all taxes" was
deleted. Please refer to Section 27(D)(3) and 28(A)(7) of the new Tax Code.
Accordingly, you were assessed for deficiency gross receipts tax on onshore
income from foreign currency transactions in accordance with the rates
provided under Section 121 of the said Tax Code. Likewise, deficiency
160

documentary stamp taxes was [sic] also assessed on Loan Agreements, Bills
Purchased, Certificate of Deposits and related transactions pursuant to
Sections 180 and 181 of NIRC, as amended.

The 25% surcharge and 20% interest have been imposed pursuant to the
provision of Section 248(A) and 249(b), respectively, of the National Internal
Revenue Code, as amended.

It is requested that the above deficiency tax be paid immediately upon receipt
hereof, inclusive of penalties incident to delinquency. This is our final decision
based on investigation. If you disagree, you may appeal this final decision
within thirty (30) days from receipt hereof, otherwise said deficiency tax
assessment shall become final, executory and demandable.24 (Emphasis
supplied)

It appears from the foregoing demand letter that the CIR has already made a
final decision on the matter and that the remedy of petitioner is to appeal the
final decision within 30 days.

In Oceanic Wireless Network, Inc. v. Commissioner of Internal Revenue,25 we


considered the language used and the tenor of the letter sent to the taxpayer
as the final decision of the CIR.

In this case, records show that petitioner disputed the PAN but not the Formal
Letter of Demand with Assessment Notices. Nevertheless, we cannot blame
petitioner for not filing a protest against the Formal Letter of Demand with
Assessment Notices since the language used and the tenor of the demand
letter indicate that it is the final decision of the respondent on the matter. We
have time and again reminded the CIR to indicate, in a clear and unequivocal
language, whether his action on a disputed assessment constitutes his final
determination thereon in order for the taxpayer concerned to determine when
his or her right to appeal to the tax court accrues.26 Viewed in the light of the
foregoing, respondent is now estopped from claiming that he did not intend
the Formal Letter of Demand with Assessment Notices to be a final decision.

Moreover, we cannot ignore the fact that in the Formal Letter of Demand with
Assessment Notices, respondent used the word "appeal" instead of "protest",
"reinvestigation", or "reconsideration". Although there was no direct reference
for petitioner to bring the matter directly to the CTA, it cannot be denied that
the word "appeal" under prevailing tax laws refers to the filing of a Petition for
Review with the CTA. As aptly pointed out by petitioner, under Section 228 of
the NIRC, the terms "protest", "reinvestigation" and "reconsideration" refer to the
administrative remedies a taxpayer may take before the CIR, while the term
"appeal" refers to the remedy available to the taxpayer before the CTA.
Section 9 of RA 9282, amending Section 11 of RA 1125,27 likewise uses the term
"appeal" when referring to the action a taxpayer must take when adversely
affected by a decision, ruling, or inaction of the CIR. As we see it then,
petitioner in appealing the Formal Letter of Demand with Assessment Notices to
the CTA merely took the cue from respondent. Besides, any doubt in the
interpretation or use of the word "appeal" in the Formal Letter of Demand with
161

Assessment Notices should be resolved in favor of petitioner, and not the


respondent who caused the confusion.

To be clear, we are not disregarding the rules of procedure under Section 228
of the NIRC, as implemented by Section 3 of BIR Revenue Regulations No. 12-
99.28 It is the Formal Letter of Demand and Assessment Notice that must be
administratively protested or disputed within 30 days, and not the PAN. Neither
are we deviating from our pronouncement in St. Stephen’s Chinese Girl’s
School v. Collector of Internal Revenue,29 that the counting of the 30 days
within which to institute an appeal in the CTA commences from the date of
receipt of the decision of the CIR on the disputed assessment, not from the
date the assessment was issued.1avvphi1

What we are saying in this particular case is that, the Formal Letter of Demand
with Assessment Notices which was not administratively protested by the
petitioner can be considered a final decision of the CIR appealable to the CTA
because the words used, specifically the words "final decision" and "appeal",
taken together led petitioner to believe that the Formal Letter of Demand with
Assessment Notices was in fact the final decision of the CIR on the letter-protest
it filed and that the available remedy was to appeal the same to the CTA.

We note, however, that during the pendency of the instant case, petitioner
availed of the provisions of Revenue Regulations No. 30-2002 and its
implementing Revenue Memorandum Order by submitting an offer of
compromise for the settlement of the GRT, DST and VAT for the period 1998-
2003, as evidenced by a Certificate of Availment dated November 21,
2007.30 Accordingly, there is no reason to reinstate the Petition for Review in CTA
Case No. 7062.

WHEREFORE, the petition is hereby GRANTED. The assailed August 23, 2006
Decision and the October 17, 2006 Resolution of the Court of Tax Appeals
are REVERSED and SET ASIDE. The Petition for Review in CTA Case No. 7062 is
hereby DISMISSED based solely on the Bureau of Internal Revenue’s
acceptance of petitioner’s offer of compromise for the settlement of the gross
receipts tax, documentary stamp tax and value added tax, for the years 1998-
2003.

SO ORDERED

G.R. No. L-20569 August 23, 1974

JOSE B. AZNAR, in his capacity as Administrator of the Estate of the deceased,


Matias H. Aznar, petitioner,
vs.
COURT OF TAX APPEALS and COLLECTOR OF INTERNAL REVENUE, respondents.

Sato, Enad Garcia for petitioner.

Office of the Solicitor General Arturo A. Alafriz, Solicitor Alejandro B. Afurong


and Special Attorney Librada R. Natividad for respondents.
162

ESGUERRA, J.:p

Petitioner, as administrator of the estate of the deceased, Matias H. Aznar,


seeks a review and nullification of the decision of the Court of Tax Appeals in
C.T.A. Case No. 109, modifying the decision of respondent Commissioner of
Internal Revenue and ordering the petitioner to pay the government the sum of
P227,691.77 representing deficiency income taxes for the years 1946 to 1951,
inclusive, with the condition that if the said amount is not paid within thirty days
from the date the decision becomes final, there shall be added to the unpaid
amount the surcharge of 5%, plus interest at the rate of 12% per annum from
the date of delinquency to the date of payment, in accordance with Section
51 of the National Internal Revenue Code, plus costs against the petitioner.

It is established that the late Matias H. Aznar who died on May 18, 1958,
predecessor in interest of herein petitioner, during his lifetime as a resident of
Cebu City, filed his income tax returns on the cash and disbursement basis,
reporting therein the following:

Year Net Amount Exhibit


Income of Tax
Paid
1945 P12,822.00 P114.66 pp.
85-88
B.I.R.
rec.
1946 9,910.94 114.66 38-A
(pp.
329-
332
B.I.R
rec.)
1947 10,200.00 132.00 39
(pp.
75-78
B.I.R
rec.)
1948 9,148.34 68.90 40
(pp.
70-73
B.I.R.
rec.)
1949 8,990.66 59.72 41
(pp.
64-67
B.I.R.
rec.)
163

1950 8,364.50 28.22 42


(pp.
59-62,
BIR
rec.)

1951 6,800.00 none 43


(pp.
54-57
BIR
rec.).

The Commissioner of Internal Revenue having his doubts on the veracity of the
reported income of one obviously wealthy, pursuant to the authority granted
him by Section 38 of the National Internal Revenue Code, caused B.I.R.
Examiner Honorio Guerrero to ascertain the taxpayer's true income for said
years by using the net worth and expenditures method of tax investigation. The
assets and liabilities of the taxpayer during the above-mentioned years were
ascertained and it was discovered that from 1946 to 1951, his net worth had
increased every year, which increases in net worth was very much more than
the income reported during said years. The findings clearly indicated that the
taxpayer did not declare correctly the income reported in his income tax
returns for the aforesaid years.

Based on the above findings of Examiner Guerrero, respondent Commissioner,


in his letter dated November 28, 1952, notified the taxpayer (Matias H. Aznar) of
the assessed tax delinquency to the amount of P723,032.66, plus compromise
penalty. The taxpayer requested a reinvestigation which was granted for the
purpose of verifying the merits of the various objections of the taxpayer to the
deficiency income tax assessment of November 28, 1952.

After the reinvestigation, another deficiency assessment to the reduced


amount of P381,096.07 dated February 16, 1955, superseded the previous
assessment and notice thereof was received by Matias H. Aznar on March 2,
1955.

The new deficiency assessment was based on the following computations:

1946

Net income per return ........................ P9,910.94


Add: Under declared income .............. 22,559.94
Net income per investigation............... 32,470.45

Deduct: Income tax liability


per return as assessed ...................................................... 114.66
Balance of tax due ........................................................... P3,687.10
Add: 50% surcharge ........................................................ 1,843.55
DEFICIENCY INCOME TAX ...................................... P5,530.65

1947
164

Net income per return ..................................................... P10,200.00


Add: Under declared income ............................................ 90,413.56
Net income per reinvestigation ....................................... P100,613.56
Deduct: Personal and additional exemption ...................... 7,000.00
Amount of income subject to tax ...................................... P93,613.56
Total tax liability ............................................................... P24,753.15
Deduct: Income tax liability per return as assessed ............ 132.00
Balance of tax due ........................................................... P24,621.15
Add: 50% surcharge ........................................................ 12,310.58 DEFICIENCY
INCOME TAX ...................................... P36,931.73

1948

Net income per return ...................................................... P9,148.34


Add: Under declared income ............................................. 15,624.63
Net income per reinvestigation .......................................... P24,772.97
Deduct: Personal and additional exemptions ...................... 7,000.00
Amount of income subject to tax ....................................... P17,772.97
Total tax liability ............................................................... 2,201.40
Deduct: Income tax liability per return as assessed ............ 68.90
Balance of tax due ........................................................... P2,132.500
Add: 50% surcharge ........................................................ 1,066.25 DEFICIENCY
INCOME TAX ...................................... P3,198.75

1949

Net income per return ....................................................... P9,990.66


Add: Under declared income ............................................. 105,418.53
Net income per reinvestigation .......................................... 114,409.19
Deduct: Personal and additional exemptions ...................... P7,000.00
Amount of income subject to tax ....................................... P107,409.19
Total tax liability ............................................................... P30,143.68
Deduct: Income tax liability per return as assessed ............. 59.72
Balance of tax due ............................................................ P30,083.96
Add: 50% surcharge ......................................................... 15,041.98 DEFICIENCY
INCOME TAX ....................................... P45,125.94

1950

Net income per return ....................................................... P8,364.50


Add: Under declared income ............................................. 365,578.76
Net income per reinvestigation .......................................... P373,943.26
Deduct: Personal and additional exemptions ...................... 7,800.00
Amount of income subject to tax ....................................... P366,143.26
Total tax liability ............................................................... P185,883.00
Deduct: Income tax liability per return as assessed ............. 28.00
Balance of tax due ............................................................ P185,855.00
Add: 50% surcharge ......................................................... 92,928.00 DEFICIENCY
INCOME TAX ....................................... P278,783.00

1951
165

Net income per return ........................................................ P6,800.00


Add: Under declared income ............................................... 33,355.80
Net income per reinvestigation ............................................ P40,155.80
Deduct: Personal and additional exemptions ........................ 7,200.00
Amount of income subject to tax ......................................... P32,955.80
Total tax liability .................................................................. P7,684.00
Deduct: Income tax liability per return as assessed ............... - o - .
Balance of tax due .............................................................. P7,684.00
Add: 50% surcharge ........................................................... 3,842.00 DEFICIENCY
INCOME TAX .......................................... P11,526.00

SUMMARY

1946 .... P5,530.65


1947 .... 36,931.73
1948 .... 3,198.75
1949 .... 45,125.94
1950 .... 278,783.00
1951 .... 11,526.00
Total .... P381,096.07

In determining the unreported income, the respondent Commissioner of


Internal Revenue resorted to the networth method which is based on the
following computations:

1945

Real estate inventory ................................ P64,738.00


Other assets ............................................. 37,606.87
Total assets ............................................ P102,344.87
Less: Depreciation allowed ...................... 2,027.00
Networth as of Dec. 31, 1945 ................ P100,316.97

1946

Real estate inventory ................................. P86,944.18


Other assets ............................................. 60,801.65
Total assets ............................................. P147,745.83
Less: Depreciation allowed ...................... 4,875.41
Net assets ................................................ P142,870.42
Less: Liabilities .................. P17,000.00
Net Worth as of
Jan. 1, 1946 ................... P100,316.97 P117,316.97
Increase in networth ................................. 25,553.45
Add: Estimated living expenses ................. 6,917.00
Net income .............................................. P32,470.45
166

1947

Real estate inventory .................................. P237,824.18


Other assets ............................................... 54,495.52
Total assets ............................................... P292,319.70
Less: Depreciation allowed ......................... 12,835.72
Net assets .................................................. 279,483.98
Less: Liabilities ................... P60,000.00
Networth as of
Jan. 1, 1947 ........................ 125,870.42 P185,870.42
Increase in networth ................................... P93,613.56
Add: Estimated living expenses ................... 7,000.00
Net income ................................................P100,613.56

1948

Real estate inventory .................................. P244,824.18


Other assets .............................................. 118,720.60
Total assets ............................................... P363,544.78
Less: Depreciation allowed ........................ 20,936.03
Net assets ................................................. P342,608.75
Less: Liabilities ................... P105,351.80
Networth as of
Jan. 1, 1948 ...................... 219,483.98 P324,835.78
Increase in networth ................................... P17,772.97
Add: Estimated living expenses ................... 7,000.00
Net income ................................................ P24,772.97

1949

Real estate inventory ................................. P400,515.52


Investment in schools and other colleges .... 23,105.29
Other assets ............................................. 70,311.00
Total assets ............................................... P493,931.81
Less: Depreciation allowed ........................ 32,657.08
Net assets ................................................. P461.274.73
Less; Liabilities .................. P116,608.59
Networth as of
Jan. 1, 1949 ...................... 237,256.95 P353,865.54
Increase in networth .................................. P107,409.19
Add: Estimated living expenses .................. 7,000.00
Net income ............................................... P114,409.19

1950

Real estate inventory .................................. P412,465.52


Investment in Schools and
other colleges ................................ 193,460.99
October assets .......................................... 310,788.87
Total assets ............................................... P916,715.38
Less; Depreciation allowed ........................ 47,561.99
167

Net assets ................................................. P869,153.39


Less: Liabilities .................. P158,343.99
Networth as of Jan. 1, 1950 ... 344,666.14 P503,010.13
Increase in networth ................................... P366,143.26
Add: Estimated living expenses ................... 7,800.00
Net income ................................................. P373,943.26

1951

Real estate inventory ................................... P412,465.52


Investment in schools and other colleges ..... 214,016.21
Other assets ............................................... 320,209.40
Total assets ................................................ P946,691.13
Less: Depreciation allowed ......................... 62,466.90
Net assets .................................................. P884,224.23
Less: Liabilities ........................................... P140,459.03
Networth as of
Jan. 1, 1951 ................ 710,809.40 P851,268.43
Increase in networth .................................... P32,955.80
Add: Estimated living expenses .................... 7,200.00
Net income ................................................. P40,155.80

(Exh. 45-B, BIR rec. p. 188)

On February 20, 1953, respondent Commissioner of Internal Revenue, thru the


City Treasurer of Cebu, placed the properties of Matias H. Aznar under distraint
and levy to secure payment of the deficiency income tax in question. Matias H.
Aznar filed his petition for review of the case with the Court of Tax Appeals on
April 1, 1955, with a subsequent petition immediately thereafter to restrain
respondent from collecting the deficiency tax by summary method, the latter
petition being granted on February 8, 1956, per C.T.A. resolution, without
requiring petitioner to file a bond. Upon review, this Court set aside the C.T.A.
resolution and required the petitioner to deposit with the Court of Tax Appeals
the amount demanded by the Commissioner of Internal Revenue for the years
1949 to 1951 or furnish a surety bond for not more than double the amount.

On March 5, 1962, in a decision signed by the presiding judge and the two
associate judges of the Court of Tax Appeals, the lower court concluded that
the tax liability of the late Matias H. Aznar for the year 1946 to 1951, inclusive
should be P227,788.64 minus P96.87 representing the tax credit for 1945, or
P227,691.77, computed as follows:

1946

Net income per return .............................................. P9,910.94


Add: Under declared income ..................................... 22,559.51
Net income ............................................................ P32,470.45
Less: Personal and additional exemptions .................. 6,917.00
Income subject to tax ............................................. P25,553.45
Tax due thereon ...................................................... P3,801.76
Less: Tax already assessed ...................................... 114.66
168

Balance of tax due .................................................... P3,687.10


Add: 50% surcharge ................................................. 1,843.55
Deficiency income tax ................................................ P5,530.65

1947

Net income per return ............................................ P10,200.00


Add: Under declared income .................................. 57,551.19
Net income ........................................................... P67,751.19
Less: Personal and additional exemptions ............... 7,000.00
Income subject to tax ............................................. P60,751.19
Tax due thereon ..................................................... P13,420.38
Less: Tax already assessed ..................................... P132.00
Balance of tax due ................................................... P13,288.38
Add: 50% surcharge ................................................ 6,644.19
Deficiency income tax .............................................. P19,932.57

1948

Net income per return .............................................. P9,148.34


Add: Under declared income ..................................... 8,732.10
Net income ............................................................ P17,880.44
Less: Personal and additional exemptions ................. 7,000.00
Income subject to tax .............................................. P10,880.44
Tax due thereon ...................................................... P1,029.67
Less: Tax already assessed ....................................... 68.90
Balance of tax due .................................................... 960.77
Add: 50% surcharge ................................................. 480.38
Deficiency income tax ............................................... P1,441.15

1949

Net income per return ................................................. P8,990.66


Add: under declared income ......................................... 43,718.53
Net income ............................................................... P52,709.19
Less: Personal and additional exemptions .................... 7,000.00
Income subject to tax ................................................. P45,709.19
Tax due thereon ......................................................... P8,978.57
Less: Tax already assessed ......................................... 59.72
Balance of tax due ....................................................... P8,918.85
Add: 50% surcharge .................................................... 4,459.42
Deficiency income tax ................................................. P13,378.27

1950

Net income per return .................................................. P6,800.00


Add: Under declared income ......................................... 33,355.80
Net income ................................................................. P40,155.80
Less: Personal and additional exemptions ...................... 7,200.00
Income subject to tax .................................................. P32,955.80
Tax due thereon ........................................................... P7,684.00
169

Less: Tax already assessed ........................................... -o- .


Balance of tax due ........................................................ P7,684.00
Add: 50% surcharge .................................................... 3,842.00
Deficiency income tax .................................................. P11,526.00

1951

Net income per return ................................................... P8,364.50


Add: Under declared income ........................................ 246,449.06
Net income ............................................................... P254.813.56
Less: Personal and additional exemptions .................... 7,800.00
Income subject to tax ................................................ P247,013.56
Tax due thereon ........................................................ P117,348.00
Less: Tax already assessed ........................................ 28.00
Balance of tax due ..................................................... P117,320.00
Add: 50% surcharge .................................................. 58,660.00
Deficiency income tax ................................................ P175 980.00

SUMMARY

1946 P5,530.65

1947 19,932.57

1948 1,441.15

1949 13,378.27

1950 175,980.00

1951 11,526.00

P227,788.64.

The first vital issue to be decided here is whether or not the right of the
Commissioner of Internal Revenue to assess deficiency income taxes of the late
Matias H. Aznar for the years 1946, 1947, and 1948 had already prescribed at
the time the assessment was made on November 28, 1952.

Petitioner's contention is that the provision of law applicable to this case is the
period of five years limitation upon assessment and collection from the filing of
the returns provided for in See. 331 of the National Internal Revenue Code. He
argues that since the 1946 income tax return could be presumed filed before
March 1, 1947 and the notice of final and last assessment was received by the
taxpayer on March 2, 1955, a period of about 8 years had elapsed and the five
year period provided by law (Sec. 331 of the National Internal Revenue Code)
had already expired. The same argument is advanced on the taxpayer's return
for 1947, which was filed on March 1, 1948, and the return for 1948, which was
filed on February 28, 1949. Respondents, on the other hand, are of the firm
belief that regarding the prescriptive period for assessment of tax returns,
170

Section 332 of the National Internal Revenue Code should apply because, as in
this case, "(a) In the case of a false or fraudulent return with intent to evade tax
or of a failure to file a return, the tax may be assessed, or a proceeding in court
for the collection of such tax may be begun without assessment, at any time
within ten years after the discovery of the falsity, fraud or omission" (Sec. 332 (a)
of the NIRC).

Petitioner argues that Sec. 332 of the NIRC does not apply because the
taxpayer did not file false and fraudulent returns with intent to evade tax, while
respondent Commissioner of Internal Revenue insists contrariwise, with
respondent Court of Tax Appeals concluding that the very "substantial under
declarations of income for six consecutive years eloquently demonstrate the
falsity or fraudulence of the income tax returns with an intent to evade the
payment of tax."

To our minds we can dispense with these controversial arguments on facts,


although we do not deny that the findings of facts by the Court of Tax Appeals,
supported as they are by very substantial evidence, carry great weight, by
resorting to a proper interpretation of Section 332 of the NIRC. We believe that
the proper and reasonable interpretation of said provision should be that in the
three different cases of (1) false return, (2) fraudulent return with intent to
evade tax, (3) failure to file a return, the tax may be assessed, or a proceeding
in court for the collection of such tax may be begun without assessment, at any
time within ten years after the discovery of the (1) falsity, (2) fraud, (3) omission.
Our stand that the law should be interpreted to mean a separation of the three
different situations of false return, fraudulent return with intent to evade tax,
and failure to file a return is strengthened immeasurably by the last portion of
the provision which segregates the situations into three different classes, namely
"falsity", "fraud" and "omission". That there is a difference between "false return"
and "fraudulent return" cannot be denied. While the first merely implies
deviation from the truth, whether intentional or not, the second implies
intentional or deceitful entry with intent to evade the taxes due.

The ordinary period of prescription of 5 years within which to assess tax liabilities
under Sec. 331 of the NIRC should be applicable to normal circumstances, but
whenever the government is placed at a disadvantage so as to prevent its
lawful agents from proper assessment of tax liabilities due to false returns,
fraudulent return intended to evade payment of tax or failure to file returns, the
period of ten years provided for in Sec. 332 (a) NIRC, from the time of the
discovery of the falsity, fraud or omission even seems to be inadequate and
should be the one enforced.

There being undoubtedly false tax returns in this case, We affirm the conclusion
of the respondent Court of Tax Appeals that Sec. 332 (a) of the NIRC should
apply and that the period of ten years within which to assess petitioner's tax
liability had not expired at the time said assessment was made.

II

As to the alleged errors committed by the Court of Tax Appeals in not


deducting from the alleged undeclared income of the taxpayer for 1946 the
171

proceeds from the sale of jewelries valued at P30,000; in not excluding from
other schedules of assets of the taxpayer (a) accounts receivable from
customers in the amount of P38,000 for 1948, P126,816.50 for 1950, and
provisions for doubtful accounts in the amount of P41,810.56 for 1950; (b) over
valuation of hospital and dental buildings for 1949 in the amount of P32,000 and
P6,191.32 respectively; (c) investment in hollow block business in the amount of
P8,603.22 for 1949; (d) over valuation of surplus goods in the amount of P23,000
for the year 1949; (e) various lands and buildings included in the schedule of
assets for the years 1950 and 1951 in the total amount of P243,717.42 for 1950
and P62,564.00 for 1951, these issues would depend for their resolution on
determination of questions of facts based on an evaluation of evidence, and
the general rule is that the findings of fact of the Court of Tax Appeals
supported by substantial evidence should not be disturbed upon review of its
decision (Section 2, Rule 44, Rules of Court).

On the question of the alleged sale of P30,000 worth of jewelries in 1946, which
amount petitioner contends should be deducted from the taxpayer's net worth
as of December 31, 1946, the record shows that Matias H. Aznar, when
interviewed by B.I.R. Examiner Guerrero, stated that at the beginning of 1945 he
had P60,000 worth of jewelries inherited from his ancestors and were disposed
off as follows: 1945, P10,000; 1946, P20,000; 1947, P10,000; 1948, P10,000; 1949,
P7,000; (Report of B.I.R. Examiner Guerrero, B.I.R. rec. pp. 90-94).

During the hearing of this case in the Court of Tax Appeals, petitioner's
accountant testified that on January 1, 1945, Matias H. Aznar had jewelries
worth P60,000 which were acquired by purchase during the Japanese
occupation (World War II) and sold on various occasions, as follows: 1945,
P5,000 and 1946, P30,000. To corroborate the testimony of the accountant, Mrs.
Ramona Agustines testified that she bought from the wife of Matias H. Aznar in
1946 a diamond ring and a pair of earrings for P30,000; and in 1947 a wrist
watch with diamonds, together with antique jewelries, for P15,000. Matias H.
Aznar, on the other hand testified that in 1945, his wife sold to Sards Parino
jewelries for P5,000 and question, Mr. Aznar stated that his transaction with
Sards Parino, with respect to the sale of jewelries, amounted to P15,000.

The lower court did not err in finding material inconsistencies in the testimonies
of Matias H. Aznar and his witnesses with respect to the values of the jewelries
allegedly disposed off as stated by the witnesses. Thus, Mr. Aznar stated to the
B.I.R. examiner that jewelries worth P10,000 were sold in 1945, while his own
accountant testified that the same jewelries were sold for only P5,000. Mr. Aznar
also testified that Mrs. Agustines purchased from his wife jewelries for P35,000,
and yet Mrs. Agustines herself testified that she bought jewelries for P30,000 and
P15,000 on two occasions, or a total of P45,000.

We do not see any plausible reason to challenge the fundamentally sound


basis advanced by the Court of Tax Appeals in considering the inconsistencies
of the witnesses' testimony as material, in the following words:

We do not say that witnesses testifying on the same transaction


should give identical testimonies. Because of the frailties and the
limitations of the human mind, witnesses' statements are apt to be
172

inconsistent in certain points, but usually the inconsistencies refer to


the minor phases of the transaction. It is the insignificance of the
detail of an occurrence that fails to impress the human mind. When
that same mind, made to recall what actually happened, the
significant point which it failed to take note is naturally left out. But it
is otherwise as regards significant matters, for they leave indelible
imprints upon the human mind. Hence, testimonial inconsistencies on
the minor details of an occurrence are dismissed lightly by the courts,
while discrepancies on significant points are taken seriously and
weigh adversely to the party affected thereby.

There is no sound basis for deviating from the lower court's conclusion that:
"Taxwise in view of the aforesaid inconsistencies, which we deem material and
significant, we dismiss as without factual basis petitioner's allegation that
jewelries form part of his inventory of assets for the purpose of establishing his
net worth at the beginning of 1946."

As to the accounts receivable from the United States government for the
amount of P38,254.90, representing a claim for goods commandered by the
U.S. Army during World War II, and which amount petitioner claimed should be
included in his net worth as of January 1, 1946, the Court of Tax Appeals
correctly concluded that the uncontradicted evidence showed that "the
collectible accounts of Mr. Aznar from the U.S. Government in the sum of
P38,254.90 should be added to his assets (under accounts receivable) as of
January 1, 1946. As of December 31, 1947, and December 31, 1948, the years
within which the accounts were paid to him, the 'accounts receivable shall
decrease by P31,362.37 and P6,892.53, respectively."

Regarding a house in Talisay Cebu, (covered by Tax Declaration No. 8165)


which was listed as an asset during the years 1945 and 1947 to 1951, but which
was not listed as an asset in 1946 because of a notation in the tax declaration
that it was reconstructed in 1947, the lower court correctly concluded that the
reconstruction of the property did not render it valueless during the time it was
being reconstructed and consequently it should be listed as an asset as of
January 1, 1946, with the same valuation as in 1945, that is P1,500.

On the question of accounts receivable from customers in the amount of


P38,000 for 1948, and P123,816.58 for the years 1950 and 1951, which were
included in the assets of Mr. Aznar for those years by the respondent
Commissioner of Internal Revenue, it is very clear that those figures were taken
from the statements (Exhs. 31 and 32) filed by Mr. Matias H. Aznar with the
Philippine National Bank when he was intending to obtain a loan. These
statements were under oath and the natural implication is that the information
therein reflected must be the true and accurate financial condition of the one
who executed those statements. To believe the petitioner's argument that the
late Mr. Aznar included those figures in his sworn statement only for the purpose
of obtaining a bigger credit from the bank is to cast suspicion on the character
of a man who can no longer defend himself. It would be as if pointing the
finger of accusation on the late Mr. Aznar that he intentionally falsified his sworn
statements (Exhs. 31 and 32) to make it appear that there were non-existent
accounts receivable just to increase his assets by fictitious entries so that his
173

credit with the Philippine National Bank could be enhanced. Besides, We do


not lose sight of the fact that those statements (Exhs. 31 and 32) were executed
before this tax controversy arose and the disputable presumptions that a
person is innocent of crime or wrong; that a person intends the ordinary
consequences of his voluntary act; that a person takes ordinary care of his
concerns; that private transaction have been fair and regular; that the ordinary
course of business has been followed; that things have happened according to
the ordinary course of nature and the ordinary habits of life; that the law has
been obeyed (Sec. 5, (a), (c), (d), (p), (q), (z), (ff), Rule 131 of the Rules of
Court), together with the conclusive presumption that "whenever a party has,
by his own declaration, act, or omission, intentionally and deliberately led
another to believe a particular thing true, and to act upon such belief, he
cannot, in any litigation arising out of such declaration, act or omission, be
permitted to falsify it" (Sec. 3 (a), Rule 131, Rules of Court), convincingly indicate
that the accounts receivable stated by Mr. Aznar in Exhibits 31 and 32 were
true, in existence, and accurate to the very amounts mentioned.

There is no merit to petitioners argument that those statements were only for the
purpose of obtaining a bigger credit from the bank (impliedly stating that those
statements were false) and those accounts were allegedly back accounts of
students of the Southwestern Colleges and were worthless, and if collected,
would go to the funds of the school. The statement of the late Mr. Aznar that
they were accounts receivable from customers should prevail over the mere
allegation of petitioner, unsupported as they are by convincing evidence.
There is no reason to disturb the lower court's conclusion that the amounts of
P38,000 and P123,816.58 were accounts receivable from customers and as
such must be included as petitioner's assets for the years indicated.

As to the questions of doubtful accounts (bad debts), for the amount of


P41,810.56, it is clear that said amount is taken from Exhibit 31, the sworn
statement of financial condition filed by Mr. Matias H. Aznar with the Philippine
National Bank. The lower court did not commit any error in again giving much
weight to the statement of Mr. Aznar and in concluding that inasmuch as this is
an item separate and apart from the taxpayer's accounts receivable and non-
deductible expense, it should be reverted to the accounts receivable and,
consequently, considered as an asset in 1950.

On the alleged over valuation of two buildings (hospital building which


respondent Commissioner of Internal Revenue listed as an asset from 1949-1951
at the basic valuation of P130,000, and which petitioner claims to be over
valued by P32,000; dentistry building valued by respondent Commissioner of
Internal Revenue at P36,191.34, which petitioner claims to be over valued by
P6,191.34), We find no sufficient reason to alter the conclusion of respondent
Court of Tax Appeals sustaining the respondent Commissioner of Internal
Revenue's valuation of both properties.

Respondent Commissioner of Internal Revenue based his valuation of the


hospital building on the representation of Mr. Matias H. Aznar himself who, in his
letter (Exh. 35) to the Philippine National Bank dated September 5, 1949, stated
that the hospital building cost him P132,000. However in view of the effect of a
typhoon in 1949 upon the building, the value allowed was P130,000. Exhibit 35,
174

contrary to petitioner's contention, should be given probative value because,


although it is an unsigned plain copy, that exhibit was taken by the
investigating examiner of the B.I.R. from the files of the Southwestern Colleges
and formed part of his report of investigation as a public official. The estimates
of an architect and a civil engineer who agreed that a value of P84,240 is fair
for the hospital building, made years after the building was constructed,
cannot prevail over the petitioner's own estimate of his property's value.

Respondent Commissioner of Internal Revenue's valuation of P36,191.34 of the


Dentistry Building is based on the letter of Mr. and Mrs. Matias H. Aznar to the
Southwestern Colleges, dated December 15, 1950, which is embodied in the
minutes of the meeting of the Board of Trustees of the Southwestern Colleges
held on May 7, 1951 (Exhibit G-1). In Exhibit 26 A, which is the cash book of the
Southwestern Colleges, this building was listed as of the same amount.
Petitioner's estimate of P30,000 for this building, based on Architect Paca's
opinion, cannot stand against the owner's estimate and that which appears in
the cash book of the Southwestern Colleges, if we take into consideration that
the owner's (Mr. Matias H. Aznar) letter was written long before this tax
proceeding was initiated, while architect Paca's estimate was made upon
petitioner's request solely for the purpose of evidence in this tax case.

In the inventory of assets of petitioner, respondent Commissioner of Internal


Revenue included the administrative building valued at P19,200 for the years
1947 and 1948, and P16,700 for the years 1949 to 1951; and a high school
building valued at P48,000 for 1947 and 1948, and P45,000 for 1949, 1950 and
1951. The reduced valuation for the latter years are due to allowance for partial
loss resulting from the 1949 typhoon. Petitioner did not question the inclusion of
these buildings in the inventory for the years prior to 1950, but objected to their
inclusion as assets as of January 1, 1950, because both buildings were
destroyed by a typhoon in November of 1949. There is sufficient evidence (Exh.
G-1, affidavit of Jesus S. Intan, employee in the office of City Assessor of Cebu
City, Exh. 18, Mr. Intan's testimony, a copy of a letter of the City Assessor of
Cebu City) to prove that the two buildings were really destroyed by typhoon in
1949 and, therefore, should be eliminated from the petitioner's inventory of
assets beginning December 31, 1949.

On the issue of investment in the hollow blocks business, We see no compelling


reason to alter the lower court's conclusion that "whatever was spent in the
hollow blocks business is an investment, and being an investment, the same
should be treated as an asset. With respect to the amount representing the
value of the building, there is no duplication in the listing as the inventory of real
property does not include the building in question."

Respondent Commissioner of Internal Revenue included in the inventory, under


the heading of other asset, the amount of P8,663.22, treated as investment in
the hollow block business. Petitioner objects to the inclusion of P1,683.42 which
was spent on the building and in the business and of P674.35 which was spent
for labor, fuel, raw materials, office supplies etc., contending that the former
amount is a duplication of inventory (included among the list of properties) and
the latter is a business expense which should be eliminated from the list of
assets.
175

The inclusion of expenses (labor and raw materials) as part of the hollow block
business is sanctioned in the inventory method of tax verification. It is a sound
accounting practice to include raw materials that will be used for future
manufacture. Inclusion of direct labor is also proper, as all these items are to be
embodied in a summary of assets (investment by the taxpayer credited to his
capital account as reflected in Exhibit 72-A, which is a working sheet with
entries taken from the journal of the petitioner concerning his hollow blocks
business). There is no evidence to show that there was duplication in the
inclusion of the building used for hollow blocks business as part of petitioner's
investment as this building was not included in the listing of real properties of
petitioner (Exh. 45-C p. 187 B.I.R. rec.).

As to the question of the real value of the surplus goods purchased by Mr.
Matias H. Aznar from the U.S. Army, the best evidence, as observed correctly by
the lower court, is the statement of Mr. Matias H. Aznar, himself, as appearing
Exh. 35 (copy of a letter dated September 5, 1949 to the Philippine National
Bank), to the effect "as part of my assets I have different merchandise from
Warehouse 35, Tacloban, Leyte at a total cost of P43,000.00 and valued at no
less than P20,000 at present market value." Petitioner's claim that the goods
should be valued at only P20,000 in accordance with an alleged invoice is not
supported by evidence since the invoice was not presented as exhibit. The
lower court's act in giving more credence to the statement of Mr. Aznar cannot
be questioned in the light of clear indications that it was never controverted
and it was given at a time long before the tax controversy arose.

The last issue on propriety of inclusion in petitioner's assets made by respondent


Commissioner of Internal Revenue concerns several buildings which were
included in the list of petitioner's assets as of December 31, 1950. Petitioner
contends that those buildings were conveyed and ceded to Southwestern
Colleges on December 15, 1950, in consideration of P100,723.99 to be paid in
cash. The value of the different buildings are listed as: hospital building,
P130,000; gymnasium, P43,000; dentistry building, P36,191.34; bodega 1,
P781.18; bodega 2, P7,250; college of law, P10,950; laboratory building, P8,164;
home economics, P5,621; morgue, P2,400; science building, P23,600; faculty
house, P5,760. It is suggested that the value of the buildings be eliminated from
the real estate inventory and the sum of P100,723.99 be included as asset as of
December 31, 1950.

The lower court could not find any evidence of said alleged transfer of
ownership from the taxpayer to the Southwestern Colleges as of December 15,
1950, an allegation which if true could easily be proven. What is evident is that
those buildings were used by the Southwestern Colleges. It is true that Exhibit G-
1 shows that Mr. and Mrs. Matias H. Aznar offered those properties in exchange
for shares of stocks of the Southwestern Colleges, and Exhibit "G" which is the
minutes of the meeting of the Board of Trustees of the Southwestern Colleges
held on August 6, 1951, shows that Mr. Aznar was amenable to the value fixed
by the board of trustees and that he requested to be paid in cash instead of
shares of stock. But those are not sufficient evidence to prove that transfer of
ownership actually happened on December 15, 1950. Hence, the lower court
did not commit any error in sustaining the respondent Commissioner of Internal
176

Revenue's act of including those buildings as part of the assets of petitioner as


of December 31, 1950.

Petitioner also contends that properties allegedly ceded to the Southwestern


Colleges in 1951 for P150,000 worth of shares of stocks, consisting of: land,
P22,684; house, P13,700; group of houses, P8,000; building, P12,000; nurses
home, P4,100; nurses home, P2,080, should be excluded from the inventory of
assets as of December 31, 1951. The evidence (Exh. H), however, clearly shows
that said properties were formally conveyed to the Southwestern Colleges only
on September 25, 1952. Undoubtedly, petitioner was the owner of those
properties prior to September 25, 1952 and said properties should form part of
his assets as of December 31, 1951.

The uncontested portions of the lower court's decision consisting of its


conclusions that library books valued at P7,041.03, appearing in a journal of the
Southwestern Colleges marked as' Exhibit 25-A, being an investment, should be
treated as an asset beginning December 31, 1950; that the expenses for
construction to the amount of P113,353.70, which were spent for the
improvement of the buildings appearing in Exhibit 24 are deemed absorbed in
the increased value of the buildings as appraised by respondent Commissioner
of Internal Revenue at cost after improvements were made, and should be
taken out as additional assets; that the amount receivable of P5,776 from a
certain Benito Chan should be treated as petitioner's asset but the amount of
P5,776 representing the value of a house and lot given as collateral to secure
said loan should not be considered as an asset of petitioner since to do so
would result in a glaring duplication of items, are all affirmed. There seems to be
no controversy as to the rest of the items listed in the inventory of assets.

III

The second issue which appears to be of vital importance in this case centers
on the lower court's imposition of the fraud penalty (surcharge of 50%
authorized in Section 72 of the Tax Code). The petitioner insists that there might
have been false returns by mistake filed by Mr. Matias H. Aznar as those returns
were prepared by his accountant employees, but there were no proven
fraudulent returns with intent to evade taxes that would justify the imposition of
the 50% surcharge authorized by law as fraud penalty.

The lower court based its conclusion that the 50% fraud penalty must be
imposed on the following reasoning: .

It appears that Matias H. Aznar declared net income of P9,910.94,


P10,200, P9,148.34, P8,990.66, P8,364.50 and P6,800 for the years 1946,
1947, 1948, 1949, 1950 and 1951, respectively. Using the net worth
method of determining the net income of a taxpayer, we find that
he had net incomes of P32,470.45, P67,751.19, P17,880.44, P52,709.11,
P254,813.56 and P40,155.80 during the respective years 1946, 1947,
1948, 1949, 1950, and 1951. In consequence, he underdeclared his
income by 227% for 1946, 564% for 1947, 95%, for 1948, 486% for 1949,
2,946% for 1950 and 490% for 1951. These substantial under
declarations of income for six consecutive years eloquently
177

demonstrate the falsity or fraudulence of the income tax return with


an intent to evade the payment of tax. Hence, the imposition of the
fraud penalty is proper (Perez vs. Court of Tax Appeals, G.R. No. L-
10507, May 30, 1958). (Emphasis supplied)

As could be readily seen from the above rationalization of the lower court, no
distinction has been made between false returns (due to mistake, carelessness
or ignorance) and fraudulent returns (with intent to evade taxes). The lower
court based its conclusion on the petitioner's alleged fraudulent intent to
evade taxes on the substantial difference between the amounts of net income
on the face of the returns as filed by him in the years 1946 to 1951 and the net
income as determined by the inventory method utilized by both respondents
for the same years. The lower court based its conclusion on a presumption that
fraud can be deduced from the very substantial disparity of incomes as
reported and determined by the inventory method and on the similarity of
consecutive disparities for six years. Such a basis for determining the existence
of fraud (intent to evade payment of tax) suffers from an inherent flaw when
applied to this case. It is very apparent here that the respondent Commissioner
of Internal Revenue, when the inventory method was resorted to in the first
assessment, concluded that the correct tax liability of Mr. Aznar amounted to
P723,032.66 (Exh. 1, B.I.R. rec. pp. 126-129). After a reinvestigation the same
respondent, in another assessment dated February 16, 1955, concluded that
the tax liability should be reduced to P381,096.07. This is a crystal-clear,
indication that even the respondent Commissioner of Internal Revenue with the
use of the inventory method can commit a glaring mistake in the assessment of
petitioner's tax liability. When the respondent Court of Tax Appeals reviewed
this case on appeal, it concluded that petitioner's tax liability should be only
P227,788.64. The lower court in three instances (elimination of two buildings in
the list of petitioner's assets beginning December 31, 1949, because they were
destroyed by fire; elimination of expenses for construction in petitioner's assets
as duplication of increased value in buildings, and elimination of value of house
and lot in petitioner's assets because said property was only given as collateral)
supported petitioner's stand on the wrong inclusions in his lists of assets made by
the respondent Commissioner of Internal Revenue, resulting in the very
substantial reduction of petitioner's tax liability by the lower court. The foregoing
shows that it was not only Mr. Matias H. Aznar who committed mistakes in his
report of his income but also the respondent Commissioner of Internal Revenue
who committed mistakes in his use of the inventory method to determine the
petitioner's tax liability. The mistakes committed by the Commissioner of Internal
Revenue which also involve very substantial amounts were also repeated
yearly, and yet we cannot presume therefrom the existence of any taint of
official fraud.

From the above exposition of facts, we cannot but emphatically reiterate the
well established doctrine that fraud cannot be presumed but must be proven.
As a corollary thereto, we can also state that fraudulent intent could not be
deduced from mistakes however frequent they may be, especially if such
mistakes emanate from erroneous entries or erroneous classification of items in
accounting methods utilized for determination of tax liabilities The predecessor
of the petitioner undoubtedly filed his income tax returns for "the years 1946 to
178

1951 and those tax returns were prepared for him by his accountant and
employees. It also appears that petitioner in his lifetime and during the
investigation of his tax liabilities cooperated readily with the B.I.R. and there is
no indication in the record of any act of bad faith committed by him.

The lower court's conclusion regarding the existence of fraudulent intent to


evade payment of taxes was based merely on a presumption and not on
evidence establishing a willful filing of false and fraudulent returns so as to
warrant the imposition of the fraud penalty. The fraud contemplated by law is
actual and not constructive. It must be intentional fraud, consisting of
deception willfully and deliberately done or resorted to in order to induce
another to give up some legal right. Negligence, whether slight or gross, is not
equivalent to the fraud with intent to evade the tax contemplated by the law.
It must amount to intentional wrong-doing with the sole object of avoiding the
tax. It necessarily follows that a mere mistake cannot be considered as
fraudulent intent, and if both petitioner and respondent Commissioner of
Internal Revenue committed mistakes in making entries in the returns and in the
assessment, respectively, under the inventory method of determining tax
liability, it would be unfair to treat the mistakes of the petitioner as tainted with
fraud and those of the respondent as made in good faith.

We conclude that the 50% surcharge as fraud penalty authorized under


Section 72 of the Tax Code should not be imposed, but eliminated from the
income tax deficiency for each year from 1946 to 1951, inclusive. The tax
liability of the petitioner for each year should, therefore, be:

1946 P 3,687.10
1947 13,288.38
1948 960.77
1949 8,918.85
1950 117,320.00
1951 7,684.00
P151,859.10

The total sum of P151,859.10 should be decreased by P96.87 representing the


tax credit for 1945, thereby leaving a balance of P151,762.23.

WHEREFORE, the decision of the Court of Tax Appeals is modified in so far as the
imposition of the 50% fraud penalty is concerned, and affirmed in all other
respects. The petitioner is ordered to pay to the Commissioner of Internal
Revenue, or his duly authorized representative, the sum of P151,762.23,
representing deficiency income taxes for the years 1946 to 1951, inclusive,
within 30 days from the date this decision becomes final. If the said amount is
not paid within said period, there shall be added to the unpaid amount the
surcharge of 5%, plus interest at the rate of 12% per annum from the date of
delinquency to the date of payment, in accordance with Section 51 of the
National Internal Revenue Code.

With costs against the petitioner.

G.R. No. L-19865 July 31, 1965


179

MARIA CARLA PIROVANO, etc., et al., petitioners-appellants,


vs.
THE COMMISSIONER OF INTERNAL REVENUE, respondent-appellee.

Angel S. Gamboa for petitioners-appellants.


Office of the Solicitor General for respondent-appellee.

REYES, J.B.L., J.:

This case is a sequel to the case of Pirovano vs. De la Rama Steamship Co., 96
Phil. 335.

Briefly, the facts of the aforestated case may be stated as follows:

Enrico Pirovano was the father of the herein petitioners-appellants. Sometime in


the early part of 1941, De la Rama Steamship Co. insured the life of said Enrico
Pirovano, who was then its President and General Manager until the time of his
death, with various Philippine and American insurance companies for a total
sum of one million pesos, designating itself as the beneficiary of the policies,
obtained by it. Due to the Japanese occupation of the Philippines during the
second World War, the Company was unable to pay the premiums on the
policies issued by its Philippine insurers and these policies lapsed, while the
policies issued by its American insurers were kept effective and subsisting, the
New York office of the Company having continued paying its premiums from
year to year.

During the Japanese occupation , or more particularly in the latter part of 1944,
said Enrico Pirovano died.

After the liberation of the Philippines from the Japanese forces, the Board of
Directors of De la Rama Steamship Co. adopted a resolution dated July 10,
1946 granting and setting aside, out of the proceeds expected to be collected
on the insurance policies taken on the life of said Enrico Pirovano, the sum of
P400,000.00 for equal division among the four (4) minor children of the
deceased, said sum of money to be convertible into 4,000 shares of stock of
the Company, at par, or 1,000 shares for each child. Shortly thereafter, the
Company received the total sum of P643,000.00 as proceeds of the said life
insurance policies obtained from American insurers.

Upon receipt of the last stated sum of money, the Board of Directors of the
Company modified, on January 6, 1947, the above-mentioned resolution by
renouncing all its rights title, and interest to the said amount of P643,000.00 in
favor of the minor children of the deceased, subject to the express condition
that said amount should be retained by the Company in the nature of a loan
to it, drawing interest at the rate of five per centum (5%) per annum, and
payable to the Pirovano children after the Company shall have first settled in
full the balance of its present remaining bonded indebtedness in the sum of
approximately P5,000,000.00. This latter resolution was carried out in a
Memorandum Agreement on January 10, 1947 and June 17, 1947.,
respectively, executed by the Company and Mrs. Estefania R. Pirovano, the
180

latter acting in her capacity as guardian of her children (petitioners-appellants


herein) find pursuant to an express authority granted her by the court.

On June 24, 1947, the Board of Directors of the Company further modified the
last mentioned resolution providing therein that the Company shall pay the
proceeds of said life insurance policies to the heirs of the said Enrico Pirovano
after the Company shall have settled in full the balance of its present remaining
bonded indebtedness, but the annual interests accruing on the principal shall
be paid to the heirs of the said Enrico Pirovano, or their duly appointed
representative, whenever the Company is in a position to meet said obligation.

On February 26, 1948, Mrs. Estefania R. Pirovano, in behalf of her children,


executed a public document formally accepting the donation; and, on the
same date, the Company through its Board of Directors, took official notice of
this formal acceptance.

On September 13, 1949, the stockholders of the Company formally ratified the
various resolutions hereinabove mentioned with certain clarifying modifications
that the payment of the donation shall not be effected until such time as the
Company shall have first duly liquidated its present bonded indebtedness in the
amount of P3,260,855.77 with the National Development Company, or fully
redeemed the preferred shares of stock in the amount which shall be issued to
the National Development Company in lieu thereof; and that any and all taxes,
legal fees, and expenses in any way connected with the above transaction
shall be chargeable and deducted from the proceeds of the life insurance
policies mentioned in the resolutions of the Board of Directors.

On March 8, 1951, however, the majority stockholders of the Company voted


to revoke the resolution approving the donation in favor of the Pirovano
children.

As a consequence of this revocation and refusal of the Company to pay the


balance of the donation amounting to P564,980.90 despite demands therefor,
the herein petitioners-appellants represented by their natural guardian, Mrs.
Estefania R. Pirovano, brought an action for the recovery of said amount, plus
interest and damages against De la Rama Steamship Co., in the Court of First
Instance of Rizal, which case ultimately culminated to an appeal to this Court.
On December 29, 1954, this court rendered its decision in the appealed case
(96 Phil. 335) holding that the donation was valid and remunerative in nature,
the dispositive part of which reads:

Wherefore, the decision appealed from should be modified as follows: (a)


that the donation in favor of the children of the late Enrico Pirovano of the
proceeds of the insurance policies taken on his life is valid and binding on
the defendant corporation; (b) that said donation, which amounts to a
total of P583,813.59, including interest, as it appears in the books of the
corporation as of August 31, 1951, plus interest thereon at the rate of 5 per
cent per annum from the filing of the complaint, should be paid to the
plaintiffs after the defendant corporation shall have fully redeemed the
preferred shares issued to the National Development Company under the
terms and conditions stared in the resolutions of the Board of Directors of
181

January 6, 1947 and June 24, 1947, as amended by the resolution of the
stockholders adopted on September 13, 1949; and (c) defendant shall
pay to plaintiffs an additional amount equivalent to 10 per cent of said
amount of P583,813.59 as damages by way of attorney's fees, and to pay
the costs of action. (Pirovano et al. vs. De la Rama Steamship Co., 96 Phil.
367-368)

The above decision became final and executory. In compliance therewith, De


la Rama Steamship Co. made, on April 6, 1955, a partial payment on the
amount of the judgment and paid the balance thereof on May 12, 1955.

On March 6, 1955, respondent Commissioner of Internal Revenue assessed the


amount of P60,869.67 as donees' gift tax, inclusive of surcharges, interests and
other penalties, against each of the petitioners-appellants, or for the total sum
of P243,478.68; and, on April 23, 1955, a donor's gift tax in the total amount of
P34,371.76 was also assessed against De la Rama Steamship Co., which the
latter paid.

Petitioners-appellants herein contested respondent Commissioner's assessment


and imposition of the donees' gift taxes and donor's gift tax and also made a
claim for refund of the donor's gift tax so collected. Respondent Commissioner
overruled petitioners' claims; hence, the latter presented two (2) petitions for
review against respondent's rulings before the Court of Tax Appeals, said
petitions having been docketed as CTA Cases Nos. 347 and 375. CTA Case No.
347 relates to the petition disputing the legality of the assessment of donees' gift
taxes and donor's gift tax while CTA Case No. 375 refers to the claim for refund
of the donor's gift tax already paid.

After the filing of respondent's usual answers to the petitions, the two cases,
being interrelated to each other, were tried jointly and terminated.

On January 31, 1962, the Court of Tax Appeals rendered its decision in the two
cases, the dispositive part of which reads:

In resume, we are of the opinion, that (1) the donor's gift tax in the sum of
P34,371.76 was erroneously assessed and collected, hence, petitioners are
entitled to the refund thereof; (2) the donees' gift taxes were correctly
assessed; (3) the imposition of the surcharge of 25% is not proper; (4) the
surcharge of 5% is legally due; and (5) the interest of 1% per month on the
deficiency donees' gift taxes is due from petitioners from March 8, 1955
until the taxes are paid.

IN LINE WITH THE FOREGOING OPINION, petitioners are hereby ordered to


pay the donees' gift taxes as assessed by respondent, plus 5% surcharge
and interest at the rate of 1% per month from March 8, 1955 to the date of
payment of said donees' gift taxes. Respondent is ordered to apply the
sum of P34,371.76 which is refundable to petitioners, against the amount
due from petitioners. With costs against petitioners in Case No. 347.

Petitioners-appellants herein filed a motion to reconsider the above decision,


which the lower court denied. Hence, this appeal before us.
182

In the instant appeal, petitioners-appellants herein question only that portion of


the decision of the lower court ordering the payment of donees' gift taxes as
assessed by respondent as well as the imposition of surcharge and interest on
the amount of donees' gift taxes.

In their brief and memorandum, they dispute the factual finding of the lower
court that De la Rama Steamship Company's renunciation of its rights, title, and
interest over the proceeds of said life insurance policies in favor of the Pirovano
children "was motivated solely and exclusively by its sense of gratitude, an act
of pure liberality, and not to pay additional compensation for services
inadequately paid for." Petitioners now contend that the lower court's finding
was erroneous in seemingly considering the disputed grant as a simple
donation, since our previous decision (96 Phil. 335) had already declared that
the transfer to the Pirovano children was a remuneratory donation. Petitioners
further contend that the same was made not for an insufficient or inadequate
consideration but rather it a was made for a full and adequate compensation
for the valuable services rendered by the late Enrico Pirovano to the De la
Rama Steamship Co.; hence, the donation does not constitute a taxable gift
under the provisions of Section 108 of the National Internal Revenue Code.

The argument for petitioners-appellants fails to take into account the fact that
neither in Spanish nor in Anglo-American law was it considered that past
services, rendered without relying on a coetaneous promise, express or implied,
that such services would be paid for in the future, constituted cause or
consideration that would make a conveyance of property anything else but a
gift or donation. This conclusion flows from the text of Article 619 of the Code of
1889 (identical with Article 726 of the present Civil Code of the Philippines):

When a person gives to another a thing ... on account of the latter's merits
or of the services rendered by him to the donor, provided they do not
constitute a demandable debt, ..., there is also a donation. ... .

There is nothing on record to show that when the late Enrico Pirovano rendered
services as President and General Manager of the De la Rama Steamship Co.
he was not fully compensated for such services, or that, because they were
"largely responsible for the rapid and very successful development of the
activities of the company" (Res. of July 10, 1946). Pirovano expected or was
promised further compensation over and in addition to his regular emoluments
as President and General Manager. The fact that his services contributed in a
large measure to the success of the company did not give rise to a
recoverable debt, and the conveyances made by the company to his heirs
remain a gift or donation. This is emphasized by the directors' Resolution of
January 6, 1947, that "out of gratitude" the company decided to renounce in
favor of Pirovano's heirs the proceeds of the life insurance policies in question.
The true consideration for the donation was, therefore, the company's
gratitude for his services, and not the services themselves.

That the tax court regarded the conveyance as a simple donation, instead of a
remuneratory one as it was declared to be in our previous decision, is but an
innocuous error; whether remuneratory or simple, the conveyance remained a
gift, taxable under Chapter 2, Title III of the Internal Revenue Code.
183

But then appellants contend, the entire property or right donated should not
be considered as a gift for taxation purposes; only that portion of the value of
the property or right transferred, if any, which is in excess of the value of the
services rendered should be considered as a taxable gift. They cite in support
Section 111 of the Tax Code which provides that —

Where property is transferred for less, than an adequate and full


consideration in money or money's worth, then the amount by which the
value of the property exceeded the value of the consideration shall, for
the purpose of the tax imposed by this Chapter, be deemed a gift, ... .

The flaw in this argument lies in the fact that, as copied from American law, the
term consideration used in this section refers to the technical "consideration"
defined by the American Law Institute (Restatement of Contracts) as "anything
that is bargained for by the promisor and given by the promisee in exchange
for the promise" (Also, Corbin on Contracts, Vol. I, p. 359). But, as we have seen,
Pirovano's successful activities as officer of the De la Rama Steamship Co.
cannot be deemed such consideration for the gift to his heirs, since the services
were rendered long before the Company ceded the value of the life policies
to said heirs; cession and services were not the result of one bargain or of a
mutual exchange of promises.

And the Anglo-American law treats a subsequent promise to pay for past
services (like one to pay for improvements already made without prior request
from the promisor) to be a nudum pactum (Roscorla vs. Thomas, 3 Q.B. 234;
Peters vs. Poro, 25 ALR 615; Carson vs. Clark, 25 Am. Dec. 79; Boston vs. Dodge,
12 Am. Dec. 206), i.e., one that is unenforceable in view of the common law
rule that consideration must consist in a legal benefit to the promisee or some
legal detriment to the promisor.

What is more, the actual consideration for the cession of the policies, as
previously shown, was the Company's gratitude to Pirovano; so that under
section 111 of the Code there is no consideration the value of which can be
deducted from that of the property transferred as a gift. Like "love and
affection," gratitude has no economic value and is not "consideration" in the
sense that the word is used in this section of the Tax Code.

As stated by Chief Justice Griffith of the Supreme Court of Mississippi in his well-
known book, "Outlines of the Law" (p. 204) —

Love and affection are not considerations of value — they are not estimable in
terms of value. Nor are sentiments of gratitude for gratuitous part favors or
kindnesses; nor are obligations which are merely moral. It has been well said
that if a moral obligation were alone sufficient it would remove the necessity for
any consideration at all, since the fact of making a promise impose, the moral
obligation to perform it."

It is of course perfectly possible that a donation or gift should at the same time
impose a burden or condition on the donee involving some economic liability
for him. A, for example, may donate a parcel of land to B on condition that the
latter assume a mortgage existing on the donated land. In this case the donee
184

may rightfully insist that the gift tax be computed only on the value of the land
less the value of the mortgage. This, in fact, is contemplated by Article 619 of
the Civil Code of 1889 (Art. 726 of the Tax Code) when it provides that there is
also a donation "when the gift imposes upon the donee a burden which is less
than the value of the thing given." Section 111 of the Tax Code has in view
situations of this kind, since it also prescribes that "the amount by which the
value of the property exceeded the value of the consideration" shall be
deemed a gift for the purpose of the tax. .

Petitioners finally contend that, even assuming that the donation in question is
subject to donees' gift taxes, the imposition of the surcharge of 5% and interest
of 1% per month from March 8, 1955 was not justified because the proceeds of
the life insurance policies were actually received on April 6, 1955 and May 12,
1955 only and in accordance with Section 115(c) of the Tax Code; the filing of
the returns of such tax became due on March 1, 1956 and the tax became
payable on May 15, 1956, as provided for in Section 116(a) of the same Code.
In other words, petitioners maintain that the assessment and demand for
donees' gift taxes was prematurely made and of no legal effect; hence, they
should not be held liable for such surcharge and interest.

It is well to note, and it is not disputed, that petitioners-donees have failed to file
any gift tax return and that they also failed to pay the amount of the
assessment made against them by respondent in 1955. This situation is covered
by Section 119(b) (1) and (c) and Section 120 of the Tax Code:

(b) Deficiency.

(1) Payment not extended. — Where a deficiency, or any interest


assessed in connection therewith, or any addition to the taxes provided
for in section one hundred twenty is not paid in full within thirty days from
the date of the notice and demand from the Commissioner, there shall be
collected as a part of the taxes, interest upon the unpaid amount at the
rate of one per centum a month from the date of such notice and
demand until it is paid. (section 119)

(c) Surcharge. — If any amount of the taxes included in the notice and
demand from the Commissioner of Internal Revenue is not paid in full
within thirty days after such notice and demand, there shall be collected
in addition to the interest prescribed above as a part of the taxes a
surcharge of five per centum of the unpaid amount. (sec. 119)

The failure to file a return was found by the lower court to be due to reasonable
cause and not to willful neglect. On this score, the elimination by the lower
court of the 25% surcharge is ad valorem penalty which respondent
Commissioner had imposed pursuant to Section 120 of the Tax Code was
proper, since said Section 120 vests in the Commissioner of Internal Revenue or
in the tax court power and authority to impose or not to impose such penalty
depending upon whether or not reasonable cause has been shown in the non-
filing of such return.
185

On the other hand, unlike said Section 120, Section 119, paragraphs (b) (1) and
(c) of the Tax Code, does not confer on the Commissioner of Internal Revenue
or on the courts any power and discretion not to impose such interest and
surcharge. It is likewise provided for by law that an appeal to the Court of Tax
Appeals from a decision of the Commissioner of Internal Revenue shall not
suspend the payment or collection of the tax liability of the taxpayer unless a
motion to that effect shall have been presented to the court and granted by it
on the ground that such collection will jeopardize the interest of the taxpayer
(Sec. 11, Republic Act No. 1125; Rule 12, Rules of the Court of Tax Appeals). It
should further be noted that —

It has been the uniform holding of this Court that no suit for enjoining the
collection of a tax, disputed or undisputed, can be brought, the remedy
being to pay the tax first, formerly under protest and now without need of
protect, file the claim with the Collector, and if he denies it, bring an
action for recovery against him. (David v. Ramos, et al., 90 Phil. 351)

Section 306 of the National Internal Revenue Code ... lays down the
procedure to be followed in those cases wherein a taxpayer entertains
some doubt about the correctness of a tax sought to be collected. Said
section provides that the tax, should first be paid and the taxpayer should
sue for its recovery afterwards. The purpose of the law obviously is to
prevent delay in the collection of taxes, upon which the Government
depends for its existence. To allow a taxpayer to first secure a ruling as
regards the validity of the tax before paying it would be to defeat this
purpose. (National Dental Supply Co. vs. Meer, 90 Phil. 265)

Petitioners did not file in the lower court any motion for the suspension of
payment or collection of the amount of assessment made against them.

On the basis of the above-stated provisions of law and applicable authorities, it


is evident that the imposition of 1% interest monthly and 5% surcharge is justified
and legal. As succinctly stated by the court below, said imposition is
"mandatory and may not be waived by the Commissioner of Internal Revenue
or by the courts" (Resolution on petitioners' motion for reconsideration, Annex
XIV, petition). Hence, said imposition of interest and surcharge by the lower
court should be upheld.

WHEREFORE, the decision of the Court of Tax Appeals is affirmed. Costs against
petitioners Pirovano.

You might also like